You are on page 1of 215

ANATOMY

QUICK REVISION NOTES


BASIC CONCEPT, TRICKS AND •• Legs together with toes in front
MAGIC OF ANATOMY
Different types of layers of body superficially to
deep: -
•• Roof :-
–– Skin
–– Superficial fascia (fat , cutaneous nerves &
vessels)
–– Deep fascia (includes collagen fibers)
•• Floor :-
1. Supine - lying on back - Cardiothoracic surgeries
–– Muscles
2. Prone - lying on abdomen - Spine or back surgeries
–– Bone
3. Lithotomy - patient lying on the back with both
Transverse dissection diagram feet supported with footrest.
•• Perineum area is exposed.
•• For
Obstetric-
gynaecological
procedures
and
Genito-
Urinary
surgeries.

Vein – thin walled & collapsing


Artery – thick walled & recoil
Nerve – no lumen, solid cord

•• All neurovascular bundle of our body have


Q
sequence as vein-artery-nerve except -
–– 1st Intercostal space
–– Popliteal fossa
4. Lateral decubitus - lie on one side of the body -
•• HILTON’S LAW: - Hilton observed that Best for ear surgeries.
nerves supplying the MUSCLE also innervate
the SKIN overlying the muscle and the JOINT
over which that muscle acts.

Position, Planes And Terminiology


Anatomical position
•• Body is erect
•• Eyes looking forward
•• Hands on side with palms directed forward
2
Cerebellum Quick Revision Notes

Planes
1. Mid-sagittal plane - Plane divides the plane into two equal halves.
2. Sagittal plane - Any plane parallel to mid-sagittal plane.
3. Coronal/Frontal Plane - Divides the plane into front & back.
4. Transverse/Horizontal plane - Divides plane into upper and lower parts, parallel to the ground.
5. Oblique plane - Any plane making angle with the ground

•• Proximal - Near to trunk.

JOINTS

Gliding:-
•• Flat surfaces of two bones glide across each other Gliding occurs between
–– Carpals
3
Anatomy

–– Articular processes of vertebrae


•• Angular movement:- movement in which there
–– Tarsals
is a change in angle
(Carpal bones mnemonic :- She Looks Too Pretty Try
a. Flexion
To Catch Her)
b. Extension
c. Adduction
d. Abduction
e. Internal rotation
f. External rotation
g. Circumduction
h. Supination
i. Pronation

j. Unlocking: Popliteal
k. Locking : Quadricops Femoris
4
Cerebellum Quick Revision Notes

l. Invertor of foot:- tibialis Anterior &


Posterior

m. Evertor of foot:- Peroneus longus & brevi •• Fusiform muscle :- biceps


•• Basic rules of muscle identification in cadaveric
images:-

2. Oblique muscle
–– Pennate
ƒƒ Unipennate :- 1st & 2nd lumbrical ,
Q
Palmar interosseous

ƒƒ Bipennate:- 3rd & 4th lumbrical, all


1. Parallel muscles dorsal interosseous
Q
•• Strap muscle fiber
–– Sternohyoid
–– Sternothyroid
–– Omohyoid
–– Longest muscle of body :- Sertorius a.k.a.
honeymoon muscle / tailor’s muscle ƒƒ Multipennate:- middle fiber of deltoid,
•• Strap muscle fiber with tendinous subscapularis muscle
intersections: Rectus abdominis
•• Quadrilateral muscle fiber
–– Thyrohyoid
5
Anatomy

PECTORAL REGION

ƒƒ Circumpennate

PECTORAL REGION
Bones of pectoral region
3. Cruciate Clavicle:-
Which crossing each other, ex:- Sternocleidomastoid,
•• It is aka collarbone / beauty bone / key bone
masseter (strongest muscle of body)
•• Clavicle parts:- medial 2/3rd & lateral 1/3rd
(Mc fracture location) < Medial 3/5th & Lateral
2/5th (GRAY'S 42nd Edition update)
•• Clavicle is the only long bone having 2
Q
ossification center
•• Clavicle is the only long bone which pierced
by the nerve – Intermediate supraclavicular
4. Twisted / spiral :- pectoralis nerve
Q

majorQ
6
Cerebellum Quick Revision Notes

Scapula:-
•• Coracoid process located in infra clavicular
groove / fossa aka delto- pectoral groove
•• Above the spine of Scapula there is supraspinous
Fossa & below infraspinous Fossa.

Muscles of Pectoral region


•• Pectoralis major & minor
•• Serratus anterior aka boxer’s muscle because
it has punching & pushing ability & it’s supplied
by bell’s nerve aka long thoracic nerve,
Q
responsible for winging of scapula .
•• Subclavius
•• Pectoralis minimus
•• Winging of Scapula:- In this, Medial border
•• Rectus sternalis
of Scapula elevated and person is not able to
Q
lift weight or do lifting exercises , serratus
anterior muscles responsible for this.
•• Forward movement of Scapula known as
protraction of scapula

Humerus / funny bone:-


•• Parts:- head, neck, greater & smaller tubercle
(intertubercular groove between them), Medial
& Lateral epicondyle
•• Below lesser tubercle there is Inter tubercular
sulcus aka bicipital groove (from here, long head
of biceps with his synovial sheets & ascending AXILLA (AXILLARY ARTERY)
branch of anterior circumflex artery is passing)
It is a truncated pyramidal shaped space on
•• At the Medial epicondyle, the ulnar nerve is the lateral side of the chest wall and medial
passing. to the upper end of the humerus.
7
Anatomy

Biceps - Short head is related to Coracobrachialis


muscle.

Contents of Axilla
V - Axillary Vein
A - Axillary Artery
N - Axillary Nerve
Lymph nodes - Axillary group of lymph nodes

Axillary Artery
BoundariesQ
Continuation of Subclavian artery.
1. Lateral wall - Intertubercular Sulcus, Biceps
2. Medial wall - Serratus anterior
3. Anterior wall - Pectoralis major & Pectoralis
minor
4. Floor - Skin and fascia
5. Apex - directed towards the neck
6. Posterior wall - Coracobrachialis, Teres major
& Scapula

Mnemonic : “STA, ATA, LTA → ACHA, PCHA, SSA”


KEY Muscle of Axillary region - Pectoralis Minor

Note:
Mnemonic for Branches of axillary artery which
supply to breast - SALI
S - Superior Thoracic Artery (STA)
A - Acromiothoracic Artery (ATA)
L - Lateral Thoracic Artery (LTA)
I - Internal Mammary Artery (IMA)
8
Cerebellum Quick Revision Notes

Brachial Plexus anatomical relation with axillary artery).

Note-Sometime contribution from other segments


also present:
•• C4 - Pre-fixed brachial plexus
•• T2 - Post-fixed brachial plexus
•• Thickest cord of Brachial Plexus - Posterior cord
•• Thickest nerve of Brachial Plexus - Radial
nerve
About cords -
•• All posterior divisions unite to form →
Posterior Cord (Radial nerve)
•• Anterior divisions of Upper and Middle trunks
Parts of brachial plexus - (Ramu Tailor Drinks Cold
Beer)/ RTDCB → Lateral Cord (Median nerve)
•• Roots - Part of nerve attached to the spinal •• Anterior divisions of Lower trunk → Medial
cord. 5 in number → C5-C8 and T1. Cord (Ulnar nerve)
•• Trunk - 3 in numbers → Upper, Middle & Lower (Note - Main continuation of respective Cords)
Trunk.
•• Divisions - 1 anterior and 1 posterior from each Branches from Brachial Plexus
Trunk.
•• Cords - 3 in number → Medial, Lateral &
Posterior Cord (named according to the

1. Nerve from roots


•• Dorsal Scapular nerve
•• Long Thoracic nerve (aka Nerve of Bell - C5-C7 → to Serratus Anterior)
9
Anatomy

2. Nerves arises from upper trunk (only


Upper Trunk gives branches) (C5-C6)
•• Nerve to Subclavius muscle
•• Suprascapular nerve

3. No Branches from Divisions


Q

4. Branches from Lateral Cord (Mnemonic


LML) (C5-C6)
•• Lateral root of Median nerve
•• Musculocutaneous nerve
Defects -
•• Lateral Pectoral nerve
•• Adducted arm - defect in Deltoid
5. Branches from Posterior Cord (Mnemonic
•• Medial Rotation of arm - defect in Teres
ULTRA)
minor
•• U - Upper subscapular nerve (C5-C7)
•• Pronated hand - defect in Biceps,
•• L - Lower subscapular nerve (C5-C7) Brachioradialis and Radial nerve weakness.

•• T - Thoracodorsal nerve (nerve to Latissimus •• Extended elbow - defect in Coracobrachialis,


dorsi) (C6-C8) Biceps, Brachioradialis.

•• R - Radial nerve (C5-T1)


•• A - Axillary nerve (C5-C7)

6. Branches from Medial Cord (Mnemonic


M4U)
•• M - Medial root of Median nerve
•• M - Medial cutaneous nerve of Arm
Aka Waiter’s/Porter’s Tip hand OR Policeman’s Tip
•• M - Medial cutaneous nerve of Forearm hand.
•• M - Medial Pectoral nerve B. KLUMPKE’s Paralysis
•• U - Ulnar nerve Causes -
•• Overstretching of Arm/Axillary area.
Clinical Integration
•• Pulling of hand during delivery.
A. ERB’s Paralysis Defects -
Injury between Head and shoulder → involve Upper Ulnar Claw hand - Ulnar nerve damage
Q
trunk (C5-C6 involve) Horner’s Syndrome - due to T1 damage

Causes -
•• Fall with a stretched arm or on shoulder.
•• Shoulder dystocia during delivery.

Erb’s point - Junction of 6 nerves


10
Cerebellum Quick Revision Notes

BACK

Triangle of Auscultation
Less muscles - easily get LUNGS sounds
•• Lateral border - medial border of Scapula
•• Medial border - Trapezius
Dissection / Surgery •• Base - Latissimus dorsi
Superficial to deep
Skin → Superficial fascia → Deep fascia → Muscles
→ Bones

Muscles
1. Trapezius - by XI / Accessory spinal
nerve
•• Shrugging of shoulder → Upper fibres
•• Retraction of scapula → Middle fibres
•• Overhead abduction → Lower fibres

2. Latissimus Dorsi - by Thoracodorsal


nerve

3. Levator scapulae - by Dorsal Scapular


nerve

4. Rhomboid Major & Minor - by Dorsal


Scapular nerve NOTE -
Winging of scapula due to paralysis of → Serratus
anterior > Trapezius > Rhomboids

SHOULDER/SCAPULAR REGION
Muscles
Deltoid - Give rounded contour to the shoulder
Rotator Cuff (SITS) - Supraspinatus, Infraspinatus,
Q
Teres minor & Subscapularis .
11
Anatomy

Bursa of the Body


Largest Bursa - Iliopsoas Bursa > Subacromial
Bursa

Uses -

•• Nerve supply of deltoid : Axillary nerve •• Act as Shock absorber.


Q
(related to Surgical neck of Humerus)
•• Provide proper joint movement.
•• Teres Minor - supplied by nerve to teres minor
(branch of Axillary Nerve → Pseudoganglion Clinical - Inflammation is called Bursitis.
present).
Subacromial Bursitis → Positive Dawbarn’s Sign
•• Teres Major - by Lower Subscapular nerve.
(Pain disappear on Abduction of arm)

Regimental Badge Anaesthesia


•• Due to injury to Axillary Nerve. (It’s posterior
branch is sensory to upper part of the lateral
arm area via cutaneous branch known as
regimental badge)
•• Cause - During Intramuscular injection or
(Mnemonic for muscles in Bicipital Groove - Lady injury at the surgical neck of the humerus.
Between the 2 Majors)
•• Subscapularis muscle (Multipennate) - Hybrid ARM
muscle → from upper and lower Subscapular
nerve.
•• Muscle least damage in Rotator Cuff injury
- Subscapularis → hence called as Forgotten
muscle of the Rotator Cuff.
•• Muscle most commonly damaged in Rotator
Q
Cuff injury - supraspinatus
12
Cerebellum Quick Revision Notes

Radial Radial groove Extensors Wrist drop


of Upper
limb
Ulnar Medial Small Ulnar/Partial
epicondyle muscles of Claw hand
hand

Median Supracondylar Flexors of - Median Claw


Area Wrist hand
- Benediction
hand
deformity

Student’s/Miner’s Elbow - Inflammation of


Olecranon Bursa (Olecranon Bursitis)

Ortho-Radio-Anat Integration
Humerus connected to 4 nerves

Nerve Site of injury Muscles Clinical


Affected Features
Axillary Surgical neck Deltoid - Regimental
of Humerus Teres minor batch
anaesthesia
- 0-90 degree
Abduction
weakness
- Loss of
rounded
contour of
shoulder
- medially
rotated arm
13
Anatomy

Supracondylar humerus Fracture


Radial Artery damage → Volksmann’s Ischaemia → Gun Stock deformity

FOREARM

A. Anterior Compartment A.1. Superficial Muscles:


1. Pronator teres (PT)
2. Flexor carpi radialis (FCR)
3. Palmaris longus (PL) → used for tendon grafting
operation of upper limb.
4. Flexor digitorum superficialis (FDS)
5. Flexor carpi ulnaris(FCU) → Pisiform bone ossify
in this muscle.
Nerve supply : All are supplied by the Median nerve
Q
(Labourer nerve) except Flexor carpi ulnaris .
14
Cerebellum Quick Revision Notes

A.2. Deep Muscles: B. Posterior Compartment


1. Quadratus pronator (QP) B.1. Superficial Group
2. Flexor digitorum profundus (FDP) (Hybrid
1. Anconeus → responsible
Muscle) → give rise to Lumbricals
for screwing movement
3. Flexor pollicis longus (FPL) 2. Brachioradialis → end at Direct branch from
Nerve supply : (All supplied by Anterior Interosseus styloid process of radius Radial nerve
nerve (Deep branch of Median nerve) except medial 3. Extensor carpi radialis
½ of FDP (supplied by ulnar nerve)
Q
longus (ECRL)
4. Extensor carpi radialis
brevis (ECRB)
by Posterior interosseous
5. Extensor Digitorum (ED)
nerve (PIN) (deep) Branch
6. Extensor digit minimi
of radial nerve
(EDM)
7. Extensor Carpi Ulnaris

B.2. Deep Group


•• Supplied by Posterior interosseous nerve (PIN)

1. Abductor pollicis brevis


(AbPL)
2. Extensor Pollicis Brevis Boundaries of anatomical
(EPB) snuff box
3. Extensor Pollicis longus
(EPL)
4. Supinator → pierce by posterior interosseous nerve
5. Extensor Indicis

•• About FDP - Hybrid muscle


•• Medial half - Ulnar nerve
•• Lateral half - Anterior Interosseous nerve >
Median nerve

Cubital Fossa
•• Triangular, muscular depression in front of the
elbow.
Importance of cubital fossa -
1. Measurement of Blood Pressure
2. Biceps jerk reflex
15
Anatomy

3. Intravenous injection site •• Base - Imaginary line joining Medial and Lateral
epicondyle.
•• Roof - Skin, superficial and Deep fascia.

Content (medial to lateral → MBBR)Q


1. Median nerve
2. Brachial artery
3. Bicipital aponeurosis
4. Radial nerve (Superficial nerve) → emerges
between Brachioradialis & Pronator teres.

Floor of Cubital Fossa (BSF)Q


B - Brachialis
BoundariesQ
•• Lateral - medial border of Brachioradialis. S - Supinator

•• Medial - lateral border of Pronator teres. F - Floor

•• Apex - meeting point of Pronator teres and


Brachioradialis. HAND
Short intrinsic muscles of hand → 20 in numbers.

•• Musician nerve - Ulnar nerve


•• Labourer’s nerve - Median nerve
•• Eye of the hand - Median nerve
•• Dupuytren's contracture- Fibrosis of palmar
aponeurosis
•• Most common site - Medial half
•• Most common finger - Ring finger
16
Cerebellum Quick Revision Notes

Anat - FMT Integration Palmar interosseiQ


Wrist Cut Injury •• Unipennate - 4/3 in number → by Ulnar nerve
Structure cut - all 5 structures above Flexor •• Function - Adduction of fingers
Retinaculum
•• Middle finger has no Palmar interossei
1. Palmaris longus tendon
•• Test - Card test
2. Ulnar nerve and artery
3. Palmar cutaneous branch of ulnar nerve and Dorsal interosseiQ
flexor retinaculum •• Bipennate - 4 in number → by Ulnar nerve
4. Palmar cutaneous branch of median nerve •• Function - Abduction of fingers
5. Palmar aponeurosis •• 2 Dorsal interossei in Middle finger
In case of suicidal cut injury Hesitation Marks/ •• Test - Egawa test
Tentative Marks.

Lumbricals

ARTERIES & VEINS OF UPPER


LIMB
Artery of upper limb
Branches of arch of Aorta (BSC)

•• 1st and 2nd Lumbricals - Unipennate → Median B - Brachiocephalic artery (Right) → give Right
Nerve. Subclavian and common carotid artery.
•• 3rd and 4th Lumbricals - Bipennate → Ulnar S - Subclavian artery (Left) → make Axillary artery
nerve.
C - Common carotid artery (Left)
•• Function - Flexion at metacarpophalangeal
Q
Joints extension at Interphalangeal Joints .
•• Test → Pen holding position (Babaji ka thullu
position)
•• Complete Claw hand → injury of both Ulnar
and Median nerve.

Palmar & Dorsal Interossei

Subclavian artery branches : (VITamin CD)


Q
17
Anatomy

It is divided by scalenus anterior muscle into 3 parts:

1st part -
Vertebral artery → Lateral Medullary syndrome
Internal mammary artery → use for CABG
Thyrocervical branch → Suprascapular artery,
Inferior thyroid artery & Transverse cervical artery
(Mnemonic - SIT).

2nd part -
Costocervical trunk (only one branch)

3rd part -
Dorsal scapular artery (only one branch)

Reverse Allen’s test


•• Instead of Ulnar artery, release Radial artery.
(for patency of Deep arch)

Veins of Upper limb

•• Main artery of the thumb - Princeps pollicis Important veins of Upper limb
artery 1. Dorsal venous arch
•• Main artery of Index fingers - Radialis indicis 2. Cephalic vein
3. Basilic vein
Allen’s testQ
4. Medial cubital vein (Antecubital vein) → Best vein
•• Make a fist → pallor occur → compress both for Intravenous Infusion (as Fixed and easily
radial and ulnar artery → open the fist → seen)
Release Ulnar artery 1st →
5. Median vein of forearm
•• If redness, then ulnar artery patency is present.
(for patency of Superficial arch)
18
Cerebellum Quick Revision Notes

NERVE OF UPPER LIMB Dermatomes of hand


•• Sensory branch supplying the Lateral 3 & half
1. Median nerve :-
of Palmar & dorsal side(known as eye of hand)
•• Have nerve root of C5 to T1 ( C5, C6, C7 :- and nail beds
Lateral root, C8, T1 :- medial root)
•• Motor branch supplies 5 muscles of hand :- all
•• This nerve gives no branch in the arm 3 thenar muscles except adductor pollicis & 1st
•• Then this nerve passes in between the two & 2nd lumbrical
head of pronator teres Muscle (pronator teres –– Different signs of medial nerve injury:-
syndrome - if this nerve compress here) (CAP-BPT)
•• In forearm it is divided into Superficial branch C:- Carpal tunnel syndrome
& deep branch (aka anterior interosseous
A:- Ape thumb deformity
nerve)
P:- Pointing index/ Oschner’s class test
•• Deep branch supply all deep flexor Muscle
(flexor pollicis longus, pronator quadratus & B:- Benediction hand / pope hand deformity
medial Half of flexor digitorum profundus) P:- Phalen’s sign
•• At the wrist joint the superficial branch T:- Tinel sign
passes through the carpal tunnel below the
flexor retinaculum. Here injury to this nerve OK sign:- AIN > median
known as carpal tunnel syndrome (commonly
seen with connective tissue disorder as RA,
myxoedema and computer worker)
19
Anatomy

Treatment of tunnel syndrome •• At the wrist, it passes through Guyon’s


tunnel:- Compression here known as Guyon
•• Exercise Q
canal syndrome .
•• Multivitamins
•• Sensory branch Supplies medial 1½ of palm &
•• Painkiller - NSAIDs medial 2½ of dorsal hand.
•• Steroid •• Motor branch supplies all hypothenar muscles
•• Surgery & one thenar muscle [adductor pollicis] (aka
graveyard of Ulnar nerve), all Palmar & dorsal
2. Ulnar nerve - Q
interossei .
•• It is the branch of the medial cord of brachial
Ulnar nerve Tests (ABCDEFGH)
plexus, nerve root is C7, C8 & T1.
•• A:- A/Ulnar nerve
•• Runs medially to the axillary artery in the •• B:- Book test
axilla.
•• C:- Card test (positive in Palmar interossei
•• Passes through medial epicondyle. damage)

•• Ulnar nerve getting thickened in leprosy •• D:- aDDuctor pollicis test


behind medial epicondyle. •• E:- Egawa test( positive in dorsal interossei
damage)
•• The nerve passes through 2 heads of flexor
carpi ulnaris (here compression of nerve known •• F:- Froment test
as cubital tunnel syndrome) & supplied FCU & •• G:- Guyon’s tunnel syndrome
Q
medial ½ of flexor digitorum profundus .
•• H:- Handlebar palsy
20
Cerebellum Quick Revision Notes

NERVE OF UPPER LIMB - 2 –– Anconeus


•• Below radial groove, it gives 4 branches:-
3. Radial nerve:- (BEBE)
•• It is the thickest nerve of brachial plexus & it –– Brachialis
is continuation of thickest cord of B. Plexus. –– External carpi radialis longus

•• Root value:- C5, C6, C7, C8, T1 . These all –– Brachioradialis


roots make 1 Posterior cord which continue as –– Elbow joint
a radial nerve. •• At forearm radial nerve divides into
•• This nerve passes behind the humerus through –– Superficial branch:- runs towards wrist
radial groove & coming in front at cubital fossa. and makes roof of Anatomical snuff box &
gives cutaneous branches to the lateral 2½
•• It will give rise to 3 branches Above radial dorsum of hand except nail beds.
groove or axilla :- (LMP)
–– Deep branch/ PIN:- pierces supinator
–– Long head of triceps muscle
–– Medial head of triceps
Clinical
–– posterior cutaneous nerve of arm
•• Crutch palsy
•• Gives 5 branches (3 muscular) in radial groove
(spiral groove):- (MLA) –– If fracture at axilla (loss of extension of
elbow, wrist and fingers)
–– Medial head of triceps
–– Lateral head of triceps

–– if fracture at spiral groove (loss of extension at wrist and fingers )


–– if fracture below radial groove (loss of extension of fingers)
•• Saturday night palsy
•• Wrist drop
•• Finger drop
•• Honeymoon palsy
21
Anatomy

Anatomical snuff box –– Medial/Posterior border:- extensor pollicis


longus
–– Roof:- Skin, Superficial fascia (Cephalic vein
– site for IV inj., cutaneous branch of radial
nerve – cause wrist watch neuropathy), deep
fascia.
–– Floor:- Styloid process of radius , scaphoid
bone (2nd Mc bone getting AVN [Mc is
neck of femur, 3rd Mc is talus] → glass
holding cast use in scaphoid fracture),
trapezium, base of 1st metacarpal bone

–– Inflammation to this tendon leads to De


•• Content:- radial artery
Quervain’s tenosynovitis:- for diagnosis
•• Boundaries:- of this, we can perform a finkelstein test.
–– Lateral/anterior border:- abductor pollicis
longus & extensor pollicis brevis
22
Cerebellum Quick Revision Notes

ANTERIOR COMPARTMENT OF
THIGH
Bones of Lower limb

PSM-Anat-Pedia Integration
•• Vastus Lateralis - site for Intramuscular
injection during Vaccination.

•• Nerve of anterior compartment - Femoral


nerve
•• All nerves by Posterior divisions of femoral
nerve except
–– SARTORIUS - Anterior division of femoral
nerve
•• Aka Tailor’s or Palthi muscle
•• Movements - Flexion at knee and hip +
Abduction & lateral rotation of thigh.
•• Also Abduction of hip →opens perineum
area for sexual intercourse → hence, aka
Honeymoon muscle.
–– Footballer’s Muscle or Kick muscle or
Locking muscle
–– Extension at knee → Quadriceps Femoris

•• Most common dislocation of hip joint - Posterior


•• Therefore, Most common nerve affected is
the Sciatic nerve.

•• Great saphenous vein - Pierces the Cribriform


fascia by making an opening called as saphenous
opening and receive 3 tributaries:
23
Anatomy

1. Superficial external pudendal vein •• Femoral Ring → Uppermost part of femoral


canal.
2. Superficial epigastric vein
•• Femoral fossa - Depression over Femoral
3. Superficial circumflex iliac vein
septa (Fat over the Femoral canal).
•• Skin over the femoral triangle is supplied by
- Femoral branch of the Genito-femoral nerve.
•• Lateral cutaneous nerve of thigh → Meralgia
paresthetica
•• Cause : Compression under Inguinal ligament
Injury during injection or trauma.

Surgery-Anat Integration
A. Femoral Hernia
1. Wider pelvis
2. Smaller vessels
•• Femoral hernia reduction - By Cutting of
Lacunar ligament.
Femoral Triangle B. Inguinal Hernia → more common in males.
•• Triangular Muscular depression below inguinal
•• External Obliques modifications:
ligament.
(Mnemonic - LIP)
•• Floor (Mnemonic - APPI) - Adductor longus,
1. Lacunar Ligament
Pectineus, Psoas major tendon & Iliacus.
2. Inguinal Ligament
•• Lateral boundary - Medial border of
sartorius 3. Pectineal Ligament

•• Medial boundary - Medial border of Adductor MEDIAL & POSTERIOR


longus COMPARTMENTS OF THIGH
Content - (Medial to Lateral → VAN)
Medial Compartment
V - Femoral vein •• Muscles - Adductor muscles →Adduction of
A - Femoral artery thigh
•• All are Supplied by → Obturator Nerve
N - Femoral nerve
•• 5 muscles : Mnemonic for adductors - LBW/M
•• Femoral sheath - Deep fascia modification
1. Adductor longus
around femoral vessels.
2. Adductor brevis
–– Contents : Femoral vein, artery and
3. Adductor magnus → also by Sciatic nerve [Ischial
Lymph nodes (Not Femoral Nerve). Q
Head] (Hybrid muscle) .
•• Femoral canal → medial most part of 4. Pectineus → also by femoral nerve (Hybrid
Femoral sheath (contain Deep Inguinal Lymph muscle).
node- → aka Lymph nodes of Rossenmuller 5. Gracilis
and Cloquet).
24
Cerebellum Quick Revision Notes

FMT-Anatomy Integration
Q
Gracilis → also known as Anti -rape muscle/Custodian
of virginity.
•• Smooth and fragile → easily break/tear during
opposite forces.
•• Use to assess cases of rape and forced sexual
offences.

Gluteal Region
Muscles:
1. Gluteus maximus
2. Gluteus medius Anat-Medicine-Paedia- Ortho Integration
3. Gluteus minimus •• Duchenne Muscular Dystrophy: GOVER’S SIGN
4. Piriformis → Key muscle of Gluteal region
5. Obturator internus - related with 2 gamelli
Superior gamelli
Inferior gemelli
6. Quadratus femoris

•• Gluteus Maximus •• Defect in Gluteus Muscle (Paralysis)


•• Origin - Posterior gluteal line, Area behind •• Nerve involve - Inferior Gluteal nerve
posterior gluteal line, outer lip of the iliac
crest & adjacent surface of sacrum/coccyx. Anat-Ortho
•• Insertion - Into Gluteal tuberosity (¼th) & IntegrationQ
Ilio-tibial tract (¾th). Trendlenberg’s Sign:
Normal = During elevation
•• Movement - Chief extensor of Hip joint → help of one limb → Gluteus
from sitting to standing position. medius, minimus & Tensor
fascia lata of opposite side
•• Nerve - Inferior Gluteal nerve contract → Pulling/Elevate
•• Gluteus medius and minimus the opposite side
ASIS/Pelvis → Preventing
•• Nerve supply : Superior gluteal nerve Sagging of Pelvis.
•• Movement - Abduction of the hip joint. Superior Gluteal nerve
injury → Sagging of Pelvis
occurs
i.e. Positive
Trendelenburg’s sign.
Gait is known as
In case of Unilateral palsy
- Lurching gait
In case of Bilateral palsy -
Waddling gait
25
Anatomy

BACK OF THIGH & POPLITEAL •• Lower part of ischial tuberosity divides by


FOSSA longitudinal ridge into inferolateral {gives origin
to add Magnus (ischial head)} & intermedial
All back of thigh muscles are having same
part (known as ischial bursa).
•• Origin (from ischial tuberosity)
•• Ischial bursitis is known as weaver’s bottom.
•• Insertion (at bone of leg)
•• Nerve supply (sciatic nerve)
•• Action (runner’s action)
–– Extension at hip
–– Flexion at knee
So, they combinedly known as hamstring muscles
(includes semitendinosus, semimembranosus, long
head of biceps femoris, add magnus)

•• Long head of biceps femoris originates from


ischial tuberosity.
26
Cerebellum Quick Revision Notes

•• Boundaries of popliteal fossa:- •• Iliohypogastric & ilioinguinal L1: situated at


the posterior surface of kidney.
–– Supero-lateral - biceps femoris
–– Supero-medial - semitendinosus & semi •• Cremaster reflex (Genito femoral nerve L1
membranes L2 is involved) - when we scratch at Inner
part of thigh then due to this reflex, testis
–– Infero-lateral - lateral head of gastroc-
nemius elevated.

–– Infero-medial - medial head of gastroc- •• Compression of Lateral cutaneous nerve of the


nemius (ossification of bone here known as thigh causes abnormal sensation characterized
Fabella) by tingling, numbness and burning pain in the
•• Content of popliteal fossa (medial to lateral outer part of the thigh as known as Meralgia
- Artery → vein → nerve (AVN) paresthetica.
–– Tibial nerve •• Obturator nerve have root value of L 234
–– Common peroneal nerve ventral division

–– Popliteal vein
–– Popliteal artery
–– Popliteal lymph nodes

NERVES OF LOWER LIMB


ANATOMY
Lumbar plexus
27
Anatomy

1. Femoral nerve

•• Longest cutaneous nerve of the body is the Saphenous nerve.


2. Obturator nerve 3. Sciatic nerve
28
Cerebellum Quick Revision Notes

Bumper’s fracture

•• Superior Gluteal Nerve (SGN L4,L5,S1)


supplies to gluteus medius, minimus and
tensor fascia lata(damage leads to positive
Q
Trendelenburg sign) .
•• Inferior Gluteal Nerve (IGN L5,S1,S2)
supplies to gluteus maximus.
•• Tibial nerve root value is L4, L5, S1 S2 S3.
•• Common Peroneal Nerve (CPN) root value is L4-5
and S1-2 and take a round at the neck of fibula.
•• Deep Peroneal Nerve (DPN) supplies the
anterior compartment of the leg.
•• Foot drop - Injury of CPN > DPN > SCIATIC
Q
NERVE . LEG COMPARTMENT
•• Superficial peroneal nerve at Lateral
compartment of leg.

Foot Drop

Transverse Section Of Leg


1. Anterior compartment of leg
29
Anatomy

•• The - tibialis anterior Anat- Ortho Integration:


•• Hospitals - Hallucis longus Jones Fracture
•• Are - artery (Anterior tibial artery)
•• Never - nerve (Deep peroneal nerve)
•• Dirty - Digitorum longus
•• Places- Peroneus tertius

•• March Fracture:-

2. Lateral compartment of leg

3. Posterior compartment:-
•• Superficial group - have gastrocnemius, soleus
(helping in cardiac output so it is aka Peripheral
Q
heart), plantaris “(GSP) ”
•• Plantaris & palmaris longus both use in tendon
grafting operations.

•• In lateral compartment of leg :- Peroneus


longus & Peroneus brevis both innervated by
Q
SPN

•• Deep structures of this compartment - Tibialis


Posterior, flexor digitorum longus, Posterior
Tibial artery, Tibial nerve, flexor hallucis
longus.
30
Cerebellum Quick Revision Notes

FOOT ANATOMY
Q
–– mnemonic : Tom Dick And Nervous Harry
–– Tom - Tibialis posterior
Arches of Foot
–– Dick- Digitorum longus
–– And - Artery (Posterior tibial artery)
–– Nervous- nerve (Tibial nerve)
–– Harry - Hallucis longus
•• Achilles tendon - The strongest Tendon of
body

•• Largest bone of foot - calcaneus

–– Gastrocnemius (lateral & medial head) +


Soleus = Achilles tendon
–– Ankle jerk reflex has root value S1 & S2.

NOTE:-
•• Medial compartment: is absent but at upper
part 3 muscles insertion present :
•• Contains Sartorius, gracilis, semitendinosus
and Tibial collateral ligament

•• Radiology integration:

•• Inversion and eversion of foot occurs at the


subtalar joint.
•• Upper part of calcaneus which support the
talus known as sustentaculum tali.
31
Anatomy

•• Talo calcaneo-navicular is ball & socket joint.

Arches of foot -
•• It is due to special arrangements of foot bone
due to close interlocking short & small bones.
•• Arches are helpful for running , walking and
standing.
•• Deformities of foot:

Clubfoot aka CTEV


•• Most common congenital abnormality in the
world.
–– Talipes
–– Equino
–– Varus
•• Presentation – CAVE:-
–– Cavus
–– Adductus
•• Deltoid ligament’s1upper end is attached to
–– Varus Q
the medial malleolus .
–– Equinus •• This ligament has Superficial & deep fibers.
•• Calcaneo-navicular ligament is aka spring
ligament.
32
Cerebellum Quick Revision Notes

•• Clinical:-
–– Smoking leads to atherosclerosis, gangrene,
thromboangiitis obliterans or buerger’s
disease.
–– Palpable arteries of LL:-
ƒƒ Femoral Artery - at head of femur
ƒƒ Popliteal Artery - lower border of
Popleteus
ƒƒ ATA- between 2 malleoli
ƒƒ PTA- behind medial malleolus
ƒƒ DPA- palpable against navicular bone
–– Venous drainage of lower limb:-
ARTERIES AND VEINS OF •• Lower limb vein damage can lead to deep venous
LOWER LIMB ANATOMY thrombosis

Arteries of lower limb


Vascular sign of Narath
33
Anatomy

Clinical HEAD, NECK AND FACE


•• Phlebotomy :- cutting the vein
Scalp & Face
•• Hemochromatosis :- excessive iron
Q
•• Sural nerve having S1 nerve root
•• Medial part of leg and foot have L4 dermatome
•• GSV used for bypass surgery in 40 – 50% MI
blockage , now a days we use internal mammary
Q
artery (radial & ulnar vein also)
34
Cerebellum Quick Revision Notes

5 Layers of scalp Dissection of face


1. Skin 1. Skin
2. Connective tissue
2. Superficial Fascia
3. Aponeurosis
3. Deep Fascia -nt but only present in buccopharyngeal
4. Loose areolar connective tissue
fascia & parotido-masseteric fascia (In all other
5. Pericranium part of face, thorax and abdomen Deep Fascia is
absent)
Clinical 4. Subcutaneous Muscles (in animal it is known as
•• Loose areolar connective tissue layer is panniculus carnosus)
Dangerous area of scalp
•• Remnants of this panniculus carnosus are -
•• Surgical layers of scalp
face muscle, Palmaris Brevis, dartos muscle,
•• Black eye cutis ani.
•• Cephalhematoma

Q
•• Winking muscle of eye - orbicularis oculi
•• Whistle muscle - Buccal
•• Smiling muscle - Zygomatic major
•• Sad muscle - levator anguli superioris
•• Grinning muscle/ winner smile muscle - Risorius
•• Horror muscle - Platysma
•• Doubt muscle - Mentalis
•• Caput succedaneum (Risk factor - vacuum
delivery) •• Grief muscle - Depressor labii inferioris
•• Dimple location:- Modiolus
Muscles of facial expressions
Motor nerve branches of face : VII nerve
It gives 5 terminal branches within parotid gland
which supplies all facial muscles
•• Temporal, zygomatic, buccal, mandibular,
cervical.
•• Muscles derived by 2nd pharyngeal arch and
supplied by facial nerve except LPS (Levator
Q
Palpebrae Superioris : by 3rd cranial nerve)
35
Anatomy

ƒƒ Superior Thyroid artery

2 posterior branches
ƒƒ Occipital artery
ƒƒ Posterior auricular artery

2 terminal branches
ƒƒ Superficial Temporal artery (Clinical:-
Temporal arteritis) → give 1 branch:-
Transverse Facial artery
ƒƒ Maxillary artery

Branches of facial artery


•• Inferior labial artery
•• Bell’s palsy - Loss of Wrinkling, Wide palpebral
•• Superior labial artery
fissure, Whistling loss, loss of nasolabial fold
and drooling of saliva. •• Lateral Nasal artery
•• Angular artery → anastomoses with dorsal
Nasal artery which is the branch of Ophthalmic
artery

VESSELS & NERVES OF FACE •• Strongest muscle of the body :- Masseter


muscle
Face •• Facial artery palpable along lower margin of
mandible, in front of masseter attachment
•• Common Carotid artery branches;- divides into
ECA & ICA
Dangerous area of face
External Carotid artery (ECA)
1 medial branch:
•• Ascending pharyngeal artery (APA) → is the
only medial branch
•• It is smallest branch of ECA
It is also the 1st branch from ECA.

3 anterior branches:
ƒƒ Lingual artery
ƒƒ Facial artery
36
Cerebellum Quick Revision Notes

•• Lower part of nose & upper lip = known as Exception: Angle of mandible supplied by greater
Dangerous area of the face auricular nerve
•• Deep Facial vein uniting with the veins of •• Cutaneous lesions of herpes zoster
pterygoid plexus
ophthalmicus:- spreads along nerve roots
•• In brain we have 1 sinus which have multiple
caves = known as Cavernous sinus
•• Emissary veins = connects extra Cranial veins
& Intracranial veins
•• Way of spreading infection of Dangerous
area:-
–– Lower part of nose / upper lip → Facial vein
→ Deep facial vein → pterygoid plexus →
emissary veins → Cavernous sinus → death
Trigeminal neuralgia
Carbamazepine > valproate , Gabapentin > Surgery

NECK
Dissection / Surgery integration
•• Skin
•• Superficial Fascia (with platysma)
•• Deep Fascia (deep cervical fascia have 6
Maxillofacial death pyramid
modification) - aka fascia colli
1. Investing layer of Deep Fascia
2. Prevertebral fascia - Form Floor of
posterior triangle of neck, cover phrenic
nerve & scalenus anterior and making
Q
axillary sheath .
3. Pretracheal fascia :- form false capsule of
thyroid gland and suspensory ligament of

Q
berry .
4. Bucco- pharyngeal fascia
Nerves of face
5. Pharyngobasilar fascia
•• Motor :- 7th Cranial nerve except LPS ( by 3rd
Cranial nerve) •• Carotid sheath - Contain 9th, 10th, 11th and
Q

•• Sensory :- 5th nerve ( V1, V2, V3) 12th CN


–– Have 2 walls - Anterior (pretracheal
fascia), Posterior (prevertebral fascia &
have sympathetic trunk)
–– Ansa cervicalis - loop of the nerve in neck
Q
(present in anterior wall
37
Anatomy

•• Digastric muscle have dual nerve supply -


Anterior branch of digastric muscle supplied
by V3 & post branch supplied by 7th CN , so it’s
Q
the only hybrid muscle of the neck .
•• Sternocleidomastoids divides neck into 2
triangles -

A. Anterior triangle of neck (by 2 muscles -


Digastric & Superior belly of Omohyoid)

It is divided into 4 sub triangles

Muscles of neck
38
Cerebellum Quick Revision Notes

1. Submandibular or digastric triangle - Contain B. Supraclavicular/ subclavian triangle :


submandibular gland
2. Carotid triangle :- contain Carotid sheath,
3. Submental triangle :- contain submental LN
4. Muscular triangle:- contain different strap
muscles (sternothyroid, sternohyoid , omohyoid
, thyrohyoid) - these all have parallel muscle
fibers & all supplied by ansa cervicalis except
thyrohyoid which is supplied by C1 root.

B. Posterior triangle of neck :-


•• Anterior border - SCM
•• Posterior border - trapezius
•• Floor by middle 1/3rd of Clavicle
CRANIAL CAVITY, NERVE, VESSELS
•• By inferior belly of omohyoid it is divided into
2 sub triangles : CCranial cavity divided into 3 parts -
a. Occipital triangle 1. Anterior cranial fossa
b. Supraclavicular / subclavian triangle 2. Middle cranial fossa
3. Posterior cranial fossa

•• CSF rhinorrhea - occur via cribriform plate


after damage of the latter.
A. Occipital triangle : •• Sella turcica/ Turkish saddle
Q
is a part of
the body of a sphenoid.
Main contents
NOTE - Mnemonic for foramina
1. Main Nerve of auricular:- Greater auricular
nerve ( C2,C3) - it is related with Frey’s Important structures & foramen from midline to
Q
syndrome and have 2 divisions - lateral
•• Anterior division - Supply angle of mandible or •• King - Pituitary
shaving area ;
•• Queen (Shy) - Foramen Lacerum
•• Posterior division - Supply auricle
•• R - Foramen Rotundum - Maxillary Nerve
2. Nerve supply of Sternocleidomastoid &
Q •• O - Foramen Ovale (MALE)
trapezius - 11th CN (Spinal root)
•• Se - Foramen Spinosum (MEN)
39
Anatomy

Foramen Ovale (MALE)


Q
Foramen Spinosum (MEN)
Q damage to Nasal fibres of Optic Chiasma mainly
→ Bitemporal Hemianopia →Tubular Vision.
•• Mandibular •• Middle meningeal
•• Accessory meningeal artery, •• Middle Meningeal artery rupture can lead to
artery, •• Emissary vein (+/-) extradural hemorrhage - biconcave/idly/ lens
shape opacity.
•• Lesser Petrosal nerve, •• Nervi spinosum
•• Emissary vein

Foramen Jugulare Foramen magnum


•• Cranial nerves 9th, •• Largest foramen of
10th & 11th pass skull
through this. •• For Spinal cord and
other associated
structures.

•• Hardest bone of the skull - Petrous part of


temporal bone / Bony labyrinth
•• Optic canal : II CN & ophthalmic artery
•• From Superior Orbital Fissure - 3rd, 4th, 6th
and V1 cranial nerves passes.
•• Content of Dorello’s Canal - 6th Cranial nerve.
•• Internal Acoustic Meatus - 7th & 8th nerve
pass via this.

FOLDS OF DURA MATER &


SINUSES OF BRAIN

Duramater has 2 layers


a. Outer : Endosteal layer
b. Inner : Meningeal layer

4 folds of dura mater is located within brain:


a. Falx cerebri: in between 2 cerebral hemisphere.
b. Falx cerebelli : in between 2 cerebellar hemisphere.
c. Tentorium cerebelli : above cerebellar hemisphere.
d. Diaphragma sellae : above pituitary gland.

Clinical correlation:-
•• Pituitary Tumor → Damage to Optic Chiasma →
40
Cerebellum Quick Revision Notes

Dural venous sinuses


41
Anatomy

Cavernous sinus LARYNX, PHARYNX NOSE &


PALATE

Formula for HNF Cadaveric section


C T M V
Cavernous sinus thrombosis : S H P O
Ophthalmoplegia:
F P M T

Cricopharyngeal junction
•• located at level of C6
•• it is the narrowest part of GIT
•• Here the pharynx ends & esophagus starts.
•• Here the larynx ends & the trachea starts.

Pharynx
Upper part of Oesophagus
•• In case of increased intracranial pressure /
Boundaries:-
head injury :- Most common CN damage is 6th
Cranial nerve. •• Nasopharynx:- Anterior - Nasal cavity ,
Superior - Base of skull, Posterior - C1, Inferior
•• Central part of Pterion → k/a Sylvian point → - Oropharynx
Cranium is very thin here (CRANIOTOMY) →
deep to it Middle meningeal vessels lies → so •• Oropharynx:- Anterior - oral cavity, Superior
ruptures easily → results in EDH
Q
– Nasopharynx, Posterior - C2,C3,
Inferior – Laryngopharynx
•• Laryngopharynx:- Anterior - larynx, Superior
– Oropharynx,Posterior - C4-C6, Inferior –
esophagus
42
Cerebellum Quick Revision Notes

PARASYMPATHETIC GANGLION Basic concept of head & neck ganglion


Nucleus, ganglion & pseudoganglion •• 4 ganglion
Q
–– Ciliary ganglion - Eye - III CN
Q
–– Otic ganglion - Parotid gland - IX CN
–– Submandibular ganglion – Submandibular &
Sublingual gland - VII CN
–– Pterygopalatine ganglion – Lacrimal, Nasal
and Palatine gland - VII CN
•• Otic , Submandibular & pterygopalatine ganglion
associate with secretion under secretomotor
pathway.
•• This all ganglion having 4 (actually 3) roots:-
–– Sensory - related to the 5th nerve or
•• Collection of many cell bodies within the CNS
its branch. 5th nerve is the anatomical/
is known as nucleus & outside the CNS is known structural & topographical nerve of all
as ganglion. ganglions.
•• Fibers before the ganglion are known as –– Motor or Parasympathetic - Secretomotor
preganglionic fibers & after it are known as pathway of different glands.
post ganglion fibers. –– Sympathetic - from T1
•• Pseudoganglion is a collection of fat & connective 1. Ciliary ganglion:-
tissue, no nerve elements are present but it
•• It is a parasympathetic Ganglion which is
appears like ganglion. It is present in the nerve
related to 3rd CN
to teres minor which is a branch of axillary
nerve & in the radial nerve too .
Q
•• Size - pin head size
43
Anatomy

•• It is present between the 2nd CN & Lateral 2. Otic ganglion:-


Rectus muscle.

•• Every Cranial nerve has 2 relations :-


Anatomical/structural/ topographical (5th CN)
& physiological (3rd CN)

•• Parasympathetic action of ciliary ganglion:-

–– 3rd CN is running to the Lateral wall of


Cavernous sinus & in its apex, it is divided
into superior & inferior division .

–– It passes through superior orbital fissure,


here superior division gives branches to
Superior rectus muscle & Levator palpebrae
superioris.

–– Inferior division supplies Medial rectus,


Inferior rectus & Inferior oblique muscle
and ends into ciliary ganglion.

–– from ciliary ganglion Short ciliary nerves


arise, which supply to constrictor pupillae.
44
Cerebellum Quick Revision Notes

3. Submandibular ganglion:-

•• Secretomotor pathway of Submandibular & tympani → join to lingual nerve → Relay


sublingual gland:- Submandibular glanlion → Post ganglionic
fibres arises→join to Submandibular gland
–– Tasty food → activate Superior Salivatory
→ secretion
nucleus → impulse go to 7th nerve → chorda
45
Anatomy

•• Secretomotor pathway of lacrimal, Nasal Foetus - Spinal cord ending at the L3 vertebrae.
and Palatine ganglion. Filum terminale - Extension of pia mater from conus
–– Superior salivatory nucleus → 7th Cranial medullaris up to the coccyx. It has 2 parts namely
nerve → Greater Superficial Petrosal nerve F.T. Internal (15 cm) and F.T. External (5 cm).
→ In pterygoid canal/ vidian’s canal → it
joins with Deep Petrosal nerve (T1 ) to form Structures ending at S2 level
vidian’s nerve → vidian’s nerve in pterygoid 1. Arachnoid mater
canal join to V2 and relay into sphenopalatine 2. Dura mater
ganglion → further communicating maxillary
nerve, zygomatic nerve → impulse reaches 3. Subarachnoid space
to zygomatico-temporal nerve → join to
Lacrimal nerve & supply Lacrimal gland → At L3-L4 junction-
Lacrimation
Q
1. Lumbar puncture done

–– Cutting vidian’s nerve known as vidian’s 2. Iliac crest level


neurectomy 3. Location of Umbilicus

Spinal Cord Adult Child


Spinal cord L1 (Middle > lower L3-L4 junction
Nervous System ending border) > L1-L2
A. Central nervous system → Brain, Brain stem junction
& Spinal cord. Lumbar L3-L4 junction L4-L5 junction
B. Peripheral nervous system → Somatic & puncture
Autonomic nervous system. level

Autonomic nervous system Anat-Peds-Anesthesia Integration


1. Sympathetic - from thoraco-lumbar part.
Activate during fight, flight and fright. Lumbar puncture → done from Lumbar Cistern at
L3-L4 junction (Best).
2. Parasympathetic - from cranio-sacral spinal
cord. Structures pierced during L.P. :
3. Enteric nervous system → from neural crest Mnemonic :
cells → brain of Gut.
3 S ILE DAS
Brain stem → Midbrain, Pons & Medulla.
•• Skin
•• Superficial fascia
Spinal cord •• Supraspinous ligament
•• Downward cord like extension from medulla •• Interspinous ligament
oblongata. •• Ligamentum flavum

•• 2 enlargements - at Cervical (gives Brachial •• Epidural space


Plexus) and Lumbar (gives Lumbar Plexus). •• Dura mater
•• Arachnoid
•• Terminal enlargement known as Conus
•• Subarachnoid
medullaris.
space containing
•• 31 pairs (C8 T12 L5 S5 Co1) of Spinal nerves cerebrospinal fluid
which give a tail-like appearance after
termination of the spinal cord called the Cauda
equina.
Adult - Spinal cord ending at the middle of the L1
Q
vertebrae > L1 lower border> L1 L2 Junction
46
Cerebellum Quick Revision Notes

Transverse section of spinal cord Tracts in White matter:


•• Tract - bundle of axons which are having similar
Origin, Coarse, Termination & Function.
•• Collection of cell bodies
–– Inside CNS (Up to spinal cord) - Nucleus
–– Outside CNS (After spinal cord) - Ganglion

Mnemonic - SAME

•• Area around central canal – Nissl’s granules → •• SA - Sensory/Afferent/Ascending


Grey appearance •• ME - Motor/Efferent
•• Rest of the area – Myelin sheath present →
white/yellow appearance Anat-Physio Integration
•• 3 projections from grey matter → called as A. ASCENDING TRACTS
HORNS Fasciculus Gracilis and Cuneatus functions:
1. Dorsal horn - Sensory → Afferent tract 1. Fine touch
(Ascending tract)
2. Tactile localization
2. Ventral horn - Motor → Efferent tract 3. Tactile discrimination
(Descending tract)
4. Vibration → Tunic fork test
3. Lateral horn - Sympathetic in nature →
5. Stereognosis → Identify objects with closed
present in Thoraco-lumbar spinal (T1-L2)
eyes
cord only.
6. Proprioception → Sense of position
Anat-Physio Integration • Dorsal & Ventral Spinocerebellar Tract →
On basis of Histocytology, Raxed divide Gray Subconscious Kinesthetic sensations
matter into 10 Lamina: • Lateral spino-thalamic tract - Pain and
(MSC In RAG) Temperature

M - Marginal nucleus (Lateral most) • Anterior spinothalamic tract - Crude touch

S - Substantia Gelatinosa nucleus [in Lamina 2] B. DESCENDING TRACT


–– Function - Pain & Temperature → Inhibited Pyramidal tracts (Cortico-spinal tract) →
by Morphine Decussate and form
C - Clark nucleus & Chief sensory nucleus 1. Lateral cortico-spinal tract
In - Interneuron 2. Medial cortico-spinal tract
R - Ranchow cells
Extra Pyramidal tracts:(ROVT)
A - Alpha motor neuron
1. Rubro-spinal tract
G - Gamma motor neuron
2. Olivaro-spinal tract

•• White matter also having 3 parts 3. Vestibulospinal tract

A. Posterior white column 4. Tectospinal tract

B. Lateral white column


C. Anterior white column
47
Anatomy

Anat-Physio-Medicine Integration Descending Tracts:


Brown Sequard Syndrome: Hemi-sectioning of Pyramidal tract
Q
spinal cord
•• 80% decussate at Medulla oblongata, called
•• I/L loss of Fine touch the Lateral Corticospinal tract.
•• C/L loss of Pain & Temperature •• 20% - Don't cross and pass anteriorly, called
•• I/L loss of Position sense the Anterior Corticospinal tract.

•• I/L motor paralysis •• Start from Area number 4 (Motor area) called
Q Pyramidal cells of Betz.
Syringomyelia: Dilatation of Central canal
•• Form Corona radiata after leaving Cortex of
•• Abnormal dilatation → Lateral spinothalamic
cerebrum.
tract affected - Loss of Pain and Temperature.
•• Anterior Spinothalamic tract is intact. Other
sensations are normal.
•• Cause - Syphilis (Treponema pallidum).
•• Dissociative anaesthesia seen → also in
Ketamine.
48
Cerebellum Quick Revision Notes

•• Passing through the Posterior limb of the


Internal Capsule.

Anat-Physio-Medicine Integration
Tabes Dorsalis morphology:
(Mnemonic DORSALIS)
•• Fasciculus gracilis and
•• Fasciculus cuneatus damage.
•• Dorsal column degeneration
•• Orthopaedic pain (Charcot joints)
•• Reflexes decreased (deep tendon)
•• Shooting pain
•• Argyll-Robertson pupils
•• Locomotor ataxia
•• Impaired proprioception
•• Syphilis

•• Medulla is a truncated bulb-like structure.


(Medulla = Bulb)
•• Hence medullary paralysis means Bulbar
paralysis .
•• At Medulla , 9th, 10th, 11th and 12th Cranial
nerve attached → so Medullary paralysis =
Bulbar paralysis = IX , X, XI, XII paralysis
•• At pontomedullary junction - 6th , 7th, 8th
BRAIN STEM cranial nerve .
•• 7th cranial nerve has 2 divisions:
Brain stem is divided into 3 parts:
a. Medial division : motor
1. Medulla
b. Sensory division : lateral
2. Pons
•• Sensory division of 7th CN is known as the
3. Midbrain nerve of Wrisberg or Nervous intermedius.
•• 8th cranial nerve also has 2 divisions:
a. Cochlear division
b. Vestibular division
•• At the ventral surface of Pons, 5th cranial
nerve present & it is divided into 3 divisions
(V1, V2, V3),
49
Anatomy

•• V CN ganglion is covered with a fold of dura located. So lesions in this area lead to death.
mater known as meckle’s cave.
•• In hanging, fracture of the odontoid process
•• Above Pons there is crus cerebri → medial to leads to compression of these vital centres
it, 3rd cranial nerve attached and 4th cranial which results in death. (ANAT- FMT
nerve attached posteriorly. Integration)
•• 1st & 2nd CN are attached on the inferior •• In the floor of IV ventricle, Area Postrema is
surface of the frontal lobe. located. This area lacks a Blood Brain Barrier.
Just below it , there is a chemoreceptor
Clinical Integration trigger zone which functions as a vomiting
Q
center. (ANAT- PHARMA Integration)
Clinical Integration :
•• 4 important facts about 4th cranial nerve:
•• In Medial medullary syndrome / Dejerine
Q
“DDLT” (Dilwale Dulhania Le Thahrenge)
syndrome – midline part of medulla is damaged
–– D:- Dorsal attachment
→ hence 12th CN damage à ipsilateral tongue
Q
deviation is seen . –– D:- Decussates

•• In Lateral medullary syndrome/ PICA –– L:- Longest intracranial nerve


syndrome / Wallenberg syndrome / Vertebral –– T:- Thinnest cranial nerve, because it has
artery syndrome → Lateral part of medulla the least axons → hence it is considered as
is damagedà laterally 10th nerve is located the smallest cranial nerve.
hence 10th cranial nerve damage → palatal &
Q
pharyngeal reflexes lost . (Anat-Pharma-Medicine Integration)

Pineal gland/ 3rd eye of lord Shiva


•• Here tryptophan converts → to 5- HT /
serotonin → N-acetyl serotonin → Melatonin.
•• Facial colliculus lesion : 7th cranial nerve
•• Melatonin responsible for biological clock
fibres > 6th cranial nerve nucleus damage. activity.
•• Damage at floor of 4th Ventricle leads to •• So, it can be used for treatment of disturbed
damage of 6th, 7th, 8th, 10th & 12th cranial biological clock activity like sleep. Eg.
nerves. a. Day night shift workers
•• At the floor of IV ventricle vital centres like b. treatment of jet lag effect
Cardiac centre & Respiratory centre are
c. insomnia
50
Cerebellum Quick Revision Notes

•• Substantia nigra :- Here dopamine synthesis –– Decrease in dopamine → leads to increase


occurs. relative level of Ach → leads to hypertonia,
rigidity, resting tremor, hypokinesia,
–– Phenylalanine → Tyrosine → Dopa shuffling gait, pill rolling movement, mask
→ Dopamine. (ANAT-BIOCHEM like face etc. This clinical situation is known
INTEGRATION) as Parkinsonism / Paralytic agitans.

–– Dopamine is an inhibitory neurotransmitter


and responsible for muscle tone.

Cranial nerve Nuclei Attachment Foramen Damage can lead to


I – olfactory No nucleus Inferior surface Cribriform Plate Anosmia
of frontal lobe of ethmoid
II - optic No Nucleus Inferior surface Optic canal Anopia
of frontal Lobe
III - oculomotor Midbrain - Medial to Crus Superior Orbital Squint
Superior Colliculus Cerebri Fissure
IV - trochlear Midbrain - Lateral to Crus Superior Orbital Squint
Inferior Colliculus Cerebri Fissure
V- trigeminal Pons Ventral surface V1- Superior Trigeminal neuralgia
of pons Orbital Fissure
jaw deviation
V2- F. Rotundum
V3- F. Ovale
VI - Abducens Pons Ponto-medullary Superior Orbital Med squint
Junction Fissure
VII - Facial Pons Ponto-medullary Internal Acoustic Bell’s palsy
Junction Foramen
VIII - Cochleo- Pons Ponto-medullary Internal Acoustic Loss of hearing
vestibular Junction Foramen
IX - Medulla Behind olive Jugular Foramen Loss of taste (Posterior
Glossopharyngeal 1/3rd of tongue)
X - Vagus Medulla Behind olive Jugular Foramen Loss of taste (Posterior
most part of tongue)
XI - Accessory Medulla Behind olive Jugular Foramen Paralysis of trapezius &
Spinal sternocleidomastoid
XII - Hypoglossal Medulla Behind pyramid Hypoglossal canal I/L deviation of tongue

Note : CN formula : 2,2,4,4


51
Anatomy

Brain stem syndrome •• 3 - 3 Anatomical lobe, 3 physiological lobe, 3


phylogenetic lobe, 3 fissures , 3 cerebellar
•• Medullary syndrome peduncles and 3 histological layers.
–– Lateral Medullary syndrome - 9, 10 & 11 CN •• 4 - 4 nuclei
damage
•• 5 - 5 neurons
–– Medial Medullary syndrome - 12th CN
damage :- I/L deviation of tongue
•• Pontine syndrome:-
–– Facial colliculus syndrome:- damage to 6th,
7th CN
–– Millard gubler syndrome:- damage to 6th,
7th, 8th CN
•• Midbrain syndrome:-
–– Posterior part damage :- parinaud's
syndrome
•• 3 Anatomical lobes - Anterior, Middle/
–– Middle part damage :- Benedikt syndrome
Posterior, flocculonodular lobe.
–– W part damage :- weber syndrome
•• 3 physiological lobes -
•• Revision:-
–– Optic chiasma damage :- bitemporal –– Median zone :- which control axial movement
hemianopia
–– Intermediate zone :- hands & foot
–– Cranial nerve supplying EOMs:- SO4 > LR6
–– Lateral zone :- Pyramidal tract & brain
–– Nerve supply to tonsil :- 9th Cranial Nerve
•• 3 phylogenetic lobes -
–– Tensor palatini is supplied by :- 5th Cranial
Nerve (3rd Part - Mandibular Nerve) –– Archicerebellum - oldest , having connection
–– Stylopharyngeus supplied by :- 9th Cranial with vestibular apparatus – so help in
Nerve
equilibrium.
–– Cricothyroid supplied by :- External
Laryngeal Nerve –– Paleocerebellum - have connection with
spinal cord – so responsible for posture
–– Palatoglossus supplied by :- 5th Cranial
Nerve < 10th Cranial Nerve ,tone and crude movements.

–– Neocerebellum - connected with the cerebral


CEREBELLUM cortex – so related to fine Movements.
Cerebellum Considered as
a. Little brain
b. Arbor vitae (life of tree)
c. Fern tree like appearance of section & histology
d. Coordination & balancing organ

Magic of 1,2,3,4,5
•• 1 - Vermin
•• 3 histological layers - Little brain “MPG”
•• 2 - Cerebellar hemisphere
52
Cerebellum Quick Revision Notes

–– M - Molecular layer •• Blood supply -


–– P - Purkinje layer a. PICA: Posterior Inferior Cerebellar Artery

–– G - Granular cell layer b. AICA: Anterior Inferior Cerebellar Artery

•• 4 Nuclei (DEGF - in large to small & Lateral to c. SCA: Superior Cerebellar Artery
medial sequence)
–– D:- Dentate nuclei
–– E:- Embolism nuclei
–– G:- Globose nuclei
–– F :- Fastigial nuclei

•• Functions of cerebellum -
–– Having proper 3 dimensional balance.
–– Maintain rate and range of direction.
–– Holding things by proper force.
•• Defect lead to - “NIDRA”
–– Loss of tone, posture and equilibrium.
–– Ataxia
–– N - Nystagmus
–– I - Intentional tremor
–– D - Dysdiadochokinesia
–– R - Rhomberg sign
–– A - Ataxia, Asynergia
•• 5 neurons of cerebellum:- (BSP GoGa)
–– B - Basket cells
–– S - Stellate cells
–– P - Purkinje cells :- largest, only efferent
fibers present, inhibitory to deep cerebellar
nuclei.
–– Go - Golgi cells
–– Ga (sir):- Granular cells
•• Basket cells & Stellate cells located in the
molecular layer.
•• Purkinje cells in the purkinje layer.
•• Golgi & Granular cells – located in the granular
layer.
53
Anatomy

CEREBRAL HEMISPHERE Superolateral area of brain


1. Frontal lobe
•• At frontal lobe - Motor area (area 4) present
from which the pyramidal tract arises.
Damage of this area can lead to contra-lateral
Q
paralysis .

•• Premotor area (area 6) gives origin to the


extra pyramidal tract and its damage can lead
to Parkinson-like symptoms and tremors.

•• Middle frontal gyrus - responsible for 2. Temporal lobe -


horizontal eye movements and damage leads
•• Superior Temporal gyrus have area 41, 42 -
to loss of conjugated saccadic movements. Q
are 1° auditory / Auditory Sensory area .
•• Pre-frontal lobe - responsible for individual –– Area 22 - 2° auditory or auditory psychic
skills, judgements & personality and damage area and due to its damage, a person will be
leads to pre-frontal lobe syndrome. able to hear but not able to analyze what it
•• Inferior frontal gyrus - divides into pars means (auditory agnosia).
orbitalis , pars triangulation & pars opercularis –– Posterior part of Area 22 known as
(broca’s area 44,45), responsible for speech , Wernicke's area or Sensory speech area
Q
damage lead to motor/ broca’s aphasia .
Q
and its damage leads to sensory aphasia .
54
Cerebellum Quick Revision Notes

–– Medial part of this lobe is known as Area 28 Revision


(olfactory area).
•• 1, 2, 3 : Sensory
3. Parietal lobe •• 4 : Motor
•• Impulse from Area 22 passes to area 39 & •• 5, 7 : Vibration & stereognosis
40, known as speech association area, then
impulse goes to area 44 and 45. •• 6 : premotor area

•• Area 5,7 - responsible for vibration & •• 8 : frontal eye field area
stereognosis, so, damage leads to pallesthesia, •• 9, 10, 11, 12 : Prefrontal lobe
& Astereognosis.
•• 17 : visual sensory area
•• Area 2,1,3 - is Sensory area, damage leads to
•• 18, 19 : visual psychic area
loss of sensation.
•• 22 : Sensory speech area , auditory sensory
•• Area 43 - for taste.
area
4. Occipital lobe •• 28 : olfactory area
•• Calcarine sulcus present which is an example •• 39 : speech association area
of Complete sulcus.
•• 43 : taste sensation area
•• Semilunar sulcus, around the calcarine sulcus,
•• Motor speech - 44, 45
is known as lunate sulcus.
•• paracentral lobule : Perineum area
•• Area 17 - is 1° visual area, damage leads to
blindness. •• Loss of Vibration - pallesthesia
•• Area 18,19 - known as 2° visual area, damage
leads to visual agnosia. BLOOD SUPPLY OF BRAIN
Medial surface of brain Sensory & motor homunculus
–– Para central lobule - where perineum area
representsq
ƒƒ Bladder
ƒƒ Bowel
ƒƒ Genital area

Inverted homunculus representation

–– Have visual-striated areas or lines of


Genneri.
55
Anatomy

Largest presentation - Face and Lips 4. Posterior communicating artery


5. Posterior cerebral artery
Artery Area of brain Damage lead to

1. MCA (Middle
Supplies Damage leads to Clinical
superolateral aphasia paralysis
cerebral artery) •• Aneurysm of COW is known as Berry aneurysm.
surface of upper limbs
Damage leads •• Berry aneurysm rupture lead to subarachnoid
to paraplegia, hemorrhage → Blood in CSF (complain of patient
2. ACA (Anterior Supplies medial patient do - worst headache of my life/ thunderbolt
cerebral artery) surface urination and headache/ thunder clapping headache)
defecation in
bed. Blood supply to brain
3. PCA (Posterior supplies inferior Damage lead to 5 branches of Internal Carotid artery
cerebral artery) surface visual problem
•• Anterior Cerebral artery
•• Ophthalmic artery
•• Middle Cerebral artery
•• Anterior choroidal artery
•• Posterior Cerebral artery

5 Branches of vertebral artery -


•• Anterior spinal artery
•• Posterior spinal artery
•• Meningeal artery
•• Medullary artery
•• Posterior inferior cerebellar artery - damage
lead to Wallenberg syndrome or Lateral
medullary syndrome

Branches of basilar artery -


•• Anterior inferior cerebellar artery
5 components of circle of willis (COW) •• Pontine - damage lead to pontine hemorrhage,
1. Anterior cerebral artery pinpoint pupil , Pyrexia
2. Anterior communicating artery •• Labyrinthine artery
3. Internal Carotid artery •• Superior cerebellar artery
•• Posterior Cerebellar artery
56
Cerebellum Quick Revision Notes

WHITE MATTER AND BASAL


NUCLEI
White matterQ
3 types - (ACP/ CAP)
•• Association fiber :- They connect adjacent
different areas in the same hemisphere.

•• Commissural fiber :- connect the same


area in the different hemisphere known as
FornixQ
Commissural fiber. Ex - corpus callosum.
•• Have all 3 types of white matter fibres.
•• Projection fiber :- connect areas from inside
•• Hippocampus - convert short term memory to
to outside of the brain . Ex - all tracts
long term memory.

•• Impulses go from hippocampus to mammillary


bodies via fornix.

•• So hippocampus is afferent for fornix &


mammillary bodies are efferent for fornix.

Papez circuit
•• Responsible for memory.

•• Corpus callosum:- •• Impulses go from the anterior nucleus of


thalamus → to cingulate gyrus (center for
–– Largest band of white matter satisfaction ) → hippocampus → mammillary
–– Type of commissural fibre. body → finally to the anterior nucleus of
thalamus again.
–– Have 4 parts - Rostrum, Genu, Body/Trunk,
Selenium.
Projection fibers
Include all tracts
57
Anatomy

Ascending:- •• Wilson’s disease aka hepato-lenticular


•• LSTT (Lateral Spino-thalamic Tract) degeneration.
•• FG (Fasciculus Gracilis)
THORACIC WALL INTERCOSTAL
•• FC (Fasciculus Cuneatus)
SPACE
Descending:-
•• Internal capsule (bundle of tracts)
•• corona radiata

Basal nuclei
Means collection of Grey matter (nuclei) at the base
of the brain.

•• T2-T8 are typical vertebrate, rest all are


atypical.
•• Basic components of Basal nuclei :- ACC SS
–– A - Amygdala is small almond like structure Contents of Intercostal space (ICS)
of brain related with food & sex activity, •• Intercostal muscles:-
so, damage to this lead to hyperphagia and
–– External Intercostal muscles - outermost;
hyper sexuality known as kluver-bucy
hands in pocket direction,runs downward,
syndrome
forward, medially. (DFM)
–– C - Corpus Striatum has 2 nucleus
–– Internal Intercostal muscles- hands in opp
ƒƒ Caudate nucleus pocket; move downward, backward, lateral.
(DBL)
ƒƒ Lentiform nucleus - has medial globus
palladium & lateral putamen –– Transversus thoracis - DIVIDED INTO 3
GROUPS:
–– C : Claustrum
a. Sterno + costal,
–– S:- Substantia nigra → damage results in
b. Innermost intercostals/ intercostal intimi,
Parkinson disease
c. Subcostalis.
–– S:- Subthalamic nucleus of luy → damage
leads to hemiballismus. •• Vessels :- VAN (Vein, Artery, Nerve) in costal
groove.
•• Best site for Thoracentesis is 8th Intercostal
58
Cerebellum Quick Revision Notes

Space > 9th at mid-axillary line, at the lower PLEURA & LUNG
part of Intercostal space and along the upper
Q
border of the rib .

Subclavian artery branches


(VIT – CD)

•• V:- Vertebral artery - Circle of Willis

•• I:- Int mammary artery - CABG

•• T:- Thyrocervical trunk - Supra scapular


artery, Inferior thyroid artery, transverse
Bronchopulmonary segments (BPS)
cervical artery (SIT).
•• Right lung -
•• C:- Costocervical trunk :- Superior intercostal –– Has 3 lobes
artery, Deep cervical artery.
ƒƒUpper lobe - has Apical, Anterior,
•• D:- Dorsal scapular artery. Posterior segments
ƒƒMiddle lobe - has Medial & Lateral
segments
ƒƒLower lobe - has Apical Basal , Anterior
Basal, Lateral Basal, Posterior Basal,
Medial Basal segments.
59
Anatomy

•• Left lung :- –– Right posterior aortic sinus


–– Upper lobe :-has Apical, Anterior, Posterior,
Superior & Inferior lingual
–– Lower lobe :- has Apical Basal, Anterior
Basal, Lateral Basal, Posterior Basal, Medial
basal (+/-)

HEART & CORONARY


CIRCULATION
•• Heart is conical muscular hollow viscera which
is responsible for pumping the blood. RCA: Course & Branches
•• it is located in the middle mediastinum within Main Branches: MTP
the pericardium.

External Features of Heart :

•• Right border of the heart - RA


•• Apex of the Heart – LV
•• Posterior surface of the heart (Base) – LA
(2/3rd) + RA (1/3rd)
LCA: Course & Branches
Radiology-Anatomy Integration
Right heart border – RA + SVC + IVC

Arterial Supply of the Heart


•• The root of the ascending aorta has semilunar
valve which is having 3 leaflets
–– Anterior leaflet
–– Left Posterior leaflet
–– Right posterior leaflet
•• Cavity b/w the aortic wall and leaflet margins
is the Aortic Sinus
–– Anterior Aortic Sinus – gives rise to Right
Coronary Artery → Rt. Conus artery
–– Left posterior aortic sinus – gives rise to
Left Coronary Artery
60
Cerebellum Quick Revision Notes

Main Branches “DAL”Q •• Largest vein – Coronary sinus (situated in the


Q
•• Diagonal artery left posterior Interventricular sulcus)

•• Anterior Interventricular artery (AIVA) – •• All the cardiac veins drain into the coronary
supplies the Apex sinus except 2 –
•• Left Circumflex artery –– Anterior cardiac veins & Venae Cordis
minimae – drain into Rt. Atria
Patho/Medicine-Anatomy Integration
•• Great cardiac vein runs with the AIVA & Lt.
AIVA is aka LADA (MC in →)
CXA
•• Left Anterior descending artery – Mc artery
involved in MI/Angina (40-50%) •• Middle cardiac vein running with the PIVA

•• Most common cause of Angina/ MI is •• Right Marginal vein making the small cardiac
Atherosclerosis vein which is draining into the Coronary sinus
Widow’s artery/ Widow maker artery – LADA
Angioplasty
Small Branches
CABG
•• Lt. Conus artery
•• Great saphenous vein was used earlier for the
•• Atrial branches
coronary graft
•• Ventricular branches
•• When there is damage to the GSV then the
Conducting system of the Heart nerve getting damaged in the lower limb is the
•• SA NODE → AV NODE → BUNDLE OF HIS → Saphenous nerve – loss of sensation along
RBB & LBB the medial part of the leg
•• All are supplied by the RCA except for the LBB •• Left Internal Mammary Artery (LIMA) is the
which is supplied by the LCA best arterial graft for CABG

Anterior Abdominal Wall


•• Abdominal cavity : 2 walls
a. anterior abdominal wall (AAW)
b. posterior abdominal wall (PAW)
•• Anterior Abdomen wall :

Venous Drainage of the Heart

–– AAW is divided into 9 quadrants by 2


vertical lines & 2 horizontal lines.
–– Transpyloric plane passing through the
Q
lower border of L1 .
61
Anatomy

–– Subcostal plane passing through upper


border of L3 level.
–– Transtubercular plane passing through
upper border of L5.
–– Umbilicus is situated at L3,L4 ( it’s a site for
Iliac crest & Lumbar puncture).
–– External Oblique modification (LIP):-
ƒƒ L:- participate in the formation of Lacunar
ligament.
ƒƒ I:- Inguinal ligament
ƒƒ P:- Pectineal ligament
•• 8 Layers of abdominal wall:

PERITONEUM & ABDOMINAL


LIGAMENTS

Skin
Superficial fascia
External oblique
Internal oblique
Transverse abdominis
Transversalis Fascia
Extra peritoneal fat
Peritoneum
62
Cerebellum Quick Revision Notes

–– Peritoneum fold :- responsible for


Movements & peristalsis.

–– If Peritoneum fold is attached with

ƒƒ Stomach → it is known as omentum


(greater & lesser).

ƒƒ Intestines → it is known as mesentery,


with duodenum known as mesoduodenum.

ƒƒ Transverse colon → known as Transverse


mesocolon.

ƒƒ Appendix → known as mesoappendix.

ƒƒ Viscera → gastrosplenic ligament,


lienorenal ligament.

Abdominal Viscera have 3 types


1. Intraperitoneal :- free Movements.

2. Retroperitoneal :- behind, it is fixed, ex :-


pancreas, kidney & related structures, duodenum.

3. Subperitoneal :- below peritoneum eg. pelvic


organs.

–– Foramen of Winslow:- It is communication


between greater & lesser sac aka epiploic
foramen.
63
Anatomy

Mesogastrium has 2 parts : b. Dorsal mesogastrium (DMG): It divides


into 2 parts →
a. Ventral mesogastrium (VMG): It divides
into 2 parts → –– Ventral part - give rise to gastro-splenic
ligament→ short gastric vessels run within
–– Ventral part - give rise to falciform ligament
it
–– Dorsal part - give rise to lesser omentum
–– Dorsal part - give rise to lino-renal
ligament→ splenic vessels run within it
•• Pringle’s maneuver:-
64
Cerebellum Quick Revision Notes

Q
Epiploic Foramen (Boundaries) -
Anterior - Right free margin of lesser omentum
Posterior-
•• IVC
•• Right Suprarenal
•• T 12
Superior-Caudate process of liver

For Portal hypertension

Mickey Mouse signs


ESOPHAGUS & STOMACH
•• Portal triad in Portal Hypertension.
Esophagus
•• Midbrain of progressive supranuclear palsy. 4 constrictions of the Esophagus – measured from
the upper incisors
•• Polyostotic Paget's disease

•• Pelvic Mickey Mouse sign: bilateral inguinal


vesical hernia

•• Dysmorphic Mickey Mouse RBCs & Ureteropelvic


junction obstruction (UPJO)
65
Anatomy

Constrictions due to Vertebra level Measurement


1. Pharyngoesophageal junction/ Cricopharyngeal
C6 vertebra 6” x 2.5 cm = 15cms
junction – narrowest part of GIT
2. AOA T4 vertebra. 9” = 22.5cms
3. left bronchus, T6 vertebra 11” = 27.5cms
4. Diaphragm T10 vertebra 15” = 37.5cms

Tricks & Magics


Structures having length – 25cms = DUDES

•• D – Duodenum

•• U – Ureter

•• D – Descending colon

•• E – Esophagus

•• S – Stomach

Surgery-Anatomy Integration
Parts of Esophagus Artery Vein Lymph node
Inferior thyroid vein –
Upper 1/3rd part Inferior thyroid artery Deep cervical LN
Brachiocephalic vein (BCV)
Esophageal br. from Descending
Middle 1/3rd part Azygous vein Mediastinal LN
thoracic aorta
Left gastric artery (esophageal
Lower 1/3rd part Left gastric vein “BAL” – mnemonic Left gastric LN
branches) “OIL” – mnemonic

Stomach ANAT-PATHO-MEDICINE-SURGERY
Parts of Stomach INTEGRATION
1. Cardiac part Pyloric antrum is the 2nd most common site for the
a. Fundus Duodenal Ulcer Disease (DUD)
b. Body Incisura Angularis is most common site for Gastric
2. Pyloric part ulcer Disease (GUD)
a. Pyloric Antrum
b. Pyloric Canal
66
Cerebellum Quick Revision Notes

MAGIC OF 2 ƒƒ Left gastroepiploic artery


Q
•• 2 ends –– Common hepatic artery
•• 2 curvatures ƒƒProper hepatic artery
•• 2 surfaces ○○ Left hepatic branch
•• 2 angles ○○ Right hepatic branch
•• 2 parts – further divided into 2 Cystic artery (to gallbladder)

Anat-Patho-Medicine-Surgery ○○ Right gastric artery (anastomose


Integration with the left)
ƒƒ Right gastroduodenal artery
○○ Right gastroepiploic artery
(anastomose with the left)
○○ Superior pancreaticoduodenal artery
Anastomose with the inferior
pancreaticoduodenal artery – br. of
the IMA
•• Anastomosis between superior and inferior
pancreaticoduodenal artery
–– Junction between foregut and midgut
Arterial Supply of Stomach –– Opening of CBD

GIT – Anat-Surgery Integration


Q
•• FOREGUT – Celiac Trunk – T12-L1 Esophageal Replacement Surgery
•• MIDGUT – SMA – L1
•• In case of esophageal replacement surgery,
•• HINDGUT – IMA – L3 the best site for the graft is the part
of the stomach where the left and right
gastroepiploic artery anastomosis or also the
lesser curvature part where the right and
left gastric arteries are anastomosing
•• To prevent the bleeding from the
Duodenal ulcer, the artery ligated will be
Gastroduodenal artery

•• Give 3 main branches


–– Left gastric artery – supply lower end of
the esophagus
–– Splenic artery –
ƒƒ Arteria pancreatica magna – supply body
of pancreas
ƒƒ Short gastric vessels
67
Anatomy

Lymphatic Drainage of Stomach

•• ● All the LNs drain into the Celiac nodes

Anat-Patho Integration
In case of Stomach Ca – enlargement of left
supraclavicular LN =
•• Virchow’s LN
•• Troisier’s sign
Duodenum
In case of Stomach Ca – enlargement of left
•• Shorterst , widest & most fixed part of small
axillary LN = Irish LN
intestine
In case of Stomach Ca – enlargement of •• 4 parts : D1, D2, D3, D3
periumbilical LN = Sister Mery Joseph Nodules
•• Extension : between L1 to L3 Vertebra, above
In case of Stomach Ca – there is trans celomic umbilicus
spread – Ovarian Ca = Krukenberg’s tumor
•• Total length : 25 cm
–– D1 → 2 inches → located at L1
SMALL & LARGE INTESTINES –– D2 → 3 inches → L2
–– D3 → 4 inches → L3
–– D4 → 1 inch
–– Total 10 inches × 2.5 = 25 cm
•• IMPORTANT Relations
–– 1st part of Duodenum: anterior to head of
pancreas
–– 2nd part of Duodenum: anteriorly related
to Rt. Kidney
–– Gallbladder fundus is in front of 2nd part of
duodenum
–– SMA & vein passing anterior to 3rd part of
Duodenum
68
Cerebellum Quick Revision Notes

•• At L3 , L4 level → Umbilicus, Iliac crest


located, & it is the site for Lumbar puncture
•• Distance between Minor duodenal papilla
& major duodenal papilla / opening of main
pancreatic duct is = 2 cm
•• Junction between duodenum & jejunum →
duodenojejunal flexure
•• At duodenojejunal flexure, the suspensory
ligament of Treitz is attached.
•• Celiac trunk pierces the suspensory ligament
of Treitz
•• Suspensory ligament of Treitz is used for
Upper & lower GI bleeding landmarks. Bleeding
above suspensory ligament of Treitz known as
upper GI bleeding & below to it known as lower
GI bleeding.

Large Intestine
Parts of large intestine
1. Ascending colon
2. Transverse colon
3. Descending colon
4. Sigmoid colon
5. Caecum & Appendix
6. Rectum
7. Anal canal
Q
•• Main 3 Features of Large Intestine
1. H :- Haustra / Saccule
2. A :- Appendices epiploicae = small pouch of
fat
3. T :- Taenia coli (3 in no.)
69
Anatomy

ARTERIES & VEINS OF GIT

SMA – Artery of The Midgut –


Branches

•• Abdominal Aorta pierces the diaphragm and


runs in downward direction.
•• At the lower border of L4 or L4-L5 junction, it
is bifurcating into common iliac arteries

•• Abdominal Aorta aneurysm are most common in


SMA – Arise at the level of L1
Infrarenal position and also Aortic Dissections. 6 branches of SMA :
•• Inferior pancreatico duodenal artery – to
duodenum and pancreas
•• Jejunal branches
•• Ileal branches
•• Ileocolic Artery – to ileum as well as cecum
•• Right colic artery
•• Middle colic artery
•• Marginal arteries of Drummond – small arteries
coming out and supplying the large intestine

IMA – artery of Hindgut – at the


level of L3
3 branches of IMA :
•• Left colic artery
70
Cerebellum Quick Revision Notes

•• Sigmoidal Artery ABDOMINAL VISCERA ORGANS


•• Superior rectal artery

Venous Drainage of the Abdomen


•• Portal vein = Splenic vein + SMV
•• Behind the neck of pancreas
•• At the level of L2

Parts of pancreas: Head, neck, body, tail


Q

Important relations:
1. Head : located within the C loop of duodenum .
2. Neck : behind it formation of portal vein .
3. Body : along upper border splenic artery runs
Micro-Parasito/Surgery/Medicine- 4. Tail : located within hilum of spleen
Anatomy Integration
Parts of duodenum & their corresponding
Entamoeba histolytica infection and also vertebral level:
Hepatocellular cancer case:
•• D1 : L1
•• Due to shorter right sided division of the
portal vein, all the infection is going to the •• D2; L2
right lobe of the liver •• D3: L3
•• Rt lobe – more common for ↑ HCC & ↑ infection •• Epiploic foramen: T12
•• This is known as Streamline phenomenon •• Portal vein formation: behind neck of pancreas
at L2
•• Portal vein = Splenic vein + Superior mesenteric
vein
•• In an uncinate process tumor → compression
of Superior mesenteric vessels

Spleen
•• Located in the left hypochondrium region.
•• Costal Surface → 9th, 10th (45°) and 11th ribs.
•• Visceral surface → Gastric, Colic, Renal and
Pancreas impressions.
71
Anatomy

LIVER

•• Weight: 1.5 kg

Magic of 5 :
•• 5 ligaments:
•• Harris dictum : Magic of Odd numbers. 1, 3, 7,
9, 11 –– Falciform: ventral part of Ventral
mesogastrium.
–– 1” 3” 5” : Dimensions
–– Ligamentum Venosum
–– 7 : ounce is weight
–– Ligamentum Teres
–– 9, 10, 11 : Ribs are costal relations.
–– Coronary ligament (superior & inferior
•• Axis along 10th rib layers)
•• Two surfaces –– Triangular ligament (right & left)
: Diaphragmatic ( outer ) ( Rib & intercostal 9,
•• Porta hepatis: 5 cm
10, 11 )
•• 5 structures within Porta hepatis:
: Visceral (inner)
–– Portal Vein
Visceral surface → gastric, colic, renal,
pancreas –– Hepatic artery

impressions “GCRP” –– Bile duct

(In uncinate process tumor: compression of –– Hepatic plexus

SMV + SMA) –– Hepatic Lymph nodes

•• Artery: splenic artery •• 5 viscera at inferior surface:


•• Vein: splenic Vein –– For stomach
•• Clinical correlation: –– For Gallbladder
–– splenomegaly –– For duodenum
ƒƒ seen in Hemolytic anemias, Sickle cell –– For right kidney
anemia and Thalassemia.
–– For Hepatic flexor
–– Hypersplenism
•• Clinical correlation:
–– Hepatitis: Acute & chronic
–– Tumor: Benign (Hepatic adenocarcinoma) &
Malignant (Hepatocellular cancer)
–– Hepatectomy (right & left lobectomy)
–– Jaundice (pre-hepatic, intra hepatic, post
hepatic)
–– Hyperbilirubinemia: Conjugated &
Unconjugated
72
Cerebellum Quick Revision Notes

––Liver failure: liver transplantation


–– Cirrhosis

Extrahepatic Biliary Apparatus

KIDNEY

•• Kidney is the main excretory organ in humans.


•• it has 2 poles , 2 borders & 2 surfaces
Gallbladder •• Structure at renal hilum: Anterior to posterior
•• 3 Parts: Fundus, body, neck - VAU (Vein, Artery, Ureter)

•• Hartman’s pouch is present at the neck →


Q
common site for stone
•• Cystic duct presents a spiral valve of Heister
(false valve).
Relations of Kidney (Magic of 1, 2, 3, 4, 5, 6)
Q
•• Artery: Cystic Artery.
•• Vein: Cystic Vein. •• 1 rib (12th) & 1 Vessel (subcostal) are related
to posterior surface of kidney (NOTE - 11th
•• Nerve: 10th cranial nerve.
rib & vessel on left side along with the above
Q
•• Lymph node: Cystic Lymph node of Lund . mentioned structures)

•• 2 Ligaments: Medial arcuate ligament and


Lateral arcuate ligament.

•• 3 nerves: subcostal Nerve (T12), Iliohypogas-


tric Nerve, ilioinguinal Nerve.
73
Anatomy

•• 4 muscles: diaphragm, psoas major, quadratus (Anterior surface) → left Suprarenal gland,
lumborum, transverse abdominis. colon, intestinal loop, stomach, spleen, pancreas.

•• 5 Structures impression on right kidney


(Anterior surface) → right Suprarenal gland, PERINEUM AND PELVIC VISCERA
colon, intestinal loop, liver, 2nd part of •• Anterior triangle (Urogenital triangle)
duodenum. •• Posterior triangle (Anal triangle)
•• 6 structures impressions on left kidney

•• Lateral – Ischial Tuberosity


Boundaries of perineum:
•• Anterolateral- Ischiopubic rami
•• Anterior- Pubic symphysis
•• Posterolateral- Sacro tuberous ligament
•• Posterior- Sacral and Coccyx
74
Cerebellum Quick Revision Notes

Urethra Central tendon of perineum (Perineal


Parts of Urethra:
body)
•• Insertion of 10 muscles in perineal body
•• Prostatic urethra (Horseshoe shaped)
–– Superficial transverse perineal muscle-2
•• Membranous urethra
–– Deep transverse perineal muscle-2
•• Bulbar urethra
–– Bulbospongiosus muscle-2
•• Penile urethra
–– Anterior fiber of levator ani-2
–– External anal sphincter 1
–– External urinary sphincter 1
75
Anatomy

OBG-Anat- Surgery Integration


Damage the perineal body →→ Paralysis of all 10
muscles→ Prolapse of uterus and rectum

Pelvic diaphragm and Urogenital Diaphragm



Levator ani + Ischiococcygeus (IsC)

Pubococcygeus (PC) and iliococcygeus (IC)= Levator
Q
Ani
Pelvic diaphragm = Ilium + Pubis + Ischium
Q Episiotomy process on the postero-lateral
Pelvic diaphragm = IC+ PC + IsC
part to avoid injury to perineal body.
Anat-OBG Integration:
76
Cerebellum Quick Revision Notes

Supply of pudendal nerve Urinary bladder: Magic of 4


Urogenital part supplied by the pudendal nerve. Tetrahedral structure has 4 angles, 4 borders, 4
surfaces and 4 connections
Relations of urinary bladder:
•• Right and left ureters – posterolateral side
•• Inferiorly- Urethral opening
•• Anteriorly- Median umbilical ligament

Clinical Significance
Pudendal nerve supplies the penis and gives a branch
to the scrotum as posterior scrotal nerve therefore,
for any hydrocele surgery pudendal nerve is to be
blocked.
Relation with Posterior surface of bladder -
Pelvic Viscera
•• Rectovesical pouch
•• Vas deferens located here
•• Seminal vesicle
•• fascia of Denonvilliers
•• Prostate gland

Extra Edge:
–– Fascia of denonvilliers' fascia- behind
urinary bladder
–– Fascia of Waldeyer- behind rectum
77
Anatomy

–– Posterior
Surgery Integration- Urethral InjuryQ
–– median
•• If rupture of bulbar or penile urethra- urine
–– right lateral
accumulates into Superficial perineal pouch,
scrotal and penile area. In very severe cases –– left lateral
it may reach up to the clavicle.
•• If posterior Urethra gets ruptured →
urine will come out from urinary bladder→
accumulate into deep perineal pouch → No
swelling over scrotum & Perineum area

Clinical Anat-Surgery-Radio Integration


•• Posterior Urethra = Prostatic Urethra +
Membranous Urethra •• Ejaculatory duct opens in Prostatic urethra
•• Anterior Urethra = Bulbar Urethra + Penile (Semilunar/Horse-shoe shaped → due to
Urethra pressure by median lobe)
Radio images of urethra •• Opening of prostatic utricle (remnant of
Paramesonephric duct) → in Prostatic urethra
•• Paramesonephric duct → Forms Uterus and
vagina in females

Anat-Physio Integration
•• Retrograde ejaculation or Urinary reflux - Inhibited
by Internal urethral sphincter (Involuntary →
Q
Smooth muscles)
•• TURP (Transverse Urethral resection of Prostate)
– Lead to removal of Internal urethral sphincter
Prostate gland causing Retrograde ejaculation and Urine reflux.
Q

•• 5 Anatomical lobes : •• Urine reflux → Causes increased risk of infections.

–– Anterior
78
Cerebellum Quick Revision Notes

Clinical integration:
•• Benign tumor - BPH (Benign prostatic
Hyperplasia) → Compression symptoms as it
occurs in Periurethral zone - Urinary hesitancy,
Urgency and increased Frequency.

•• Prostate carcinoma – Peripheral zone involved Abdominal Aorta


It is divided into Two Common Iliac artery, which
Anat-Pharma-Medicine Integration further divided into two -

BPH (Benign prostatic Hyperplasia/Hypertrophy) 1. External iliac artery

•• Management - Alpha-blockers (Prazosin, 2. Internal iliac artery


Phentolamine) → Anterior division - supply Pelvic viscera
•• Side effect of prazosin – Postural hypotension •• Blood supply of Urinary Bladder - Superior &
→ prescribed at night before sleep and gradually Inferior vesical artery
increases in dose.
•• Blood supply of URETHRA - inferior vesical
•• Selective Alpha-1a blocker - Tamsulosin (Less side artery (No need in female)
effects and selective)
•• Blood supply of Rectum -
–– Along with 5-alpha reductase inhibitors – –– Superior Rectal artery - from Inferior
Finasteride Mesenteric artery ranch
•• If Medical management fails - Surgery (TURP) –– Middle Rectal artery - from Internal Iliac
artery
–– Inferior Rectal artery - from Internal
Pudendal artery
79
Anatomy

Q
→ Posterior division
Branches from Posterior Division of Internal iliac
artery
(Mnemonic - ILS - I Love Salman)
1. Iliolumbar artery
2. Lateral sacral Artery
3. Superior Gluteal artery

Female Genital Organ

1. Fallopian tube
Length : 10 cm.
80
Cerebellum Quick Revision Notes

Parts: –– Antero-superior to Fallopian tube


a. Fimbria
Spaces/Pouches in Female
b. Ampulla - site of fertilisation
c. Isthmus - Physiological sphincter
d. Intramural part - Anatomical sphincter

→ Most dependent part in supine position - Pouch of


Q
Douglas (Recto-uterine pouch)
→ Most dependent part in supine position -
Q
Hepatorenal/Morrison’s Pouch

Supports of uterus
Q
2. Uterus 1. Primary Supports/ Major Support

a. Fundus •• Muscular/ Active Support: (Mnemonic: PPU)

b. Body 1. Pelvic Diaphragm

c. Cervis 2. Perineal Body

Uterine Artery - Tortuous Course and present in 3. Urogenital Diaphragm


broad ligament at the lateral border of Body of •• Fibromuscular & Mechanical Support:
Uterus → anastomosis with ovarian artery. (Mnemonic: PUT RU)

Anat-Obg-Surgery Integration 1. Pubocervical Ligaments

•• HYSTERECTOMY - surgical removal of 2. Uterosacral Ligaments


uterus prevent ureteric injury and ligation as 3. Transverse-Cervical (Cardinal; Macken-
it is in close relation with the uterine artery rodt’s)
at the lower part of the body of the uterus.
4. Round Ligaments of The Uterus
Angle of Anteversion and Anteflexion 5. Uterine Axis

Function - Prevention of Uterine Prolapse.


•• Relation at Cornua of Uterus (Posterior to
Anterior)
2. Secondary Supports/ Minor Supports (BRU)
1. Ovarian Ligament
1. Uterovesical Fold of Peritoneum
–– Postero-inferior to Fallopian tube
2. Rectovaginal Fold of Peritoneum
2. Fallopian tube
3. Broad Ligaments
3. Round ligament of uterus (Homologous to
Spermatic Cord)
81
Anatomy

Basics of Histo-Pathology How to Identify Epithelium


•• Histo- Pathology is important for Diagnosis of Characteristic of Nucleus Epithelium
Clinical Diseases Nucleus parallel to Flat squamous epithelium
•• 2 basic requirements to observe cell structures Basement membrane
Rounded nucleus Cuboidal epithelium
1. Microscope Nucleus perpendicular to Columnar epithelium
Basement membrane
Different powers of the Objective lens are used for
various purposes.

2. Contrast agent/ Dye/ Staining


colour
•• Pink/Red - Eosin Dye → AcidIc dye, hence
attracted by Cytoplasm (pH = 7.3-7.4).
•• Blue/Black - Hematoxylin → Basic dye, hence
attracted by Nucleus (Acidic histone proteins).
•• Cell membrane - just like cytoplasm → Appear
Pink (line structure)
82
Cerebellum Quick Revision Notes

Anat-Pathology Integration 3. After fixing → Cutting of tissue along with a


paraffin block of desired thickness (5 micron)
Steps for making slides using Microtome.

1. Submerge tissue sample in Formalin solution.

4. Put a tissue ribbon into the Water bath & then


onto the slide
2. L-block is used → Put Paraffin Wax and tissue
sample inside the space created by L-block.

5. Dip in acetone/ alcohol (organic solvent) followed


by Staining.

EPITHELIUM
•• The term "epithelium" refers to layers of cells that line hollow organs and glands.
•• It is also those cells that make up the outer surface of the body
83
Anatomy

1. Squamous epithelium b. Cornea,


•• Nucleus is parallel to the basement membrane . c. Nasal vestibule,
•• Cells are flat & pavement like in appearance. d. Oral cavity,
•• Function: Gaseous / Nutrition exchange, e. Tip of urethra and
Diffusion like Lung alveoli, endothelium, henle’s f. Glans penis,
loop
g. lower part of Vaginal canal,
h. lower part of Anal canal etc.

2. Cuboidal epithelium
•• Nucleus is rounded .
•• All dimensions of cells are equal hence cube
like appearance.
•• Function : Synthesis and secretion

•• 2 types - Simple or Stratified


•• Stratified squamous epithelium are of 2 types
1. Keratinized Stratified squamous epithelium (Dry
area) - Skin
2. Non-keratinized Stratified squamous epithelium
Q
(Wet areas with Friction) - E.g .
a. Conjunctive,
84
Cerebellum Quick Revision Notes

•• In Thyroid, PCT and DCT. •• Triad of Kartagener Syndrome (Mnemonic -


SBI)
•• Update - Thyroid with high activity – Simple
Columnar epithelium S - Situs inversus
B - Bronchiectasis
Thyroid with low activity – Simple Squamous
epithelium I - Infertility
Q
•• Thyroidization of Kidney → Chronic Pyelonephritis
[Anat-Patho-Medicine Integration]
causes thyroid-like appearance of PCT & DCT of
kidneys.

3. Columnar epithelium
•• Nucleus is Perpendicular to the basement
membrane .
•• Height of the cell is more.
•• Function : Synthesis and Storage function
•• Stored material present in the Apical area.
4. Urothelium/Transitional Epithelium
•• Present in the urinary system hence known as
Urothelium.
•• It shows transition / change in appearance
when stretched hence known as transitional
epithelium.
•• Upper most cell appear like Umbrella hence
known as Umbrella cells.

Umbrella cells :
Q

•• No colour - •• Thick glycoprotein layer present → Prevent


absorption.
1. Fat - globular / Rounded Shape
•• May appear Binucleate.
2. Air - Uneven/ Irregular Shape
•• Internalisation of cell membrane present.
•• Present in Urinary system distal to the
Collecting duct like Bladder, Urethra (Except
Membranous part & tip of urethra) etc.

GLANDSQ
3 types on the basis of Mode of Secretions

A. Holocrine glands - Entire glands are ruptured


and secretion released. E.g. Sebaceous
Glands. Block of duct → Acne occurs.
Clinical integration : B. Apocrine glands - Only the apical portion of
the gland is ruptured and secretions released
•• Immotile Cilia Syndrome/Kartagener
in the surroundings or duct. E.g. Modified
Syndrome - because of absence of Cilia/
Sweat gland → Pheromones secreting glands
Dysfunctional cilia → repeated Infections →
repeated inflammation → Dilatation/ Ectasia (present in Axilla, Perineum) Breast.
of bronchus occurs → aka Bronchiectasis.
85
Anatomy

C. Merocrine / Eccrine glands - Secretions


show Exocytosis (No destruction). E.g. Sweat
glands of Palms and soles.
HAIR
KERATIN

SEBACCOUS

GLAND
•• Cardiac Muscle
(HOLOCRINE)

Muscle

•• Smooth Muscle

CARTILAGE & BONE


A. Cartilage
•• Skeletan Muscle
•• Identify (Chondrocytes) - contain Chondroitin
Sulphate → Attract basic stain → Blue stain.
•• On histology slide -
Group of cells with blue stained nucleus and clear
Lacuna.
Q

1. Hyaline cartilage F - Foetal cartilage

(Mnemonic - Hii - GF - CAR) C - Costal cartilage

Hii - HYAline A - Articular cartilage

G - Growth plate R - Respiratory tube cartilage


86
Cerebellum Quick Revision Notes

2. Elastic cartilage T - Tip of nose

(Mnemonic - ETC NEWS) T - Tip of arytenoid cartilage

E - Ear pinna T - Tritiate cartilage

E - Eustachian tube C - Corniculate

E - Epiglottis C - Cuneiform

3. Fibrocartilage I - Intervertebral Disc


(Mnemonic - FIAT Logo) A - Articular disc (Menisci, TMJ disc)
F - Fibrocartilage T - Tendon insertion
Logo - Labrum (of Glenoid & Acetabulum)
87
Anatomy

Collagen NEET Pattern 2022


•• Most common protein in body
•• Types = 28

Important
Types of Locations
collagen
I Bone, Aponeurosis & Ligaments (BAL)
II Cartilages
IV Basement Membrane (Kidney, Lungs)

Few Recent Question

FMGE 2021

NEET PG 2021

Pharyngeal Apparatus
•• Pharyngeal apparatus : Special structures
developing near pharynx
•• Pharyngeal Apparatus : Pharyngeal Arches +
Ph. Pouch + Ph. Cleft + Ph. Membrane
•• Arches- Mesodermal thickenings
•• Pouches – Endodermal out bulgings
•• Cleft – Ectodermal in dipping
•• Pharyngeal membrane : Meeting points in
between ectoderm, mesoderm & endoderm in
pharyngeal apparatus.
88
Cerebellum Quick Revision Notes

•• Endoderm, mesoderm, ectoderm meeting Pharyngeal Arch


→ Pharyngeal membrane → 1st pharyngeal
•• Mesodermal in origin
membrane → tympanic membrane.
•• 6 arches develops
•• 5th arch disappears
•• so 5 arches persist
89
Anatomy

Pharyngeal Nerves:
Pharyngeal Arch Pharyngeal nerve
I → V3
Pharyngeal Arch: Mesodermal
II → VII
Each pharyngeal arch gives rise to 4 structures:
III → IX
a. Pharyngeal nerve IV → Superior Laryngeal Nerve > X
b. Pharyngeal muscle VI → Recurrent Laryngeal Nerve > X
c. Pharyngeal cartilage
•• Along caudal border → Pre- trematic nerve
d. Pharyngeal artery
90
Cerebellum Quick Revision Notes

Pharyngeal Muscles Pharyngeal Arch Cartilages


•• 1st arch muscles → These are supplied by V3.

→ 4 muscles of mastication:

1. Temporalis

2. Masseter

3. Med. Pterygoid

4. Lat. Pterygoid

(MAT 2):

•• Mylohyoid

•• Anterior Belly of Digastric

•• Tensor Tympani

•• Tensor veli Palatini

•• 2nd arch muscles → These are supplied by the •• 1st arch cartilage: Meckle’s cartilage → 5 ‘M’
Facial nerve. (SPF)
1. Malleus & Incus
1. Stylohyoid
2. Malleolar (Anterior) ligament
2. Stapedius
3. SphenoMandibular ligament
3. Platysma
4. Mandible
4. Posterior Belly of Digastric
5. Maxilla
5. Facial muscle except Levator Palpebrae
•• 2nd arch cartilage: Reichert cartilage → 5 ‘S’
Superioris (eye muscle)
1. Stapes (except footplate & Otic capsule)
•• 3rd arch muscle:
2. Styloid process
Stylopharyngeus
3. Stylohyoid ligament
•• 4th arch muscle → All are supplied by 10th nerve
4. Smaller cornu of hyoid
1. All muscles of palate except Tensor Veli
Palatini (1st arch : V 3 ) 5. Superior ½ of body of the hyoid

2. All muscles of pharynx except •• 3rd arch cartilage:


STYLOPHARYNGEUS (3rd arch : IX nerve) 1. Greater cornu of hyoid
3. Cricothyroid → by 10th < SLN < RLN 2. Inferior ½ of hyoid
•• 6th arch muscle → supplied by RLN •• 4th & 6th arch cartilage:
All larynx muscle except Cricothyroid –– Laryngeal cartilage
91
Anatomy

First Arch Syndromes


Pharyngeal arch Nerve Muscle Cartilage
1 st
V3 •• 4 muscles of mastication 5M
•• Mylohyoid
•• Anterior Belly of Digastric
•• Tensor Tympani
•• Tensor veli Palatini
2nd VII •• Stylohyoid 5S
•• Stapedius
•• Platysma
•• Posterior Belly of Digastric
•• Facial muscle except Levator Palpebrae
Superioris (eye muscle)
3rd IX Stylopharyngeus Greater Cornu & lower ½ of hyoid
4 th
SLN / ELN Palate, pharynx, CT Laryngeal cartilage
5 th
Disappear - - -
6 th
RLN Larynx Laryngeal cartilage

All muscles of Supplied by Except Supplied by


Palate IX → X < VAC < pharyngeal plexus TP V3
Pharynx X Stylopharyngeus IX
Larynx X (recurrent laryngeal) Cricothyroid X (superior laryngeal)
Tongue XII Palatoglossus X

Pharyngeal Cleft
•• Ectoderm is dipping in between 2 arches known as Pharyngeal cleft.
•• 1st cleft: forms External auditory canal → 6 hillocks develops around 1st cleft → pinna is formed

•• 2nd, 3rd & 4th clefts disappear.

–– 2nd arch grows rapidly & adjoins with basal part. In between (includes 2nd, 3rd, 4th cleft), it makes
cervical sinus
92
Cerebellum Quick Revision Notes

Any embryonic tract with abnormal :


•• 2 openings: fistula
•• 1 opening: sinus
•• No opening: cyst

Pharyngeal Pouch
•• Endodermal out bulging in between 2 arches
Clinical :
known as pharyngeal pouches.
•• total 4 in no. DIGEORGE SYNDROME:
•• 1st pouch: make auditory tube (tubo + tympanic •• Features Vary Widely
recess) and tympanic membrane cavity.
•• CATCH 22
•• 2nd pouch: make tonsil
•• Cardiac Abnormality (Interrupted Aortic
Arch, Truncus Arteriosus, Tetralogy of Fallot)
•• Abnormal Facies
•• Thymic Aplasia
•• Cleft Palate
•• Hypocalcemia/Hypoparathyroidism
•• Velocardio-Facial Syndrome or Shprintzen
Syndrome:

•• 3rd pouch: dorsal part makes 2 inferior –– Cause: Microdeletion of chromosome 22


parathyroid glands ; ventral part makes thymus.
•• 4th pouch: dorsal part makes 2 superior
parathyroid glands ; ventral part makes lateral
lobe of thyroid.
93
Anatomy

Pierre Robin Syndrome (GCR)


•• Anomalies of chromosome 17>> 2/ 11
–– Glossoptosis
–– Cleft Palate
–– Retrognathia

First Arch Syndromes GENERAL EMBRYOLOGY


•• First Arch Syndromes occur due to the failure Study of Formation, Development and Maturation of
of the migration of neural crest cells into the the Embryo is known as Embryology.
first arch.
•• Site of fertilization – Ampulla > Ampullary-
•• The First Arch Syndrome includes Treacher- Isthmic Junction
Collins Syndrome and Pierre-Robin Syndrome •• Morula – 16-cell stage (3rd day)
1. Treacher-Collins Syndrome •• Advanced morula : 32 cell stage without cyst
2. Mandibulofacial Dysostosis > 16 to 32 cell stage

1. Treacher-Collins Syndrome •• Blastocyst – on 4th day, 32 cell stage with


cavity
•• Mandibulofacial Dysostosis
•• Implantation of Blastocyst – 6th day (6th –
–– Mutation in Chromosome 10th/12th day)
–– Mandibular Hypoplasia
94
Cerebellum Quick Revision Notes

•• Blastocyst shows 2 types of cell lines : → it decides cranio-caudal axis


a. Inner cell mass : Forms 1st cell line of •• Opposite to prechordal plate → proliferation
embryonic stage known as Epiblast → Forms of epiblast occurs → forms primitive streak →
true embryo decides right & left side of embryo.

b. Outer cell mass : Forms trophoblast → •• Formation of a new layer in between the
gives nutrition to embryo → divides into Epiblast and endoderm is the Mesoderm.

i. Cytotrophoblast (CTB) •• Remaining cells of epiblast → converted into


ectoderm.
ii. Syncytiotrophoblast (SCTB)
•• Sequence of Germ cell layer formation –
•• Epiblast cells give rise to → Hypoblast cells →
–– 1st – Endoderm
Hypoblast forms the Endoderm
–– 2nd – Mesoderm
•• Bilayered Embryonic Disc
–– Epiblast –– 3rd – Ectoderm (last)

–– Endoderm –– Neural crest cells are considered as 4th


germ cell layers.
•• Endoderm proliferates → forms Prochordal
plate – it is formed near Cranial end of embryo
95
Anatomy

General Embryology Simplified


96
Cerebellum Quick Revision Notes

Mitochondrial Inherited Disease – common in Mesoderm


Maternal side
3 Parts -
•• Outer cell mass – forms the Trophoblast
L – Lateral plate Mesoderm
•• GIT is lined by – Endoderm
I – Intermediate Mesoderm
•• 3 Germ Cell layer formation – Gastrulation
P – Para Axial Mesoderm
97
Anatomy

•• Amniotic cavity - Nutrition to the baby –– Bucco-pharyngeal membrane – forms the


oral cavity
•• Outer layer – Ectoderm
–– Anal membrane – forms Anal Canal
•• Mesoderm develops everywhere except for 2
areas – ƒƒ If mesoderm persists here – causes Anal
98
Cerebellum Quick Revision Notes

Stenosis – Fecal material will not pass – Important Days


requires surgery
•• 2 Days: 2 Cell Stage
•• Vitello – Intestinal duct
•• 3 Days: Morula Formation
Surgery-Anatomy Integration •• 4 Days: Blastocyst
Vitello – Intestinal duct •• 8 Days: Bilaminar Disc Formation
•• Remnant → Meckel’s Diverticula (2% Rule) •• 14/ 15 Days: Head & Tail End Is Decided
[Prochordal Plate & Primitive Streak Appears]
•• 16 days: Gastrulation /3 Layered Embryonic
Disc Is Formed /IEM Appears
99
Anatomy

CNS DEVELOPMENT •• Vitamin B9- Folic acid is required for neural


tube formation in fetus

–– ↓Vitamin B9 – NTD
•• Notochord disappears
–– Remnant of Notochord – Nucleus Pulposus
(inside the IVD)

Anterior Neuropore defect


Cranial vault is absent
•• Exophthalmos
•• Chin resting over thorax

•• Vth Ventricle: Cavity of Septum Pellucidum •


VIth Ventricle: Cavum Vergae
–– False/ misnomer
•• The 2 corners meet together and form a tube
like structure known as Neural Tube
–– Opening on the above - Anterior Neuropore
(closed on 25th day)
–– Another opening on the downside – Posterior
Neuropore (closed on 28th day)
•• Complete Neural tube formation is known as
Neurulation and is completed by 28th day of
IUL

–– Multiple fusion sites

ƒƒ 1st site – cervical region

•• If no proper fusion of the neural tube, this is


known as Neural Tube Defect (NTD)
100
Cerebellum Quick Revision Notes

–– Absence of Hyaluronic acid – leads to


Q
NCC migration failure
ƒƒ Achalasia Cardia
ƒƒ Hirschsprung Disease

NCC Derivatives
Tricks & Magic
•• BHU – Banaras Hindu University, founder was –
MMM – Madan Mohan Malviya
Q
•• PAEDS doing DiSCo in front of MMM
–– P- Parasympathetic ganglion
Posterior Neuropore defect –– E- Enteric plexus – Auerbach plexus
A. Spina bifida occulta – two halves not uniting –– A- ANS ganglion
together but SC and meninges are in their original
–– D- Dorsal root ganglion
position.
–– S- Schwann cell
•• Only tuft of hair seen
–– D- Dentine tissue[odontoblast]
•• No symptoms
–– S- Sclera
B. Spina Bifida with meningocele – meninges coming
out with CSF –– C- Choroid, connective tissue of thyroid,
parathyroid, thymus, connective tissue of
C. Spina Bifida with meningomyelocele – meninges gland –lacrimal, nasal, oral, salivary gland,
coming outside as well as nerve elements palatine gland.
D. Spina Bifida with Myeloschisis – meninges getting –– M- Melanocyte
ruptured so CSF leaking out and spinal nerves visible –– M- Mesenchymal bone of HEAD & FACE
on the back side
–– M- Meninges
–– Adrenal medulla

NCC
•• Terrorist cells – can go anywhere and do their
job
•• During embryonic development there is
release of the hyaluronic acid and creates the
pathway. Through these pathways NCC migrate
to different parts of the body
101
Anatomy

GIT DEVELOPMENT

which forms the Urinary Bladder and Rectum.


Surgery-Anatomy Integration
Vitello – Intestinal duct
•• Remnant – 1 opening – is Meckel’s Diverticula
(2% Rule)
•• 2 opening – Fecal Fistula (FMGE 2021)

•• Mesoderm develops everywhere except for 2


areas –
–– Stomodeum - Bucco-pharyngeal membrane –
forms the oral cavity
–– Proctodeum - Anal membrane – forms Anal
Canal
•• If mesoderm persists here – causes Anal
Stenosis – Fecal material will not pass –
requires surgery
•• Foregut – from stomach up to the opening of
the CBD

•• Midgut – from 2nd part of the duodenum up to


the Ileum

–– Artery of the midgut – SMA – for Rotation


of the GIT

ƒƒ Part above the artery – Pre Arterial


Pedia/Surgery-Anatomy Integration segment
Proctodeum
Persistence of mesoderm results in ƒƒ Part below the artery – Post Arterial
segment
•• Anal Atresia
•• Anal Stenosis •• Hindgut – from left 1/3rd of the transverse
•• Distal to the Allantoic diverticulum is the cloaca colon up to to anal canal (dentate line)
102
Cerebellum Quick Revision Notes

Development of Tongue

Embryonic part Nerve supply


Part of tongue
from which derived General sensation Taste Motor
Mandibular (V3) – Chorda tympani (branch
Anterior 2/3rd 1st Arch Q Q -
lingual nerve of VII)
Glossopharyngeal Glossopharyngeal nerve
Posterior 1/3rd 3rd Arch Q -
nerve (IX) (IX)
Posterior most part 4th Arch Vagus nerve (X) Vagus nerve (X) -
Muscles of the tongue Occipital myotome - Hypoglossal nerve (XII)
Pharyngeal plexus > VAC >
Palatoglossus muscle - Q
X Nerve
103
Anatomy

CVS DEVELOPMENT •• The development of this area will create the


beginning of the formation of the heart. So
•• In the embryo, development of the heart this area is known as the Cardiogenic area
starts near the 3rd week of intrauterine life
•• Development of the heart starts near the
i.e., 21st day
pharynx
•• Bilaminar disc where there is development of
–– In case of MI/ Angina – pain can be referred
intraembryonic mesoderm developing from the to the neck area near to the left jaw
epiblast cells and proliferating near the head end.

•• Ventricles will proliferate and will come Mnemonic – BVAS: BV Aur Saas (kyunki saas
forward and downward. Atria will go backwards bhi kabhi bahu thi)
Q

and superiorly.
•• B - Bulbus cordis
•• Now the heart shows a conical structure where •• V – Ventricle
Atria is above and posteriorly – forms the base •• A – Atria
of the heart.
•• S – Sinus venosus
104
Cerebellum Quick Revision Notes

•• Umbilical vein on the left side forms


Ligamentum Teres

•• Formation of the spiral septa – Conotruncal


septa derived from the NCC – this will help
Pedia/Medicine- Anatomy Integration
the blood from the left ventricle be pumped
Failure in the development of the spiral septa, there
will be the common trunk of AA and PT – Truncus to the AA and from the right ventricle to the
Arteriosus (Right to Left shunt) pulmonary trunk.
105
Anatomy

3. Ebstein’s Anomaly
Pedia/ Medicine- Anatomy
Integration 4. Transposition of Great Vessels
Coeur en Sabot (French for “clog-shaped heart”) 5. Truncus Arteriosus
Or 6. Total Anomalous Pulmonary Venous Return
Boot shaped heart (TAPVR)

Pedia/Medicine- Anatomy Integration


•• Failure in the development of the spiral septa,
there will be the common trunk of AA and PT
– Truncus Arteriosus (Right to Left shunt)
Pharyngeal arch Artery
•• Spiral valve not developing in the midline
and is shifting then Aorta is occupying more
space while Pulmonary valve becomes narrow
causing -Pulmonary Stenosis
•• Due to the shift of the spiral valve the
interventricular septum covers more area of
the Aorta which is further covering the area of
both the ventricles – Overriding of the Aorta

CHD: Left to Right Shunt (Anat- 1. 1st ,2nd & 5th arch arteries disappear
Medicine-Pedia Integration) 2. Remnant of 1st arch artery → Inferior alveolar
•• ASD artery – Br. of Maxillary Artery
3. Remnant of 2nd arch artery → Stapedial artery &
•• VSD
Hyoid artery
•• PDA
4. 3rd arch artery → divides into 2 parts:
•• Ductus Arteriosus Aneurysm
a. Proximal part → CCA
•• Aortico -Pulmonary Window
b. Distal part → ICA

CHD: Right to Left Shunt (Anat- c. New growth → ECA

Medicine-Pedia Integration) •• 4th arch artery →

1. Tetralogy of Fallot a. On right side: Right subclavian artery

2. Tricuspid Atresia b. On left side: Arch of Aorta


106
Cerebellum Quick Revision Notes

•• 5th arch artery → disappears


–– Proximal part: right pulmonary artery
–– Distal part: disappears
•• 6th arch artery →
–– On right side
ƒƒ Proximal part: right pulmonary artery

ƒƒ Distal part: disappears

–– On left side

ƒƒ Proximal part: Left pulmonary artery

ƒƒ Distal part: DUCTUS ARTERIOSUS →


Q
Ligamentum Arteriosum (on left side)
PHYSIOLOGY
QUICK REVISION NOTES

GENERAL PHYSIOLOGY
Homeostasis & Regulatory System. Positive Feedback
•• Walter B. Cannon •• H2
•• Milieu Intérieur - Claude Bernard. * (Father of –– Head’s paradoxical reflex
Physiology)
–– Hodgkin cycle
•• Regulatory System – Feedback and Feedforward
•• C – Clotting cascade
•• L – LH surge
•• A – Activation of digestive enzymes
•• M – Milk let down reflex
•• P – Parturition Reflex

Feedforward Mechanism
•• Anticipatory Control
•• Correction Occurs Even Before the Variable
Has Changed.
Q
–– e. g. Temperature Regulation
–– ↑ HR Before Exercise
–– Cephalic Phase of Gastric Acid Secretion

Gain
Efficiency of Regulatory Mechanisms.

Cell Membrane
Singer & Nicolson - Fluid Mosaic Model
Most Common Regulatory Mechanism → Negative
Protein Lipids CHO
Feedback.
50 -55% 45-50% ≤ 5%
Positive Feedback Phospholipid → Most Common Lipid
A→⊕B→⊕C Lecithin or DPPC
Sphingomyelin
Negative Feedback
A→⊕B→-C Protein: Lipid Ratio* = 1:1
•• Very High in Inner Mitochondrial Membrane
Q
–– Exception – Reversal – Myelin sheath
110
Cerebellum Quick Revision Notes

Solubility –
•• Lipid Soluble – can easily cross the membrane
•• H2O Soluble – cannot cross the membrane

Lipid – Functions
1. Solubility
2. Structural Integrity
3. Flexibility & Fluidity Q → Cholesterol
•• Lipid which is never present in the CM –
Proteins (Integral & Peripheral)
Triglyceride (TG)Q
•• Act as Channel (Aquaporins), Pumps, Receptor,
Transporter, Carrier, Ag etc.

RBC Membrane
•• GPI anchor protein
–– DAF
–– MIRL
(Prevent the hemolysis by inhibiting complement
activation)
111
Physiology

•• In the absence of CD59 & CD55, complement at night) and leads to hemolysis. → Paroxysmal
proteins will not be inhibited anymore (activated Nocturnal Hemoglobinuria.

Cytoskeleton
Function Protein

Mechanical SupportQ
Microfilament F & G ActinQ
Muscle Contraction Diapedesis

Cytokeratin DesminQ
Intermediate Filament Cell Markers
Vimentin, GFAP

Microtubule Intracellular Transport Cell Division α & β Tubulin

Intercellular Junction •• Actin, Catenin, Cadherin

1. Gap junctions –
•• Quick Transport (ICF of 1 cell→ICF of another)
•• Syncytium
•• Connexin→ Connexon

Body Water

2. Tight Junctions -
•• Claudin, Occludin, JAMQ
•• BBB (tight tight), Nephron (both – tight tight &
leaky tight)

Estimation of Body Fluids

3. Adherens Junction
•• Dye Dilution / Indicator Method
•• Anchor Cell Together
•• V = A-e / C
•• Strength
112
Cerebellum Quick Revision Notes

••

•• Hypotonic
–– Loss – DI
–– Gain – SIADH, Distilled (Tap) Water

•• Hypertonic
–– Loss – Hypoaldosteronism – not able to
absorb Na+.
–– Gain – 2% NaCl
Direct & Indirect methods of
measurementsQ
•• Direct
–– TBW→ D2O, T2O, aminopyrineQ
–– ECF → SIMaN
Transport Across CM
ƒƒ S – sucrose
•• Active
ƒƒ I – InulinQ
–– Against the gradient
ƒƒ Ma – mannitolQ
–– Needs ATP
ƒƒ N – Na+ thiosulfate
–– Uphill movement
–– PlasmaQ→ Evan’s blue dye, Radiolabelled •• Passive
Albumin
–– Along the gradient
–– RBC → Cr / Fe labelled RBC
–– Does not need ATP
–– Blood volume = Plasma / 1- Haematocrit
–– Downhill movement
•• Indirect Q
Active Transport
–– ICF = TBW – ECF
1. Active Transport
–– Interstitial fluid = ECF – plasma
–– Pump
Shifts of Body water –– Directly uses energy
•• Darrow Yannet DiagramQ –– E.g., Na+ K+ ATPase
•• Isotonic 2. Active Transport
–– Loss – Vomiting, Diarrhoea, Hemorrhage –– Symport/ Antiport

–– Gain – Infusion of Normal Saline –– Usually depended on Na+ K+ ATPase pump


–– Indirect use of energy
–– E.g., SGLT, NCX (Na+ Ca2+ Exchanger)
113
Physiology

Vesicular Transport
•• Endocytosis
•• Exocytosis
•• Transcytosis

Passive Transport
Diffusion & Osmosis

Revision Summary
•• Most Common Regulation - Negative Feedback
•• Main Lipid in Membrane – Phospholipid – Lecithin
•• Barriers – Tight junctions (BBB)
•• Syncytium – Gap junctions
•• Body Water Rule – 60:40:20 – TBW: ICF: ECF
•• Dye used – ECF - SIMaN
–– Plasma – Evan’s Blue dye
•• Simple→ along the gradient, Ex: - Gases, lipid •• Active – Against the gradient
soluble •• Passive – Along the gradient
•• Facilitated→ require carrier protein, saturation
Nerve Muscle Physiology
– Ex:- GLUT,Q specificity, competition (NEET
2023) •• CM – Semipermeable
•• Non-ionic→ NH3 & H+→ NH4+ •• Diffusible Ions – K+ >> Cl-

Receptor Mediated Endocytosis (INICET)* •• Non-diffusible - Others- Na+, Ca2+, HCO3-

•• Protein- Clathrin, Caveolin Gibbs Donnan Equilibrium –


•• Detachment - Dynamin –– Effect of Non-Diffusible Ion on the
Distribution of Permeant Ions
Cations Anions
ICF K , Mg2
+ +
PO43-, Protein
ECF Na+, Ca2+ Cl-, HCO3-

Equilibrium Pot. - Nernst Equation


•• Single Ion – EP
•• Net Flux – Zero
•• Formula
E.P. = RT/ZF logn Co/ Ci
R = Gas Constant, T = Temperature (37°c), Z= Valency;
F = Faraday’s constant; Co = concentration outside,
Ci = concentration inside
•• RMP→ EP & P
•• GHK Equation*
114
Cerebellum Quick Revision Notes

RMP = £ (EPx × Px) + (EPy × EPy) + (EPz × EPz) Site of Genesis – AP


•• Motor - Initial Segment
EP
•• Sensory - Nodes of Ranvier
•• Na+ = +60 mv
•• (MIS SeNORita)Q
•• Ca2+ = +100 to 130 mv
•• Cl- = -70 mv
AP in Nerve – DinKer
•• Depolarisation – Influx of Na+
•• K+ = -90 mv
•• IPSP/ EPSP •• Repolarisation – K+ Efflux

RMP
•• Neurons = -70 mv
•• Skeletal & cardiac muscle = -90 mv
•• Smooth muscle = -50 mv
•• SA node, BER (GIT) = -40 to -60 mv
•• RMP Genesis → K+ >> Cl- >>> Na+ K+ ATPase
•• NoKiA - (Na+ Out K+ In ATPase)
•• Coupling Ratio – 3:2
•• Max ATP Consumption at Rest/ Basal Condition
→ BMR
•• Maintain High Na+ Out & K+ In - Maintain
Gradient→ RMP Genesis
•• Electrogenic Pump- MINOR ROLE

NOKIA – (INICET)
(Na+ Out K+ In ATPase)
•• OK-NAPQ
–– O – Ouabain/Digoxin blocker
–– K – K+
–– N – Na+
–– A – ATP
Refractility
–– P – Phosphorylation site
•• Absolute refractory period
–– No matter how strong the stimulus is, 2nd
response will not be generated.
–– Due to longer ARP, heart muscle can’t be
tetanized (sustained contraction)Q
•• Relative refractory period
–– Stronger stimulus (>>> threshold) can produce
a response 2nd AP
–– RRP→ abnormal stimulus → PVC (Extra
systole)Q
–– Most risky period for Arrythmia.

Nerve Injury
Sunderland’s Classification
115
Physiology

•• Endoneurium Antegrade - KinesinQ


•• Perineurium •• From Soma → Axon terminal.
•• Epineurium •• Carry growth factors, vesicle, Mitochondria

Retrograde - DyneinQ
•• From Axon terminal → Soma.
•• Carry empty vesicle, Toxins, viruses → Rabies,
Tetanus, Polio, HerpesQ
•• Perfect in Lungs, FT, sperms → they help in
movement of cilia

Erlanger & Gasser ClassificationQ


•• A → A α → Motor + ProprioceptionQ → thick
–– 1°- Compression→ NeuropraxiaQ myelinated
–– 2° - Axonotmesis→ Axon gets damagedQ –– A β → Pressure, Vibration, Touch,
–– 3° - Neurotmesis→ Axon + Endoneurium gets Stereognosis etc
damaged
–– A γ → Motor
–– 4° - Axon + Endo + Perineurium get damagedQ
–– 5° - Axon + Endo + Peri + Epineurium gets –– A δ → Fast PainQ, Temperature
damaged (complete transaction) •• B → Preganglionic Autonomic Nerve → thin
myelinated
MYELIN
•• C → Postganglionic Autonomic Nerve, Slow
CNS – OligodendrocytesQ PNS- Schwann cellsQ Pain,Q Temperature → thin unmyelinated

Lloyd & Hunt → Numerical Classification (Only


for Sensory)
•• I → Ia → Annulo-spiral endings → Muscle
SpindleQ
Ib → Flower-spray endings → GTOQ
•• II → Aβ
•• III → Aδ
•• IV → C (slow pain)Q

E-C coupling– Skeletal Muscle

Axoplasmic Flow
116
Cerebellum Quick Revision Notes

•• Excitation-contraction coupling - the excitation


of nerve causes excitation of muscle followed
by contraction of muscle
•• At NMJ, 2 Major roles of Ca2+ – Exocytosis &
Contraction
•• Neurotransmitters released by exocytosis
comes into synaptic cleft & goes towards the
motor end plate (neurotransmitters in skeletal
muscle: AchQ) & binds to Nicotinic receptorQ
•• When Ca+ bind to nicotine receptor, ligand-
gated Na+ channel openQ
•• Triad:Q Applied

–– 1 T-tubule •• RyR Gain of Mutation - Malignant Hyperthermia

–– 2 L-tubule (Sarcoplasmic reticulum) aka


Cisternae
ƒƒ These tubules have special receptor,
known as DHP receptor

Action potential→ + DHP receptorQ

MechanicalQ Attachment of T-tubule with L-tubule

RyRQ channel open

Allow Ca+ to free from Calsequestrin binding from to


sarcoplasQ

Contraction after binding of Ca2+ to troponin c Q


–– Lambert Eaton Syndrome: affects pre
Troponin- Ca2+ complex will remove tropomyosin
synaptic membraneQ
away from the active site of actin
ƒƒ Ab against VGCC, Ca2+ accumulates
Active sites of actin are free to bind with myosin head outside (repeated stimulus is required to
open the Ca2+ channel)
Sliding filament theory → causing contraction of the muscle –– Isaac’s Syndrome: Ab against VGKC, cause
117
Physiology

cramps in muscleQ •• Inside the cell, dystrophin is bound to


–– Myasthenia gravisQ : Is effect of post- Filamentous Actin protein.
synaptic membrane, Ab against Ach receptor •• Gower’s sign – Absence of Dystrophin in children

Skeletal Muscle – Sarcomere –– Mutated/ Partially present – Becker’s


Muscular DystrophyQ
–– Absent – Duchenne Muscular DystrophyQ

Length – Tension Relationship

•• ↑ Cross Bridge
–– ↑ FOC (Force Of Contraction) ↑ Active
Tension → upto a Physiological limit onlyQ

•• Optimum length of the sarcomere – 2 – 2. 2μm Revision Summary


Dystrophin •• Max. Divalent Cation In ICF – Mg2+
•• Eq. Pot. is given by - Nernst Equation (Formula)
•• NoKiA binding sites – OK-NAP
•• AP Ionic – Din (Depolarisation influx Na+) Ker
(K+ efflux - Repolarisation)
•• AP Genesis – MIS SeNOR
•• E-C Coupling – Skeletal – Triad (RyR)
•• Erlanger- Gasser Classification –
–– Hypoxia - B>A>C

–– Pressure – A>B>C

Cardio-Vascular System
Heart Action Potential- 2 Types of Tissues
118
Cerebellum Quick Revision Notes

Nodal Tissue –
Auto-rhythmicity
FuNNy - Na+ channels
Cat – Ca2+t (t – transient)
Pec – K+ early closure

Cardiac Muscle

Order Of Activation Cardiac cycle


•• Purkinje – Max gap junction – Max. velocity •• Duration – 0.8 sec
•• Depolarization – Endo to Epicardium Atria Ventricles

•• Repolarization - Epi to Endocardium Systole 0.1 0.3 (0.27)

•• Max frequency of AP – SA node Diastole 0.7 0.5 (0.53)

Last Part to Depolarized:Q Duration of Cardiac Cycle = 60 / HR


••
Left Part of IVS, Pulmonary Conus, Posterobasal
Portion of the LV (last part to depolarize)Q Cardiac cycle - Systole
•• S1 to S2
Heart Sounds
•• S1 – AV Valve Closure
•• S1 – AV closure
•• S2 – Semilunar closure
•• S3 – early filling (2/3rd)
–– Normal in pregnant woman
•• S4 – late filling (active)
–– Always pathological
119
Physiology

•• v wave – Venous Return


•• y wave – Atrial emptying

Volume Changes
•• End Diastolic Volume (EDV) – 120- 130 ml

–– Blood left/ filled in the ventricle at the end


of the diastole

–– Index – Preload (Depends on VR)Q

•• Stroke Volume (SV) – 60-70 ml

Cardiac cycle - Diastole –– Blood ejected/beat

•• Protodiastole – SL valve & Aortic valve are still •• End Systolic Volume (ESV)- 40-50 ml
open for fraction of second (0.04 sec)
–– Blood remained in the ventricle at the end of
•• S2 to S1 systole
•• S2 – SL Valve Closure
–– ESV = EDV – SV
•• Ejection Fraction (EF) – 55-65% → Ventricular
Efficiency

EF = SV / EDV ; if<40-45%-Heart failure

Pressure Volume Curve –NEET*

JVP ≈ CVP ≈ RAPQ

Mnemonic: MAAM COCO


Diastolic filling

•• a wave – Atrial Contraction M: Mitral valve Closure (a)
•• c wave – Tricuspid valve (TV) bulging into RA ↓
IVC
–– Isovolumetric contraction

•• x wave – Atrial relaxation
A: Aortic valve Open (b)
•• x’ wave – TV returning to normal position
120
Cerebellum Quick Revision Notes


Ejection

A: Aortic valve Closes (c)

IVR

M: Mitral valve Opens (d)

P-V Curve- Applied - INICET*

•• After QRS – 1st HS


•• After T wave – 2nd HS
•• b/w 2nd & 4th - 3rd HS
•• After P wave - 4th HS

Cardiac Output
•• CO = SV X HR
•• Cardiac Index = CO/BSA = 3.2L/min/m2
••

•• Aortic regurgitation – EDV ↑ = VO2 / CaO2 - CvO2


•• In case of Aortic regurgitation and Aortic
Measurement of CO
stenosis - ↑ afterload
•• Fick’s Principle – Best for Low CO
•• Aortic Stenosis –
•• Dye Dilution Method – Indocyanine GreenQ
–– Acute case – up & right
•• Thermodilution – Gold Std.
–– Chronic case – up & left
Swan Ganz CatheterQ
•• Mitral regurgitation - EDV ↑
•• ECHO – Most CommonQ
•• Mitral stenosis - EDV ↓ - graph shifts towards
•• Velocity Encoded Phase Contrast MRI – Most
the left and No ↑ in afterload.
Accurate * (INICET)
Wigger’s Diagram Regulation of CO
Frank Starling Law
Heterometric Regulation
•• Initial Length of Muscles Directly Proportional
to SV (Physiological Limit)
•• VR α FOC
•• EDV α SV
–– EDV ↑-↑ stretching (Passive tension) & ↑
121
Physiology

initial length Poiseuille Hagen law


ƒƒ ↑ cross-bridges F = δP/R
ƒƒ ↑ Troponin C sensitivity - ↑ FOC (Active R (Resistance) = 8ηL/πr4
tension) - ↑ output (SV)
Therefore, F = δPπr4/8ηL
Inotropic Effect- Homeometric
Most Important Factor = Radius (r) > DiameterQ
Regulation
•• CHF – Beyond Physiological limit CO = BP/TPR

•• T/T of CHF- Inotropic Drugs Therefore, BP = CO x TPR

Autoregulation of Blood flow


Organs Control Their Own Blood Flow By Changing
Their Resistance
Most Important Factor - Resistance
Best Autoregulation In
1. Heart → Via Adenosine (Vasodilation)
2. Brain → Aerobic Metabolism only
•• ↑ pCO2 - Vasodilation

Types of Blood vessels •• Hypoxia

1. WindKessel Vessel – Recoil – DBPQ – Aorta •• Hypercapnia → Most Important

2. Capacitance Vessels- Store Blood – Prevent Autoregulation Absent In Lungs & Skin.
Shock (mild to moderate hemorrhage) – large
Different Organs Have Variable Blood Flow Due To
Systemic Veins
Variable ResistanceQ
3. Exchange Vessels - Permeability – Parallel
Organ With Maximum Flow At Rest - Minimum
Arrangement (low pressure low vol) – Capillaries
ResistanceQ
4. Resistance Vessels- Distribution - (Arterioles –
Organ With Minimum Flow At Rest - Maximum
max smooth muscle)
ResistanceQ
5. Conduit Vessels – Transfer – Arteries
Type of Blood Pressure:
Reynolds Number
SBP – Due to ventricular systole
•• Reynold Number: Turbulence
Depends on SV (Directly) & Compliance (Inversely)
•• Formula – ρDV/η
DBP – Due to aortic recoil
ρ = Density
Depends on TPR
D = Diameter
PP – SBP-DBP
V = Velocity MAP = 2/3rd DBP + 1/3rd SBP
η = Viscosity = DBP + 1/3rd PP

•• Most Important Factor – Velocity Category SBP DBP


Normal BP ≤ 120 or/and ≤ 80
Inference :
High-normal BP 120-139 or/and 80-89
≥ 3000 = Turbulent flow
≤ 2000 = Laminar flow Grade 1 Hypertension 140-159 or/and 90-99
Grade 2 Hypertension ≥ 160 or/and ≥ 99
122
Cerebellum Quick Revision Notes

Summary of Baroreceptor Reflex –– ↑ HR (Indirectly via medulla & Directly via


SA node)
–– Tachycardia Reflex
2. Bezold-Jarisch Reflex - I.V. Irritant → Stimulate
Ventricular Chemoreceptor
–– ↓HR, ↓BP, Apnea
–– Bradycardia Reflex

Carotid Artery Ligation - INICET* 3. Cushing’s Reflex - ↑ ICT = Compression Hypoxia →


Stimulate VMC
1. Occlusion of CCA - Decrease blood flow to
Baroreceptors → BP & HR ↑ –– ↑ BP, ↓ HR, Irregular Breathing

2. Clamping of Carotid Artery - –– Bradycardia with Hypertension

–– Proximal to Carotid Sinus - Decrease blood Revision Summary:


flow to Baroreceptors → BP & HR ↑
•• Pacemaker Pot – FuNNy Cat Pec → Funny Na+
–– Distal Carotid Sinus - Overstretching of Channels, Ca2+ & K+ early closure
Baroreceptors → BP & HR ↓↓ •• Cardiac Cycle – 2 phases (Systole and Diastole)
Traube Hering Wave •• Ejection Fraction – SV/EDV

•• Physiological wavesQ •• Frank- Starling Law – Heterometric Regulation


(SV ∝ EDV)
•• Change - 2-4 mmHg
•• Resistance Vessels – Arteriole (Maximum
Inspiration - Venous Return Increases – ↓ BP Smooth muscles)
Expiration - Venous Return Decrease – ↑ BP •• Autoregulation Absent in Lungs and skin
•• Baroreceptor Most sensitive to Pulse pressure
>> MAP (maintain)

Respiratory System

Vasomotor/ Mayer’s Wave


–– Pathological waves
–– d/t Alternate Activation of Baro &
Chemoreceptors
–– Most commonly seen in Severe Hemorrhage.
–– Change – 10 – 40 mmHg

CVS Reflexes Weibel’s Classification:


1. Brainbridge Reflex - Rapid Infusion Increases VR (0 - 23 generations)
→ Stimulate Low Pressure Baroreceptors
123
Physiology

–– 0 - Trachea
–– 0 - 16 - Terminal Bronchiole
–– 0 - 16 - Conducting zone
–– 17 - Respiratory Bronchiole
–– 23 - Alveoli
–– 17-23 → Respiratory zone/Transitional zone

ITP - Why Negative?


–– Opposite Recoil of Lungs & Chest wall.
(Vacuum created in this closed system)

Trans-pulmonary Pressure
Difference in between
Mechanism of Respiration: Intra-alveolar and Intrathoracic pressure.
Boyle’s Law - P ∝ 1/V = IAP - ITP = +2.5 mm Hg
Pressure Volume
Trans-thoracic
Inspiration -ve Pressure/↓ ↑
Difference between
Expiration +ve Pressure/↑↑ ↓↓
Intrathoracic and Atmospheric pressure.
= ITP - Atm. P. = -2.5 mm Hg
Respiratory Pressures – IAP & ITP
Alveoli - Intra-alveolar/Intrapulmonary Pressure Trans-respiratory
(IAP/IPP) Difference between Intra-alveolar and
Inside Chest wall - Intra thoracic Pressure(ITP)/ Atmospheric Pressure
Intrapleural Pressure = IAP - Atmospheric P. = 0
IAP ITP
During Inspiration -1 mm Hg - 6 mm Hg
At Rest 0 mm Hg - 2.5 mm Hg
During Expiration + 1 mm Hg
124
Cerebellum Quick Revision Notes

Muscles used- Inspiration & Expiration Surface Tension (ST):


–– Inward force which is maximum at Air-
Water interface.

Compliance:
It is Distensibility / Stretchability.
Formula Compliance ΔV/ΔP
Compliance & Recoil
–– Depend on Elasticity
Emphysema (α-1 antitrypsin deficiency) High ST causes -
Poor recoil → Air Trapping (Expiration affected) –– Collapse of Alveoli → called Atelectasis.
But Compliance Increases –– Pull the fluid from Pulmonary Circulation,
Fibrosis (Parenchymal Disease) leading to Pulmonary Edema.

Compliance is Poor Surfactant:


•• Reduce Surface tension – Outward Force
•• Type II Pneumocytes >> Clara Cells
(Surfactant Proteins)
•• Composition:
70% – Lipids [Major → Phospholipids]
Maximum Phospholipid - Phosphatidyl
Phosphocholine (DPPC)/LecithinQ
Minor Phospholipid - Sphingomyelin
30% – Proteins & Mucopolysaccharides

Hysteresis Loop:
If we replace Air with saline, then no Air-water
Interference present.
Compliance curve:
No interference → No ST
125
Physiology

Hysteresis – Gap in Normal Compliance curve because Volumes and Capacities:


of ST.
ST absent if we replace Air with saline → No Gap
between Inspiration & Expiration curve.

Applied:
Factors ↓ Surface
Tension (ST)

Steroid Thyroid

Disease causing ↓
Surfactant:

→ Respiratory
Distress Syndrome
(Hyaline Membrane
Disease) CV :- closing volume a amount of air coming out of lung
E.g. Preterm Babies when the alveoli in dependant part of lung are begin
to close Q
Laplace Law: –– ITP :- at apex :- 8 mmHgQ
P = 2T/R –– ITP :- at base :- 2 mmHg
P - Distending Pressure CC (Closing Capacity): CV + RV
T - Wall Tension
Estimation of FRC:
R - Radius
•• Helium Dilution MethodQ
•• Nitrogen washout / Fowler’s MethodQ
•• Body plethysmography- BestQ

During Inspiration
Low Surface tension → long distance between
surfactant molecules
During Expiration Pressure Volume
INSPIRATION Lung ↓ ↑
High Surface tension → surfactant molecules comes
closer → preventing alveolar collapse Box ↑ ↓

If Radius ↓ - Wall tension to maintain the Distending ↑ ↓


EXPIRATION Lung
Pressure also ↓ .
(Vise-versa)
Box ↓ ↑
126
Cerebellum Quick Revision Notes

Restrictive Lung Disease:


Dynamic Air Volumes:
Right shift - Parenchymal Disease
No shift, but become small-Extra-parenchymal
Disease

Obstructive Lung Disease:


Left shift occur

Tiffeneau Pinelli index = FEV1%Q


FEV1/FVC × 100 = FEV1 % = 80-90 %

Obstructive Lung Disease - FEV1% DecreasedQ


Restrictive Lung Disease - FEV1% IncreasedQ

Flow – Volume Curve:


Upward Path - Effort dependent
Downslope path - Effort independent
Vital capacity = IRV + EC or IC + ERV
TLC = VC + RV
Maximum flow - PEFR (Peak Expiratory Flow Rate)
MEFR - Mid-Expiratory Flow Rate
Gold standard for Small Airway Disease - MEFRQ
Emphysema - Scooped out Pattern.
127
Physiology

Reflexes:Q
Reflexes Mechanism Significance
Head’s Paradoxical Reflex Positive feedback → Inflation stimulate Occurs in newborns to stimulate
further Inflation lung inflation.
Hering Breuer Inflation Reflex Inflation inhibits further inflation Prevent Alveolar Bursting
Hering Breuer Deflation Reflex Deflation inhibits further deflation Prevent Alveolar collapse

J Reflex – Juxtracapillary J-receptors stimulated → Apnea, Decrease - by A. S. Paintal


Reflex BP and Heart rate - Always pathological

Ventilation:
Pulmonary/ Minute VentilationQ Alveolar VentilationQ
Air coming in & going out of lung/min Air getting exchanged/min
MV = TV x RR AV = (TV – DS) x RRQ
= 500 x 12 = 6L /min = (500 – 150) x 12 = 350 x 12 = 4.2 L/min

Dead Space: –– Healthy individual – 0 mlQ


•• Physio DS = Anat DS + Alveolar DS
Physiological dead space (total)
•• Physio DS = 150ml = Anatomical dead space
Divided into 2 types:
•• Physiological DS > Anatomical DS (when Alveolar
1. Anatomical DS DS ↑)Q
–– Trachea to terminal bronchial.
Physiological DS Measurement - Bohr’s EquationQ
–– 150 ml
Anatomical DS Measurement – Fowler’s Method
2. Alveolar DS

Applied:

V/Q = 0/Q = 0 ↓V/Q = ↓ V/Q = 0.8 V/Q↓ = ↑ V/0 = ∞


Wasted Perfusion Q
Wasted VentilationQ

V/Q Ratio-Q
Apex - Maximum
Base - Minimum

V/Q Curve
128
Cerebellum Quick Revision Notes

Partial blockage - In between the curve Q INI-CET:


Gas Transport – CO2
Normal → 4 ml% or 200 ml/min Q
Forms →
B - Bicarbonate (HCO3-) → 70% Q
C - Carbamino form → 20%
D - Dissolved form → 10%

Gas Transport – O2
Increase in pH - Alkaline → Left shift
Normal → 5 mL % or 250 mL/min
Decrease in pH - Acid → Right shift
•• Arterial blood – 19 mL %
•• Venous – 14 mL % Regulation of Respiration:Q
Forms → A. Neural:
1. Hb bound → 97-98% Q

2. Dissolved → 1-3%

Oxygen-Hemoglobin Dissociation
Curve:Q

Site of Lesion Type of Respiration


Loss of voluntary control
Lesion below cortex
(Automatic respiration)
Right TAG – Increases
Bulbar Polio (Brain stem Ondine Curse (Only
T - Temperature damage) Voluntary control present)
A - Acidic → H+, CO2 & Lactic acid Mid pontine lesion
G - Glycolysis → 2,3 - BPG 1. With vagotomy Q Apneusis
Left shift – Blood bank and HbF (Fetal Hb) Increase depth of
2. Vagus intact Q
respiration (Slow & deep)
Between pons & medulla Irregular & shallow
Below medulla Respiration will stop
129
Physiology

B. Chemical Regulation –
Chemoreceptors:Q

Central Chemoreceptor stimulated by – H+ in CSF >> Hypoxia:


↑ pCO2

Glomus Cell - I:
–– Oxygen sensitive K+ channels.Q
–– Hypoxia → K+ Channels close – Depolarization
occurs → Opening of Ca2+ channels →
Increase Ca2+ concentration which leads to
Dopamine Q release.
–– Dopamine stimulates the 9th and 10th nerve.
Periodic Breathing:
A.
Cheyne Stokes BreathingQ → Hyperventilation
followed by Apnea
Gradual Increase and decrease.
E.g. High Altitude, Sleep Apnea, CHF
B. Biot’s BreathingQ → Irregular rate and Rhythm
E.g. Meningitis, Bulbar Polio, Uremia
C. Kussmaul’s BreathingQ / Air Hunger → Increased
Rate and Depth
E.g. DKA

Dysbarism/ Caisson’s / Decompression


Sickness:Q
Deep Sea – ↑ Atm. Pressure
Gases Compressed – Dissolved N2 (deposited in fat
tissues)
Hypoxia & Ventilation:
If f/b Rapid Ascend – Decompression of Gases N2
•• ↑PCO2 – ↑ Ventilation → Bubble Formation.
•• ↑PCO2 & ↓O2 – Steeper Curve –– Block Capillaries
•• ↑PCO2, ↓O2, ↓pH – Shift of Apneic Point –– Bones & Muscles – Bends
–– CNS – Strokes
–– Lungs – Chokes
130
Cerebellum Quick Revision Notes

Revision Summary: Synapse:


•• Terminal Bronchioles: Generation 16 Types -
•• IAP & ITP at Rest: 0 & -2.5 mmHg A. Chemical synapse:
•• High Compliance Seen in: Emphysema •• ≥ 99%
•• Surface Tension: Inward force •• Unidirectional
•• Surfactant: DPPC/Lecithin
B. Electrical synapse
•• FEV1% Increased in: Restrictive Lung Disease
•• Through the Gap junction (connexon)Q
•• Mid-pontine Lesion With B/L Vagotomy:
Apneusis •• BidirectionalQ
•• Central Chemoreceptor sensitive to: ↑H+ in the
CSF

Central Nervous System


CSF:
Daily Production – 500 ml
Normal Value – 150 ml
Normal Pressure – 70- 80 mm H2O
SeCretion – Choroid Plexus
Absorption – Arachnoid Villi Synaptic Transmission:
CSF vs Plasma: Have 2 essential components:- Neurotransmitters
exocytosis
CSF has
•• Low Concentration – Glucose, Proteins, Ca2+, K+ Require SNARE Proteins:

•• Equal Concentration – Osmolarity, HCO3- Q T – SNARE: Ex:- SNAP -25 , SyntaxinQ


•• High Concentration – Mg2+, Cl-, PCO2 V – SNARE: Ex:- SynaptobrevinQ, Synaptotagmin

Applied: –– If no contraction patient will suffer from


Flaccid paralysis
•• Botulinum toxinQ: - inhibit synaptobrevin –
Inhibit Ach (no contraction) –– Use in treatment of Achalasia cardiaQ,
Removal of wrinkle, Spasm, ContractureQ
131
Physiology

•• Tetanus toxin: - Inhibit Synaptobrevin & Rapidly adapting


Glycine releaseQ – Relaxation Inhibited 1. Meissner corpuscle
–– Spastic paralysis due to disinhibition of •• Most abundant, on Glabrous skin.Q
Muscle •• Fine touchQ, 2 point discrimination.
–– Ex: - Pre synaptic inhibition •• Best (Minimum distance)- 1-3 mm fingertips > lipsQ

Somatosensory System: 2. Pacinian corpuscle (onion shaped)


•• At Deep dermis, joint capsule
•• Sense high frequency vibration & PressureQ
3. Hair end organ
•• Sense light movement on skin

Slowly adapting
1. Merkel cells
•• Fine touch – 2 point discriminationQ, edges,
point (Merkel’s >> Meissner)
•• Blind - Braille readingQ
2. Ruffini ending
Tactile receptor: •• Mechano-sensitive receptors
Rapidly adapting Q
Slow adapting •• Stretch & sustained pressure
Detect - Rate of change Detect - Steady stimulus 3. Free nerve ending
Phasic receptors Tonic receptors •• Non-adapting & Sense Pain (Protective)Q
Ex:- Ex:- •• TRP (transient receptor potential channel)
•• Paci has – Pacinian •• Merkel cells Q involves
•• Messy – Meissner •• Ruffini cells •• Non capsulated present in Superficial dermis
•• Hair – hair end organ •• Free Nerve ending
4. Glabrous skin
•• Non hairy skin
•• Have Maximum Meissner >> MerkelQ

Pain:
132
Cerebellum Quick Revision Notes

Sensory/Ascending Tracts:
Pathway Mnemonic Functions

Anterior spinothalamic tracts Ant crawling Itch, Crude touchQ, Sexual sensation

Lateral spinothalamic tracts Laat maarna Pain & temperatureQ

Posterior column Rest all Fine touch, Pressure discrimination,Vibration, Stereognosis,


Graphesthesia & 2 points discriminationQ

Visual Pathway:
1. Dark = 11-cis RetinalQ + opsin → Depolarization
takes place via opening cGMP dependent Na+
Channels.
2. Light → 11-cis convert to All-trans retinalQ which
activate Transducin.
Transducin activates phosphodiesterase → Break
cGMP (Na+ Channels not open) → K+ Channels causes
Hyperpolarization.

Brown-Sequard Syndrome -
Hemisection of spinal cord
•• C/L - Pain & TemperatureQ
•• I/L - Rest all sensations

Penfield Homunculus:
•• Area devoted to particular part on the basis of Hearing:
usage of body parts
EAM → Tympanic Memb. → MIS → Oval Window →
•• Maximum: hands → thumb; Face → lipsQ Perilymph Vibration →
•• Minimum: trunk, hipQ Movement Of Cilia On Hair Cells → Opening Of Tip
•• Cortical plasticity: ability to new synapse with Links – K+ Entry.
neighbouring areaQ •• Depolarization
•• Cochlear Microphonic Potential (after
Summation) – by OHC
•• Endolymphatic Potential – via Stria Vascularis.
133
Physiology

Auditory Cortex – Area 41, 42


1. Superior Temporal gyrus → Understanding of
speech – Wernicke’s area (Area 22)Q
Defect – Sensory/fluent/Nonsense aphasiaQ
2. Inferior frontal cortex motor speech → Broca’s
area (Area 44)Q
Defect – Motor/Nonfluent aphasiaQ

Decerebrate Vs Decorticate
rigidity rigidity
Between TransductionQ Below cortex
superior &
inferior colliculus

Lost Rubrospinal tractQ Intact


Intact Reticulospinal Intact

Pyramidal Tract: Extended, ArmsQ Flexed,


adducted adducted
At Medulla - 80-85 % CST cross → form Lateral CST
Extended with Legs Extended with
•• Function - Skilled, fine, voluntary movement
plantar flexion plantar flexion
•• Supply - Distal muscles
Anterior CST → 15-20% uncrossed fibres UMN Lesion vs LMN Lesion:
•• Function - Tone, Posture & Balance UMN perform 2 functions -

•• Supply - Proximal and Axial muscles •• Stimulate Alpha Motor neuron

•• Inhibit Gamma motor neuron


Extrapyramidal Tract:
UMN Vs LMN
Medial Descending tract - 4 types
Increased Tone Low
1. Axial muscle – Tone, balance, postureQ
Absent/
2. Reticulospinal – Pontine, medulla Brisk/Increased DTRQ
Decreased
3. Tectospinal – Superior colliculus Absent Muscle atrophy Present
4. Vestibulospinal – innervates Alpha motor neuron
Not present (Babinski Fasciculation / Present (No
of extensor of LL
sign present) fibrillationQ Babinski sign)
Rubrospinal → Stimulate Alpha motor neuron of flexor
of ULQ
Stretch Reflex:
Monosynaptic / loading / DTR / myotatic / segmental
Mechanism: Passive stretch (Increase length) →
Muscle spindle (Length sensor) (via Ia & II) →
Stimulate Alpha MN → Contraction (decrease length)
134
Cerebellum Quick Revision Notes

Inverse stretch Reflex: (aka Lengthening for 5 years age)


reaction) 5. Night terrorQ ( fearful dreams → not
remembered)
Mechanism: Excessive stretch (increase tension) →
Golgi tendon organ (Tension sensor) (via Ib) → Inhibit REM-
Alpha MN → Relaxation of muscle (Decrease tension)
•• Hallmark - PGO spikeQ (Ponto-Geniculo-Occipital
Intrafusal Muscle Fibers: 2 types spike)

Nuclear bag fibers Nuclear chain •• Paradoxical sleep / active sleep- alpha & beta
fibers waves

Static & dynamic ResponseQ Static •• Sleep paralysis→ Muscle AntoniaQ except
respiratory muscle EOM , sexual arousal
Change in length / Sense Length, resting -
tissue & Velocity initial, final •• Memory formation will take place in this →
motion Nightmares
Hypothalamus
ECG WAVES: (Mnemonic - BAT Dance) 1. Circadian rhythm – Suprachiasmatic nucleus
B (14-30 Hz ) – RAS → + Cortex → alert & active, –– Biological (Internal clock)
awake (visual stimuli)Q
–– Rhythm → Endocrinal & GI function
A (8-13Hz) – Parietal (Occipital) Cortex, Thalamus →
–– Sleep wake cycle (> 24 hr)
–– Thalamo-cortical oscillation
–– External clock → Sun
–– Meditation (awake – eyes closed)
–– Pineal gland → MelatoninQ
–– Mind freely wondering Q
2. Control ANS – Paraventricular nucleus
T (4-7Hz) – Hippocampus → light sleepQ → Easy
arousal, degenerative disorders Sympathetic Parasympathetic
Fight & flight response Rest & digest
D (0-3Hz) - Thalamus→ Deep sleep,Q organic brain
disorder (difficult to wakeup) Lateral Hypothalamic Ant Hypothalamus
Nucleus Q Nucleus Q
NREM vs REM 3. Regulate sexual behaviourQ → Anterior
NREM – Hypothalamus, Preoptic nucleus
4. Control Thirst → ↑ osmolarity, Hypovolemia
•• Relaxed state → + Parasympathetic, ↓HR, BP,
→ SFO, OVLT (osmoreceptors) → lateral
RR (slow & deep)
hypothalamus & supra-optic nucleus (ADH)
•• Muscle tone → intactQ
5. Appetite regulationQ -
•• Stages
↑Appetite - MANGO ↓ Appetite
1. I → Alpha & theta waves , light sleep
M – MCH Αα-MSH (POMC)
2. II → theta wave & high frequency pattern
→ K Complex & Sleep SpindleQ A – AgRP (Agouti related Leptin
protein)
3. III & IV → deep sleep, delta wave Insulin
N – NPY
•• NREM – ParasomniasQ → altered events during Intestinal hormones
sleep G – Ghrelin
1. Sleep talking O – Orexin
2. Sleep walking (somnambulism)
•• Satiety centre- ventromedial nucleus of
3. Teeth grinding (bruxism) hypothalamus
4. Nocturnal enuresis (bed wetting → normal •• Hunger centre- lateral nucleus of hypothalamus
135
Physiology

6. Regulate Sleep & Wakefulness


–– Sleep → GABA, AchQ
–– Wakefulness → OrexinQ, Histamine,
Serotonin
7. Temperature regulation –

Ant nucleus/ Post Nucleus/
VM nucleusQ Dorsomedial Nucleus
↓ Temperature ↑ Temperature
Hot adaptationQ Cold AdaptationQ
Sweating, Vasodilation, Shivering, ↑ DTR
2. Operant – B. Skinner
↑ Thirst
–– Learning By Operating In The Environment
8. Regulation of endocrine secretion
–– Synthesis of hormones (Post Pituitary)
ƒƒ ADH → Supraoptic nucleusQ
ƒƒ Oxytocin → Paraventricular nucleusQ
–– Produce releasing & inhibitory hormone
ƒƒ GHRH & GHIH
9. Role in memory (Mammillary body) Synaptic Plasticity-
Papez circuit – MAC PE Hippo •• Post Tetanic Potentiation- Short Term/
Working Memory
Hippocampus Mammillary Body
Repeated stimulus → ↑Ca2+→ ↑Neurotransmitters
Entorhinal Cortex Ant. Nucleus (Thalamus)
•• Long Term Potentiation – CA1 neurons in
Int. Capsule Sommer’s area
Parahippocampal Gyrus Cingulate Gyrus Glutamate → NMDA Receptors → Kinases →
Cingulum Response
ƒƒ ↑ Neurotransmitter release
•• Mammillary body – ANS control
ƒƒ ↑ Protein that help build receptors
•• Anterior nucleus – relay centre
ƒƒ ↑ Pre synaptic & postsynaptic surface
•• Cingulate gyrus – cognition & emotional process area
•• Entorhinal cortex –spatial memory ƒƒ ↑ Synaptic knobs no.
Associative – Conditioning- Amygdala, Cerebellum ƒƒ Kinases → AMPA
1. Classical Conditioning •• Long Term Depression –
–– Ivan Pavlov GABA → Motor Memory → Cerebellum
–– Innate Reflex - UCS → UCR
Revision Summary:-
–– Pairing - CS + UCS → UCR
•• SNARE Protein Involved- Botulinum & Tetanus
–– Repeated – CS → CRQ – Synaptobrevin
•• Rapidly Adapting Receptor -Pacinian, Meissner’s
& Hair end organ
136
Cerebellum Quick Revision Notes

•• Brown Sequard Syndrome-C/L Loss of Pain & •• Medulla


Temperature.
–– Long loop Nephron – most of the nephron parts
•• Greater Distance In 2 Pt. Discrimination- on are present in the medulla – Juxtamedullary
the Back
Nephron Q
•• Decorticate Vs Decerebrate Posture – UL
–– Divided into Inner and Outer Medulla
Flexed Vs UL extended
•• Frequency of EEG Waves- BAT Dance –– Loop of Henle and Medullary CT are present
in the Medulla
•• Role of Hippocampus in Memory- Short Term
Memory → Long Term Memory Types Of Nephron
Cortical Medullary
RENAL SYSTEM
80-85% Contribution 15-20%
Shorter Loop of Henle Q
Longer
Peritubular Branches – Vasa
Shorter
Capillaries Recta
Maximum Blood FlowQ Less
O2 Consumption,
Maximum Less
PO2
O2 Extraction
Less Maximum
Ratio
Normal Concentration Of Concentrated
Urine UrineQ Urine

Glomerular Filtration Membrane

Introduction
•• Structural & Functional Unit – Nephron → 1
-1.2 million/kidney
•• Excretory system – excrete H2O soluble
substances
•• Nephrons ↓ with age → after 40 years; every 10
years we lose around 10% of nephrons •• 3 LayersQ
•• Renal Blood FlowQ 1. Endothelial cells (GC) → Glycocalyx, GAG
–– 20-25% of CO (negatively charged)

–– 1200-1250 ml/min 2. Basement Membrane – (maximum negatively


charged)Q
•• Cortex
○○ Heparan Sulphate
–– Short Nephron – Cortical NephronQ
○○ Proteoglycans , Sialoproteins
–– Glomerulus, PCT, DCT, Cortical CT present in
the cortex 3. Foot processes of podocytes
137
Physiology

○○ Epithelial cells – Bowman’s capsule •• Maximum Secretion of all substances occurs in PCT
Except K+ →Distal tubule, Urea → LOH
Criteria of a Substance for Filtration
•• PCT – NHE & NBC
•• Effective Pore size – 2nm – 4.5nmQ
–– Na+ H+ exchange → 50% (Max. Na+
–– <2nm = freely filtrable. E.g., Glucose,
transporter)Q
electrolytes, Urea
–– Na+ cotransporter → 45%
–– 2 – 4.5nm = Restricted filtration
–– SGLT → 5%
–– >4.5nm = Impermeable filtered. E.g., WBC,
Macroglobulin, Platelet –– Na+ k+ ATPase
Charge Dependant Restricted Filtration –– Na+ bicarbonate Co transporter
–– Passive Reabsorption Na+ & Cl- Q
•• SGLT

Cationic > Neutral > AnionicQ Loop of Henle – Counter Current


MechanismQ
Tubular transport •• Characteristics
•• PCT → cuboidal epithelium with brush bordersQ
1. 2 vessels
– Carbonic anhydrase
2. Parallel to each other
•• Brush border have Maximum surface area, Max
Reabsorption & secretion, Max blood supply, O2 3. Close to each other
supplyQ 4. Flow → opposite direction
5. Differential Permeability
•• Sites: - kidney, skin, GIT, testisQ
•• Aim: - water conservation→ concentrated urineQ
•• Hypertonic medullary InterstitiumQ

C.C. Multiplier

•• Maximum Reabsorption of all substances occurs


in PCT
Except Magnesium→ ascending loop of Henle
138
Cerebellum Quick Revision Notes

Descending LOH Ascending LOH & Medullary TAL – Bartter Syndrome & DCT
CD

•• At Apical side –
–– Na+ K+ 2Cl- transporter (NKC)Q
Descending LOH Ascending LOH & –– ROMK Channels Q – secrete K+
Medullary CD
•• On Basal side –
Drinks the water Na+, K+, 2Cl- & Urea
–– Na+ K+ ATPase channel
Passive diffusion of Active & passive transport
H2 O –– Cl- channels (ClC-Kb) – Barttin protein is
required to activate this channel
•• Creates the medullary gradient, makes medulla
4X hypertonic. •• Tight junctions
–– Made up of Paracellin-1 = help in the
C.C. Exchanger -Vasa Recta absorption of Mg2+ >> Ca2+ Q
•• Both limbs of Vasa recta → freely permeable –– Familial Hypomagnesemia – due to mutation
for both H2O & solute of Paracillin -1
•• Vasa recta maintains hypertonicity of medulla TAL – BARTTER SYNDROME - Types
1. Type I- NKCC
2. Type II- ROMK
3. Type III- CLC-Kb
4. Type IV- Barttin

Early DCT
139
Physiology

•• Drug that block NCC – Thiazide •• Amiloride & ANP – Inhibit the ENaC channel
•• TRP5/6 – Reabsorb Calcium •• Aldosterone – stimulate
•• Calcium is exchanged via NCXQ channel to –– ENaC
peritubular capillary
–– ROMK
DT – CD + DCT –– Na+ K+ ATPase
–– H+ K+ Exchanger
Applied -

ADH ↑↑ - SIADH ADH ↓↓/ Absent –


D.Insipidus

↑↑ Water permeable ↓↓ Permeability

↑↑ Water Reabsorption ↓↓ Reabsorption

↑↑ Urine Osmolarity ↓↓ Urine Osmolarity

↓↓ Plasma Osmolarity ↑↑ Plasma Osmolarity

Low volume, concentrated High volume, dilute


Hypertonic Urine – 1200 Hypotonic Urine = 50-
mosm/L 100 mosm/L, Polyuria,
Polydipsia

TmG & Renal threshold


•• Tmax for glucose – TmG → 375 mg / minQ
•• 2 cells
–– Maximum transport rate of a substance/
–– P cell - Principle cells unit time
–– I cell - Intercalated cells –– Tubular / filtered load = GFR × Plasma
I-cells conc. of substanceQ
•• α-intercalated cells – acid secretion
•• Renal threshold
•• β-intercalated cells – Base secretion
–– Plasma conc. of a substance above which
P-cells above which the kidneys begin to remove it
•• On P-cell- V2 receptors are present where ADH into the urine.
comes & acts on it → ↑cAMP → Aquaporin 2Q –– Plasma conc. above which Glycosuria startQ
channel on apical side →reabsorption of water
–– Expected RT → 375 mg /dl
•• Aquaporin 3/4 →present on basal side
–– Actual RT → 180-200 mg/dlQ
•• If ADH is not there – 80-85% water reabsorbed
Obligatory H2O Reabsorption –– This difference known as Renal splayQ due
to heterogenicity of nephron
•• Facultative H2O Reabsorption →ADH ⊕ → 10-
15% of H2O → DT –– All nephrons

ENaC ƒƒ Do not have same Tmg

•• Gain of mutation – Liddle’s Syndrome ƒƒ Are not active at Tmg all the time

•• Loss of mutation – Pseudo Hypo Aldosteronism


140
Cerebellum Quick Revision Notes

Renal splay –– RPF Estimation


•• Maximum filtration & Reabsorption
–– Ex :- Glucose & AA → complete Reabsorption,
clearance = 0
–– Na+ & H2O → clearance << filtrated (GFR)Q
•• PAH > Creatinine > Urea > Na+ & H2O > Glucose, AA

GFR – Autoregulation

Clearance
•• Amount of substance getting cleared from
plasma per minute
•• CL = UV/P = (filtration + secretion –
Reabsorption)/P Q

–– U = urine conc.
–– P = plasma conc.
–– V = urinary flow rate
•• Only filtration → No Reabsorption & no secretion
1. Myogenic Mechanism
–– CL = GFR
↑ BP → Stretch on Aff. Arteriole →↑ GFR
–– Ex :- Inulin → GFR EstimationQ

•• Filtration + secretion- (slight)
–– Opening of Ca2+ Ch.
–– Clearance > GFR (eGFR)

–– Ex:- Creatinine
–– Constriction of Aff. Arteriole
•• Filtration + complete secretion
–– PAH → Para amino hippuric acidQ
141
Physiology

2. T-G Feedback

JG Apparatus

•• Autoregulation → Myogenic MechanismQ


–– →BP→ stretch on afferent arteriole→↑ GFR

Stretch on smooth muscles of afferent (-)


arteriole

Ca++ channel Opening→ Constriction
of aff arteriole
–– GFR
ƒƒ Autoregulation

•• JG Cells – modified Smooth muscle cells & ƒƒ Resistance of afferent & efferent
afferent arteriole, Granular → ReninQ (via RAAS arterioles
system produces AG II → release aldosterone
Afferent arterioles RPF GFR FF
& cause vasoconstriction
Constriction ↓↓ ↓↓ N
•• Macula Densa- columnar epithelium ,
Dilation ↑↑ ↑↑ N
chemosensors (Na+ & Cl-)Q
•• Lacis cell aka Polkissen cells or extra glomerular
Efferent arteriole
Mesangial cellsQ . Anti-inflammatory .
ConstrictionQ ↓↓ ↑↑/↓↓ ↑/↓
T-G FeedbackQ
Dilation ↑↑ ↓↓ ↓
•• ↑ GFR → ↑ NaCl Filtration → Adenosine
FF = GFR /RPF
•• ↑ Adenosine → ↑calcium in smooth muscle
cells of afferent arterioles → constriction of GFR
arterioles → ↓ GFR
•• Calculations –
Afferent & Efferent Arteriolar
Constriction
142
Cerebellum Quick Revision Notes

NFP = HPG –πG – (HPB- πB) ENDOCRINE PHYSIOLOGY


GFR= Kf x NFP
Classification of receptor
–– NFP – depends on starling forces
•• Intracellular -
–– OP – colloid oncotic pressure OR Oncotic Pressure
1. Nuclear Receptor - Nuc TAD
–– NFP = (HPG - πG) – (HPB - πB)
Q
○○ T:- ThyroidQ
= (60-32) – (18- 0)
○○ A:- vit A
= 10 mmHg
○○ D:- vit D
–– GFR = Kf × NFP Q
2. Cytoplasmic Receptor
= 12.5 × 10 mmHg
○○ Mineralocorticoid – Aldosterone
= 125 ml/min
3. Both - glucocorticoids, sex steroids
–– FF = GFR / RPF (estrogen/ progesterone/ testosterone)
= 125/ 625 ×100 •• Extracellular
= 0.2% or 20% 1. GPCR – via 20 messenger

Acid Secretion & Acidification 2. Tyrosine kinase Ex:- InsulinQ, GF

•• Acid secretion in kidney – by all parts of kidney ○○ IGF, PDGF, NGF, FGF
except DTSQ 3. Cytokine → Janus Kinase → + JAK – STAT Q
•• Maximum in PCT (no acidification) – Max. ○○ GH & prolactin (Twin hormones),
buffers are availableQ LeptinQ
•• Acidification: CD > DCT 4. Serine kinase R Ex:- activin, AMH / MIS
•• Normal urine pH: 6-8
GPCR / 7 Pass / Transmembrane
•• Minimum urine pH: 4.5 → limiting pH of urineQ Receptor
Urinary Buffers •• Action- Through Secondary Messengers

•• Main urinary buffer: HCO3- Q •• cGMP cAMP IP3/ DAG

•• Only inducible buffer: NH3 Q


•• Only titratable buffer/main at CD: PO4-3 Gt Gi Gs ↑ Gq
↑Ca2+, NO →
Revision Summary: ANP, BNP, CNP
•• Max. Blood Flow in Kidney- PCT contractile
•• Type of Nephron –Conc. Of Urine - •• E.g. of IP3 DAG
JuxtraMedullary
–– OxytocinQ
•• Max. Transport of Substances Occur in PCT
–– Vasopressin (V1)Q
•• C.C Multiplier & Exchanger – LOH , MCD &
–– Ag II
Vasa Recta
–– Adrenergic
•• Filtration + Complete Secretion → CL = RPF
–– AchQ (M 1,3,5)
•• Renal Splay – Difference between Actual &
Expected TmG –– Exception → GnRH , GHRH, TRH

•• GFR= Kf x NFP ; NFP = HPG –πG – (HPB- πB) •• cAMP → relaxation, regulation, metabolism
•• Gs→ ACTH, TSH, LH, FSH, CRH, PTH, MSH,
ADH (V2), glucagon, CalcitoninQ
143
Physiology

•• Gi → Adrenergic, Ach (M2,4), somatostatin THYROID GLAND HORMONES


Oxytocin & Prolactin Types of Cells
•• Oxytocin – •• Follicular
–– Activated by Paraventricular nucleus –– Simple cuboidal cells
–– Causes milk letdown reflex/milk ejection –– Help in → T3, T4 Synthesis
reflexQ
•• Para-follicular Cells
–– Parturition reflexQ
–– Synthesis of Calcitonin → Ca2+ HomeostasisQ
–– In males- ejaculation of sperm
•• Prolactin
–– Milk production occur via prolactin
–– Prolactin inhibitory hormone (PIF)- DopamineQ
•• ADH

Growth hormone
•• Factors affecting GH
↑ GH ↓GH
Hypoglycemia Q GH & IGF-1
Protein Meal- AA REM
Psychological Cortisol,
Stress/Pyrogens Somatostatin
Hormones –estrogen/
androgen/ADH
Drugs-α1 adrenergic,
dopamine agonists
144
Cerebellum Quick Revision Notes

Actions
•• Skeletal Muscle - ↑ Protein CatabolismQ
•• Hyperthyroidism → muscle weakness thyrotoxic
myopathy
•• CVS – Upregulate Gene for SERCA, β-Adr.
Receptor, Na+K+ ATPaseQ
↑ HR, ↑ FOC, ↓TPR
•• CNS – Brain Development, Deficiency- Mental
retardation
Excess –hyperexcitability, irritability, anxiety
Synthesis of T3, T4 •• Bone – Formation, Maturation, Ossification
•• T – Trapping of I- •• Metabolism – Catabolic HormoneQ
–– NIS – present in Parotid gland, Thyroid •• ↑ Blood Glucose – ↑ Glucose Reabsorption,
gland, Mammary gland, PlacentaQ
Glycogenolysis, Gluconeogenesis
–– Pendrin – (I- Cl- exchanger)Q
Lipolysis, Proteolysis
Pendrin Protein
•• Calorigenic Effect – ↑ Heat ProductionQ
•• Present in Thyroid & Ear
•• ↑ O2 consumption, ↑ BMR
•• Mutation – Pendred SyndromeQ
•• Hypothyroid – cold intolerance, weight gain
–– Hypothyroidism or Goitre
•• Hyperthyroid - heat intolerance, weight loss
–– SNHL

•• O – Oxidation

•• B – Binding/ Conjugation/ Organification
–– Tyrosine + I2 → MIT
–– Tyrosine + I2 + I2 → DIT
–– ↑↑↑ I2 → ⊕ Organification/ Binding → Jod
Basedow EffectQ
ƒƒ I2 Induced Hyperthyroidism
–– ↑↑ I2 → Θ Organification → Wolff Chaikoff Insulin Structure
EffectQ
•• C – Coupling
–– MIT & DIT (bind on 3,5 positions of tyrosine)
–– MIT + DIT → T3
–– DIT + DIT → T4
–– DIT + MIT → RT3
•• E – Endocytosis
•• R – Release
145
Physiology

•• Islets of LangerhansQ – Max. at tail → Glucose •• Amylin → promote the insulin


metabolism DM → T/T Amylin analogue – PramlinitideQ
•• ↓ Blood glucose → + Alpha cells → glucagon C peptide → Connecting peptideQ
•• Glycogenolysis, gluconeogenesis, lipolysis, •• Release → equimolar concentrations of insulin
proteolysis →↑ blood glucose → + β cells → •• Therefore to assess the endogenous insulin
insulin → ↓ blood glucose release C-peptide assay is done.

Insulin secretion from β cell-

Factors Affecting Glucose


Increasing Decreasing
ATP – Glycolysis, glycogen, FPA, AA Insulin, Hypoglycemia, hypokalemia
Parasympathetic activation (Ach, M3) Sympathetic activation (alpha >> beta)
K channel blockers (sulphonylureas)
+
K + channel openers Q (Diazoxide t/t insulinoma)
GI hormones (secretin, CCK, gastrin ) Best ↑ by: - GIP, GLP-1Q 2 deoxy- glucose

Adrenal Gland

Applied Cushing syndrome -


•• Stick on lemon appearance
•• BEL ↓
–– B- Basophils
–– E- Eosinophils
–– L- Lymphocytes
•• Neutrophils & Monocytes ↑↑
146
Cerebellum Quick Revision Notes

Zona Reticularis •• PTH Action on S. Ca2+ ↑↑ S. PO43- ↓↓


DHEA & Androstenedione
GASTROINTESTINAL SYSTEM
Introduction
Testosterone Estrogen (granulosa cell-
Leydig Cell FSH, theca cell-LH) •• Length of Digestive Tract – 8 to 10 meters
•• Organisation
–– Serosa
Sex steroids Androstenedione, Adipocytes
(Aromatase) –– Muscularis

20 sexual characteristics ƒƒ Longitudinal


ƒƒ Circularis - Myenteric plexus aka
Vitamin D Auerbach’s plexusQ (b/w longitudinal and
circularis layer)
–– Submucosa
ƒƒ Rich in Blood supply as well as Glands
ƒƒ Submucosal plexus→ aka Meissner’s
plexusQ
–– Mucosa

Ca2+ HomeostasisQ
Hormones S. Ca+2 S. PO43- Action
PTH ↑↑ ↓↓ Bone Resorption
Calcitonin ↓↓ ↓↓ Prevent bone
Resorption
Vitamin D ↑↑ ↑↑ Bone ossification ENS – brain of GIT
INTRINSIC
Revision Summary:
Submucosal plexus Myenteric plexus
•• Insulin Hormone Acts Via Tyrosine Kinase
Aka – Meissner’s plexus Aka – Auerbach’s plexus/
•• 20 Messenger of ADH from V2 Receptor- cAMP
Myenteric Q
•• Effect of Fasting/Starvation on GH – increases
Function - Secretion Q Function - Motility
GH
EXTRINSIC
•• Wolff-Chaikoff Effect-↑↑ I2→ Θ
Organification Sympathetic Parasympathetic
•• C-Peptide Assay Used to - assess the Inhibit both the Intrinsic Causes activation of
endogenous insulin release C system both the plexuses
•• VMA Levels in urine is done for – Dx of
Pheochromocytoma
Basal electrical rhythm
•• Produce Electrical Activity – Basal Electrical
147
Physiology

Rhythm (BER) Q
–– Cyclic, fluctuation (rhythmic) in RMP
–– Unstable RMP (-40 to -60mV)Q

BER → Spike Potential


Peristalsis vs Segmentation -
•• BER Alone – No Contraction (RMP)
•• BER + Stimulus → Ca2+ influx → Contraction due
to spike potentialQ

Factors Affecting Motility/Smooth Muscle


Contraction
•• Stimulators - Ca2+, Ba2+
•• Distention/ Stretching,Q Cold Temperature
•• Parasympathetic Nerves – Ach → M3 → IP3/
DAG → Spike potential & Contraction
•• Serotonin, Nicotine, Histamine, Motilin
–– Motilin → Mo cells - ↑ motilityQ
Mass Movement
–– Prokinetic drugs like Erythromycin with
stimulate Mo cellsQ and cause diarrhea •• Massive Propulsive Wave of Contraction

•• Inhibitors – Sympathetic Nerves – Epinephrine, •• 1-3 Times/ Day


Nor-epinephrineQ •• Strong In Infants

Peristalsis – Esophagus •• Can Be Initiated By Gastro-Colic ReflexQ

Pacemaker – Interstitial cells of CajalQ Migrating Motor Complex


•• Waves of Contraction Produced By An Empty
StomachQ
•• Aka known as ‘Hunger pangs’
•• Present in the interdigestive phase
•• Start from stomach & go till distal ileum
•• 1MMC → 90-110 mins, Speed – 5cm/minQ
•• Helps is clearing the GIT form the remnant
particles
Gastric Motility – Proximal •• Aka Broomstick of GITQ
•• Receptive, adaptive & feedback relaxation – •• Prepare GI for its next meal
Reservoir
148
Cerebellum Quick Revision Notes

•• Stimulated by hormone motilin (Mo cells)-


prokinetic drugs like Erythromycin also
stimulate Mo cell.
Applied
•• Paralytic Ileus → prolonged handling of Gut/
Trauma/ continuous irritation to peritoneum
(can be due to surgery) → Adynamic ileusQ (Θ
Gut movement) → ↓ intestinal motility
Facts-
•• pH •• Post Prandial Alkaline Tide

•• Stomach → 1.2 – 3 –– After eating the food → ↑ HCO3- in blood →


↑pH blood after food ingestion
•• Saliva → 6 – 7
•• Intestine → 7.5 – 8
Pancreatic Secretions
•• Zymogens – Inactive forms = to prevent
•• Pancreas → 8 – 8.3
autodigestion of pancreas
•• Brunner’s gland → 8.3 – 9 (most alkaline
•• CCK-PZ – Rich in pancreatic enzymes
secretion)
•• Secretin – rich in HCO3- to neutralize the acidic
•• Max HCO3-→ Pancreas (pancreatic secretions)
Q pH

•• Max K+ Secretion → SalivaQ


–– Concentration → ColonQ
•• Max Na+ (Least K+) → Bile
•• Least Na+ → Saliva
•• Max Cl- → Stomach
Gastric Secretions - Phases
•• Cephalic (20-30%) – HCl secretion
–– Thought, smell, sight of food → ⊕
Parasympathetic → ↑ Salivary & Gastric
secretion
•• Gastric (60-70%) –
–– Food enters the stomach → ⊕ Vagus Nerve
→ + Distention
•• Choleretics - ↑ Bile acid secRETion (Secretin,
•• Intestinal (10%)Q Bile salts & Vagus N.)
–– Food/chyme enters the intestine → ↓ Acid •• Cholagogue – Gall bladder Contraction (CCK)Q
& ↑ HCO3- secretion with the help of GI
hormones Iron absorption -
ƒƒ Secretin, GIP, GLP 1&2, CCK-PZ, VIP
Applied
Post Prandial Alkaline Tide -
149
Physiology

•• Iron in diet – Ferric – Fe3+ Q •• In stomach, B12 is protected by Haptocorrin


(HC) Q
•• Storage form in the enterocyte – Ferritin – Fe3+
•• Intrinsic Factor is released in the stomach by
•• Ferroportin – transfer Fe2+ from enterocyte to
the Parietal Cells
the blood
•• Intrinsic Factor (IF) – protects Vitamin B12
•• Hephaestin inQ on Ferroportin 1 - help in the
from destruction into the intestineQ
conversion of Fe2+ to Fe3+
•• Antibodies of Parietal cells cause – Pernicious
•• Negative regulator of Ferroportin – HepcidinQ
AnemiaQ
•• Transferrin - help in the transportation of the
•• Resection of terminal ileum – no absorption of
ferric form
vitamin B12 - lead to Pernicious AnemiaQ
Vit B12 absorption –
Revision Summary:
•• Functions of ENS Plexus – Secretion & Motility.
•• Pacemaker of GIT- interstitial cells of Cajal.
•• Contractions By An Empty Stomach - MMC
•• Max. pH is of – Brunner Gland >> Pancrease
•• Activation of Trypsinogen is Done by
Enterokinase
•• Role of Hepcidin in Iron Regulation - Negative
regulator of Ferroportin
•• Vit B12 is Protected in Stomach by Haptocorrin
BIOCHEMISTRY
QUICK REVISION NOTES

CONCEPTS to give 3 fatty acids and glycerol.


•• Fatty acid and Glycerol are smaller molecules,
Basics so they enter adipose tissues.
•• Fed: Within 2 hours of food intake •• In adipose tissues, they rejoin to form TGs
•• Fasting: 12-18 hours period (lack of food) again, which are stored here (anabolic role).
•• Starvation (prolonged fasting): 1-3 days (lack of Hormone Sensitive Lipase (HSL)
food)
•• This enzyme is present inside the cells of
Smile Formula-1 adipose tissues
•• Stored triglycerides in adipose tissues are
Anabolic Pathways List broken by HSL into 3 fatty acids and glycerol
•• Glycogenesis (synthesis of glycogen)
•• 3 fatty acid go in blood and then taken by liver
•• HMP (synthesis of Ribose-5-P & NADPH) where β- Oxidation of fatty acids (catabolic)
occurs to give energy in Liver.
•• Fatty Acid Synthesis
•• Triglyceride Synthesis Synthesis of fat (lipogenesis)

•• Cholesterol Synthesis
Smile Formula 2
•• Insulin acts in Fed state, so is always an anabolic
•• Enzyme: Lipoprotein LipaseQ
hormone
Catabolic Pathways List •• Glucagon acts in fasting/starvation state, so is
•• Glycolysis (breakdown of glucose to pyruvate) a catabolic hormone

•• Link reaction by PDH complex [Pyruvate (3C) → •• Insulin activates all anabolic pathwaysQ
Acetyl CoA (2C)] –– Exception: Insulin also activates 2 catabolic
•• Glycogenolysis (breakdown of glycogen) pathways:

•• β- Oxidation of fatty acids (breakdown of fatty 1. Glycolysis


acid) 2. Link Reaction
•• Gluconeogenesis (occurs in fasting/starvation •• Glucagon activates all catabolic pathways
state as something is broken down for making
–– But it will not activate two catabolic pathways:
glucose)
glycolysis and link reaction
•• Ketone body synthesis (fats broken down to
form KB) Phosphorylation and Dephosphorylation
•• Ketone body utilization
•• Enzyme: Hormone sensitive lipase (HSL)Q

Lipoprotein Lipase (LPL)


This enzyme is present in endothelium of blood vessels
•• Lipoprotein lipase enzyme breaks triglycerides
(TGs) of Lipoproteins (Chylomicrons and VLDL)
153
Biochemistry

•• Insulin always ↓cAMP and causes Sources Of Blood Glucose


DephosphorylationQ
1.Food
–– Insulin activate phosphodiesterase
2. Liver Glycogen (provides blood glucose for 12-18 hours)
–– Lack of cAMP causes dephosphorylation
3. Gluconeogenesis
•• Glucagon always ↑cAMP and causes
Phosphorylation Note: Never answer fats for above question

–– Glucagon activate adenyl cyclase thereby Preferred/Main fuel for body


synthesizing cAMP from ATP
•• 1st is carbohydrates
–– cAMP causes phosphorylation
•• 2nd is fats
Smile Formula 3 •• 3rd is amino acids/proteins
•• The pathways/enzyme which are activated by Important Information
insulin (All anabolic pathways + Glycolysis & Link
reaction) are always active in dephosphorylated •• Fats can never by converted to carbohydrates
state as link reaction is irreversible

•• The pathways/ enzyme which are activated by Exception: Two breakdown products of fats can be
glucagon are always active in phosphorylated converted to glucose:Q
state 1. Glycerol → Break down product of TG

Smile Formula 4 2. Propionic acid → obtained from Break down


product of Odd chain F.A
Which pathway occurs in which compartment of the
cell?
•• All catabolic pathways occur in Mitochondria
•• All anabolic pathways occur in cytoplasm
–– Exception: Glycolysis and Glycogenolysis
(Catabolic pathways BUT occurs in Cytoplasm)
Three pathways occur in both Mitochondria &
CytoplasmQ
1. Urea cycle
2. Heme synthesis
3. Gluconeogenesis
Note: Start (first enzymatic reaction) of these 3
pathways occurs in mitochondria
CYTOPLASM MITOCHONDRIA
HMP Beta oxidation of fatty acids,
Link reaction
Glycogenesis Apoptosis
FA synthesis Ketone body synthesis &
Ketone body utilization
Cholesterol synthesis Vital pathways -TCA & ETC
Exceptions – Glycolysis, Replication, Transcription,
Glycogenolysis Translation (for mitochondrial
DNA)
154
Cerebellum Quick Revision Notes

Scene in fasting state 2. Acetyl CoA not available as it is diverted to KB synthesis,


3. NAD not available as it is used in beta oxidation.

FUELS FOR BODY IN VARIOUS STATES


Fed Fasting Starvation
Glucose
Brain Glucose (as F.A can’t Ketone bodies
cross B.B.B)
Heart Fatty acids Fatty acids Ketone bodies
Amino acid/
Liver Glucose Fatty acids
protein
Muscle Glucose Fatty acids FA, KB and AA
Adipose
Glucose Fatty acids Fatty acids
Tissue
Glucose
Glucose
Important Information (As no
As no
•• Acetyl CoA only activates first step of RBC Glucose mitochondria
mitochondria,
gluconeogenesis but can never be converted to For F.A
Can’t use K.B]
glucose. It is never glucogenic.Q utilization)

Diabetes Important Information


•• Decreased or absent Insulin •• Heart uses fatty acid in fed state.
•• Glucose is not able to go to peripheral tissues –– Fetal Heart: Fuel is glucose
& cells
–– Heart Failure: Fuel is glucose
•• So, it is same state as fasting/starvation
•• RBC always uses glucose [Any state]
•• So catabolic pathways will be activated, and
anabolic pathways will be deactivated. Basics Of Diet
Catabolic pathways ↑ Anabolic pathways ↓ •• Diet
HSL –– 60-70% carbohydrates
Lipolysis
–– 15-20% fats
β- oxidation of FA Lipoprotein lipase
–– Rest is proteins
Gluconeogenesis
KB Synthesis •• Only 50% of carbohydrates consumed, are used
to give energy, Rest 50% is stored:
Important Information
→ 10 % as glycogen
•• Only one anabolic thing happens in liver of
chronic diabetic patients → 40% is converted into fats known as
endogenous fats as formed inside body. They
–– There will be excess of acetyl CoA in liver
get transported as VLDL lipoprotein
(due to increased β- oxidation)
•• 15-20 % fat taken in diet is exogenous fat
→ This excess Acetyl CoA is forced to be
and is transported in the form of Chylomicron
used in Endogenous fatty acid synthesis lipoprotein.
and triglyceride synthesis (↑VLDL in lipid
Q. A person on fat free, carbohydrate rich diet
profile) and also Cholesterol synthesis
continuous to grow obese. Which lipoprotein is
TCA suppression occurs in DM because: increased?
1. Oxaloacetate not available as it is diverted to Ans. Excess carbohydrates get converted to fat
gluconeogenesis, inside body k/a endogenous fat causing weight gain.
155
Biochemistry

Endogenous fat/TGs are transported in the form of


VLDL lipoproteins in blood.
Important Information
•• Atkin’s Diet – is for weight reduction- Low calorie,
Low carbohydrate diet
•• Fructose is the most lipogenic sugar

Thermogenic/Thermic effect of food/


SDA (specific dynamic action)
SDA: Amount of energy required to digest, absorb, Important Information
transport & metabolize food in body.
•• NADPH Involved in Reductive biosynthesis- in
•• It is maximum for Proteins > Carbohydrates > anabolic pathways
Fats.Q
•• NADH & FADH2 in Catabolic pathways
Enzyme Classification NADPH
•• 6 categories / Enzyme commission numbers /
Synthesized fromQ
Enzyme E.C number / Enzyme code numbers
1. HMP (Main Source)
•• Mnemonic: OTHLIL
2. Malic enzyme (Minor source)
Oxidoreductases Oxidation/Reduction reactions
Transfer of group from one molecule 3.Cytoplasm Isocitrate dehydrogenase (Minor Source)
Transferases
to other (Molecular formula changed) Note: All 3 enzymes lie in cytoplasm
Hydrolases Breaks bond using H2O
Enzyme Function EC No.
Make/Break a bond without H2O &
Lyases Dehydrogenase Does oxidation 1
ATP
Reductase Does reduction 1
Interconverts one isomer to other
Isomerases Transfer organic Phosphate
(Molecular formula remains same) Kinases 2
from ADP or ATP
Ligases Uses ATP to make the bond
Phosphorylase Transfer inorganic /free (Pi): 2
Phosphatase Removes PO4 using H2O 3

Important Information
Substrate Level Oxidative Phosphorylation/
•• Additional of O2 → Oxidation
Phosphorylation ETC
•• Addition of H2 or e- → Reduction In Glycolysis & TCA by Many oxidoreductases
•• H2/H atom/reducing equivalent = e- enzyme Kinase enzymes & ATP synthase

•• H+ ion → Proton ATP formed in only one Lot of steps


step
Gives only few ATPs in Gives most ATPs in the cells
Dehydrogenase
the cells
Causes removal of H2 i.e., some compounds are getting
oxidized. Synthesis
•• Synthase
–– ATP not used
–– E.C No = 4 (lyase)
•• Synthetase
–– ATP used
–– E.C No = 6 (ligase)
156
Cerebellum Quick Revision Notes

All Synthase Are Lyases Except: Cell Organelle Marker

Enzyme EC No. •• Plasma Membrane •• Na+ K+ ATPase, 5’


Nucleotidase, Adenyl Cyclase
Nitric oxide synthase 1
•• Golgi Apparatus •• Galactosyl Transferase
Citrate synthase 2
•• Cytosol •• Lactate DH
Glycogen synthase 2
•• Ribosome •• rRNA
ATP synthase 3 •• ER •• Glucose-6-Phosphatase
•• Peroxisomes •• Catalase
Carboxylase •• Nucleus •• DNA, DNA Polymerase

•• Addition of CO2 to a molecule •• Mitochondria •• Succinate DH, Glutamate DH

•• Requires ABC (Mnemonic) Q


•• Lysosome •• Acid Phosphatase
–– A - ATP
–– B - Biotin / Vit B7 PREVIOUS YEAR QUESTIONS
–– C - CO2 Q. Cause of formation of ketone bodies in Diabetic
Ketoacidosis are all EXCEPT: (JIPMER Dec 2019)
•• Also ‘Mg’ is required for all the carboxylases
A. Acetyl CoA not participating in TCA Cycle
•• E.C. Number = 6 (Ligase category)
B. Increased activity of beta hydroxy butyrate
Decarboxylation (removal of CO2) dehydrogenase
C. Mobilization of fats from adipose tissue
1. Oxidative
D. Decreased fatty acid Oxidation
•• Requires Vit B1 as co-enzyme
Exp: There is increased beta oxidation of Fatty acids
•• E.C No = 1 due to higher fat mobilization from adipose tissues.
•• Enzyme used: Dehydrogenase i.e., Removal of
Q. K e t o n e b o d i e s a r e n o t u t i l i z e d b y ?
CO2 along with oxidation.
(JIPMER Dec 2019)
2. Simple A. Brain
•• Requires Vit B6 as co-enzyme B. RBC
C. Heart
•• E.C No = 4
D. Skeletal muscle
•• Here CO2 removed without use of H2O (Lyase
category) Exp: RBCs lack mitochondria and, therefore, cannot
utilize ketone bodies as a source of energy

Q. In case of Lipoprotein Lipase (LPL) deficiency,


which of the following will increase after a fat rich
diet? (NEET Jan 2019)
ExamplesQ A. Chylomicron

1. Histidine → Histamine B. HDL


C. Lipoprotein (a)
2. Tryptophan → Tryptamine
D. LDL
3. Tyrosine → Tyramine
Exp: LPL deficiency impairs the hydrolysis of triglycerides
4. DOPA → Dopamine in chylomicrons and VLDL, leading to elevated levels of
5. Serine → ethanolamine chylomicrons and VLDL, after a fat-rich diet.

6. Cysteine → β-mercaptoethanolamine Q. Enzyme activated by decrease in Insulin: Glucagon


ratio? (NEET Jan 2020)
7. Glutamate → GABA
8. Lysine → Cadaverine (a diamine)
157
Biochemistry

A. Glucokinase Q. Highest thermic effect is of which food? (INICET


B. Hexokinase May 2023)
C. PFK A. Carbohydrate
D. Glucose 6 phosphatase B. Fat
Exp: Glucose-6-phosphatase is an enzyme of C. Protein
gluconeogenesis, so a catabolic enzyme which is D. Mixed
activated by low insulin concentration and high
glucagon. Exp: Protein has the highest thermic effect of
food, meaning it requires more energy for digestion,
Q. RBC uses which of the following substances during absorption, and utilization compared to carbohydrates
fasting / starvation? (INICET July 2021) and fats.
A. Glucose
B. Ketones RATE LIMITING ENZYMES AND
C. Amino acids REGULATION
D. Fatty Acid
Glycolysis
Exp: RBCs rely predominantly on glucose as their
energy source, even during fasting or starvation, •• Three Regulatory /Irreversible Enzymes
as they lack mitochondria and cannot utilize other 1. Hexokinase /Glucokinase
substrates like ketones, amino acids, or fatty acids.
2. Phosphofructokinase-1 (PFK-1)
Q. Insulin inhibits which of the following lipase
3. Pyruvate Kinase
enzymes? (INICET July 2021)
A. Hormone sensitive lipase Gluconeogenesis
B. Lipoprotein lipase •• 3 Rate Limiting Enzymes
C. Acid lipase
1. Pyruvate carboxylase
D. Alkaline lipase
2. PEPCK (Phosphoenol Pyruvate carboxykinase)
Exp: Insulin is an anabolic hormone and inhibits
catabolic enzymes. HSL is a catabolic enzyme. 3. Fructose 1-6 Bisphosphatase (Most Important)

Q. In low insulin: glucagon ratio. What is correct? TCA Cycle / Kreb’s Cycle
(NEET PG 2023) •• 3 Rate Limiting Enzymes
A. Increase activity of HSL 1. Citrate synthase
B. Increase activity LPL
2. α-Ketoglutarate dehydrogenase
C. Increase glycogenesis
3. Isocitrate dehydrogenase: Most important
D. Increase glycolysis
ƒƒ Stimulated By ADP
Exp: HSL is a catabolic enzyme so activity increases
under low insulin (anabolic hormone) and high glucagon ƒƒ Inhibited by ATP and NADH
(catabolic hormone) concentration.

Q. Which of the following is found only in cytoplasm:


(FMGE Jan 2023)
A. TCA cycle
B. Beta-oxidation of Fatty acids
C. Glycolysis
D. Gluconeogenesis
Exp: All anabolic pathways occur in cytoplasm. Exception:
Glycolysis and Glycogenolysis (Catabolic pathways
occurring in Cytoplasm)
158
Cerebellum Quick Revision Notes

Pathway Rate Limiting Enz Activators Inhibitors


Glycolysis PFK-1 AMP, fructose 2,6 bisphosphateQ ATP, Citrate
Glycogenesis Glycogen synthase Insulin, glucose 6-P Glucagon, Epinephrine
glycogen cAMP, kinase, S’AMP, Glucagon, Ca-calmodulin, Insulin, Phosphatase, ATP,
Glycogenolysis
phosphorylase Epinephrine, Nor – EpinephrineQ Glucose, Glucose-6-P, Fructose-1 P

Pathway Rate limiting Enz Activators Inhibitors


HMP G6PD NADP NADPH
PRPP Glutamyl
De novo purine synthesis - AMP, GMP, IMP
Amidotransferase
CPS-II (Euk) (carbamoyl
Pyrimidine synthesis ATP UTP
phosphate synthetase )Q
Urea cycle CPS - IQ NAG (N-Acetyl Glutanate) -
Purine Catabolism Xanthine oxidase - AllopurinolQ

Pathway Rate limiting Enz Activators Inhibitors


ketone body synthesis HMG Co-A synthase
Cholesterol body synthesis HMG CoA reductase Insulin, thyroxine Glucagon, cholesterol,
mevalonate, Bile acids, statins
Fatty acid synthesis Acetyl CoA carboxylase or Insulin, citrate Glucagon, Palnitoyl CoA / acyl
malonyl CoA synthetase CoA

Pathway Rate limiting Enz Activators Inhibitors


ß-oxidation of FA CPT – I or CAT – I (carnitine palmitoyl / acyl transferase) - Malonyl CoAQ

Bile acid synthesis 7-a-Hydroxylase - -


Catecholamines synthesis Tyrosine hydroxylase (requires Tetra Hydro Biopterin (THB), - -
NADPH)
Vitamin D synthesis 1-a-hydroxylase (in kidneys)

Pathway Rate limiting Enz Activators Inhibitors


Haem synthesis ALA Synthase - -
Niacin / Vit B3 synthesis QPR Tase (Quinolate Phospho Ribosyl Transferase) - -
159
Biochemistry

Reciprocal Regulation:
2 opposite pathways never occur together:
1. Glycolysis & Gluconeogenesis: Fructose 2,6 Bisphosphate is the reciprocal regulator, which inhibits
gluconeogenesis and activates glycolysis, as it is formed in fed state.
2. Beta oxidation & Fatty acid synthesis: Malonyl CoA is formed in fed state in Fatty acid synthesis and it
inhibits CPT-1 (RLE of beta oxidation of Fatty acids)
G6PD deficiency Pyruvate kinase deficiency
Hemolysis Hemolysis
1st most common human enzyme deficiency 2nd most common human enzyme deficiency
Heinz bodies present Heinz bodies absent
•• In prokaryotes, RLE of Pyrimidine synthesis is: ATC (Aspartate Trans Carbamoylase)

ENERGETICS Total ATP generated during anaerobic conditions is =


In this chapter, energetics of following explained: 4ATP - 2 ATP = 2 ATP (No ATP’s from NADH produced
Glycolysis, RL Shunt, TCA cycle, Warburg effect, ETC, as seen in aerobic glycolysis)
NADH Shuttles, Urea cycle, Galactose & Fructose
Important Information
metabolism, Gluconeogenesis, Beta oxidation of Fatty
Acids & Ketone body breakdown. •• Purpose of extra step of lactate dehydrogenase
is to regenerate NAD which again can be used
Energetics Of Glycolysis in glycolysis to keep the glycolysis going.
Under aerobic conditions Q. In anaerobic glycolysis, End Product is?

A. 2 ATP + 2 NAD
B. 2 ATP
C. 2 ATP + 2 NAPH
D. ATP + 2 FADH2

Q. In Anaerobic glycolysis, there is gain of?

A. 2 ATP + 2 NAD
•• 2 ATP used: Hexokinase, Phosphofructokinase B. 2 ATP
•• 2 SLP give 4 ATPs as phase II of glycolysis C. 2ATP + 2 NADH
starts with 2 molecules of glyceraldehyde-3-
phosphate. D. 4 ATP + 2 FAPH2

•• Total ATPs generated = 7 ATPs (aerobic 2 ATP as net gain of NAD is ‘Zero’.
conditions).
Q. No. of ATP produced in RBCs in fed state, fasting
state, aerobic and anaerobic condition.
Under Anaerobic Condition
Ans. As there are no mitochondria in RBC, so, there
is no ETC. So, no ATP will be produced from NADH via
ETC. Hence, total ATP produced in RBC in fed, fasting,
aerobic or anaerobic state is always 4-2 = 2 ATP

Rapoport-Leubering Shunt / Cycle


•• Occurs only in RBCs
160
Cerebellum Quick Revision Notes

Energetics of TCA Cycle

•• In RL Shunt, one substrate level phosphorylation


(SLP) step doesn’t occur (see fig).
•• Thus 2, 3-BisPG is produced at the expense of
2ATP. •• Dehydrogenases of TCA produce 3 NADH & 1
FADH2
•• Thus, ATP formed in RBC via RL shunt is 2 ATP
- 2 ATP = Zero ATPQ •• There is 1 SLP (Substrate level phosphorylation)
by succinate thiokinase
Important Information
•• So, from 1 acetyl CoA we get:
•• The purpose of RL shunt is not to produce
ATP but to form 2,3-BisPG –– 3 NADH → ET C → 7.5 ATP

•• 2,3-BisPG helps in release of O2 from HbA –– 1 FADH2 → ETC → 1.5 ATP


–– 1ATP/GTP → SLP → 1 ATP
Full Story of RBC
–– Total = 10 ATP per Acetyl CoA.

Energetics of complete breakdown of


Glucose

Important Information Pasteur’s Effect


•• If no RL Shunt occurs in RBCs, we get → 2 ATP •• Occurs in any normal cell
•• In phase II, •• In the presence of O2, Anaerobic glycolysis is
–– If one triose enters RL shunt we get → 1 ATP inhibited.

–– If two triose enters RL shunt we get → 0 ATP Warburg’s Effect


•• Occurs in cancer cells
•• In this, even in the presence of O2, glucose is
converted to lactate
•• Hence it is known as “Aerobic glycolysis”Q
161
Biochemistry

•• Lactate is the dead end of Glycolysis, so, no Important Information


oxidative phosphorylation further
•• Glycerol phosphate shuttle is more important
•• In conclusion, Warburg’s Effect is Aerobic
in brain and skeletal muscle as it provide quick
glycolysis with no oxidative phosphorylation
supply of energy.
•• But Here end-product is lactate and ATP
Q. Shuttle is used for?
formed = 2 ATPs
A. Glycolysis
ETC
B. TCA
•• NADH gives 2.5 ATP in ETC
C. Link Reaction
D. All
Ans: A) Because shuttles are required for transport
of NADH from cytoplasm to mitochondria whereas
TCA and link reaction already produce their NADH in
mitochondria. So, no shuttle is required by them.

Q. If aerobic glycolysis use glycerol-3-phosphate


•• FADH2 gives 1.5 ATP in ETC shuttle, how many ATPs are produced?

A. 2 ATP
B. 5 ATP
C. 7 ATP
D. 3 ATP
Ans. 5 ATP

•• Be careful what is asked: ATP from NADH/


FADH2 or from individual complexes I/II/
III/IV and answer accordingly.

Shuttles
•• NADH is starting material for ETC but it is
formed in cytoplasm whereas ETC occurs in
mitochondria.
•• 2 Shuttles help in transport of NADH across
the inner mitochondrial membrane (IMM) of
mitochondria.

Q
162
Cerebellum Quick Revision Notes

Energetics of Urea Cycle Energetics of Galactose Metabolism

•• Here Glucose-6-P enters glycolysis directly


thus hexokinase is not used.
•• Therefore, energetics of Galactose metabolism
is same as glucose metabolism.
•• So, like glucose metabolism galactose
metabolism produces
–– under aerobic condition = 7 ATPQ
•• At CPS-I step- 2 ATP used –– under anaerobic conditions = 2 ATPQ
•• At arginosuccinate synthetase step: ATP is Energetics of Fructose Metabolism
converted to AMP which is equivalent to 2 ATP
consumed
•• So, total ATP used to make 1 urea = 3 ATP or 4
high energy phosphates

Important Information
•• CPS-I is absent in brain
•• Arginase is absent in kidneys, so last product
of urea cycle in kidney is arginine. So, source
of arginine (semi-essential) in body is kidneys.

•• Here 2 ATPs have been used to Form


Glyceraldehyde-3-P (GAP) which is equivalent to
phase I of glycolysis where too we use 2 ATPs.
•• Also, phase II of fructose metabolism is same
as that of glycolysis. Thus, fructose metabolism
energetics is same as glucose.
163
Biochemistry

Important Information •• 2 Alanine → glucose via pyruvate → 6 High


energy phosphates
•• Fructose is the most rapidly metabolized
monosaccharide because Energetics of β-Oxidation of Fatty
–– It by-passes PFK-1 Step (rate limiting Acid
enzyme of glycolysis and a time-consuming
step)
–– So, Fructose rapidly forms Pyruvate →
Acetyl CoA → Fats. That’s why Fructose
Is also the Most Lipogenic Sugar

Gluconeogenesis Energetics Activated FA from cytoplasm reaches mitochondria


through carnitine systemQ

•• CPT-I (Carnitine Palmitoyl Transferase -I) is


Energetics the rate limiting enzyme.
•• 2 ATP used by Pyruvate carboxylase –– In fasting state, it allows acyl-CoA to go into
•• 2 GTP used by PEP carboxykinase mitochondria

•• 2 ATP used by Phosphoglycerate Kinase –– In fed state, it does not allow acyl CoA to go
into mitochondria
•• So, 4 ATPs and 2 GTPs or 6 high energy
phosphates are used to make 1 glucose from 2 •• Liver – Mitochondria
pyruvate –– Here activated FA comes for β-oxidation
•• Lactate gets converted to pyruvate and most commonly from Palmitic Acid(16C)
pyruvate is used in gluconeogenesis; no ATP is
used or produced from lactate to pyruvate.
•• Alanine is the most glucogenic amino acidQ
which is converted to glucose; Alanine gets
converted to pyruvate by Alanine transaminase
(here also no ATP used or produced)

ATP used for various substrates


•• 2 pyruvates → glucose → 6 High energy
phosphates
•• 2 lactates → glucose via pyruvate → 6 High
energy phosphates
164
Cerebellum Quick Revision Notes

–– 4 ATP produced via SLP


–– 2 ATP used each by (Hexokinase, PFK-1)
•• End products of liver glycogen is glucose, thus
ATP formed here = 2
•• But, end-product of muscle glycogen is glucose-
6-P. Thus, when it enters glycolysis, hexokinase
is not required and there is one less use of ATP.
•• Therefore, Net ATP synthesis = 4 SLP ATP -
1 ATP consumed by PFK-1 = 3 ATP

Energetics of Ketone Body Breakdown


Energetics from acetoacetate
Fate of acetyl CoA
•• Acetoacetate gets converted into 2 molecules
•• 1st → TCA of acetyl CoA
•• 2nd → KB synthesis •• 2 Acetyl CoA =20 ATP
•• 3rd → Activation of 1st step of Gluconeogenesis •• 1 less because Succinyl CoA → Succinate does
•• When acetyl CoA goes into 2nd or 3rd fates, not yield any ATP
we don’t get 80 ATPs which are generated if it •• Acetoacetate = 19 ATP
goes under TCA.
Energy From β-OH Butyrate
Q. In β-oxidation of Palmitic acid if final product is
acetoacetate then net gain of ATP is?

A. 21
B. 26
•• 1 NADH = 2.5 ATP
C. 106
•• So, for β-hydroxy butyrate total ATP produced
D. 12.9
will be = 19 (from acetoacetate) + 2.5 (from 1
Ans. B NADH) = 21.5
•• As it will not be used in TCA cycle. So, 80 ATP
Important Information
will not form.
Pathways producing zero ATPQ
•• Instead acetoacetate will form ketone bodies.
•• HMP
•• Hence, only 28 ATP are produced
•• Uronic acid pathway
•• So, net gain = 28-2 (2 ATP required for
activation of FA = 26 ATPs •• RL shunt

Q. If muscle glycogen is used for anaerobic glycolysis, •• Oxidation of VLCFA


how many ATPs are formed? •• α-oxidation of FA
A. 2
B. 7
C. 3
D. Zero
Ans. C.
•• Normally in anaerobic glycolysis
165
Biochemistry

Lysosomal Storage Diseases •• This enzyme lies in Golgi apparatus and make
•• If some lysosomal enzyme (Hydrolases) is/ hydrolases reach to lysosomes.
are absent or deficient, then that particular •• If this enzyme is defective, hydrolases will not
substrate (mostly macro-molecule) accumulates reach lysosome.
and get stored in lysosomes giving rise to
lysosomal storage disorders. •• It is also known as protein targeting disorder
as protein do not reach its target organelle
1. Mucopolysaccharidosis (MPS)
•• Clinical features
2. l-cell disease
–– Same as MPS
3. Pompe’s disease (also a Glycogen Storage
Disease) –– Serum hydrolase ↑↑

4. Sphingolipidoses 3. Pompe’s Disease


5. Wolman’s disease •• Glycogen storage disease
6. Cystinosis •• Enzyme deficiency: Acid maltaseQ

1. Mucopolysaccharidosis (MPS) •• Usually occur in children and child dies within 2


years of age
Hurler’s Disease Hunter’s disease
•• Clinical features: Hypotonia, Hepatomegaly &
Enzyme deficient: α-L- Enzyme deficient: Iduronate CardiomegalyQ
Iduronidase Sulfatase
Corneal clouding Corneal clouding absent 4. Cystinosis
present (Clear vision)Q •• Defect in cystine transporter
•• Cystine get deposited in lysosomes in various
parts of body
•• Rx: take cysteamine, which chelates cystine
•• Here, cyanide nitroprusside test is positive.

5. Sphingolipidoses
•• Dermatan sulfate •• Dermatan sulfate
+ Heparan Sulfate + Heparan sulfate
accumulation accumulation
•• AR •• XR (So, mostly males
patients)Q
•• Severe and Inguinal •• Known as Mild Hurler with
Hernia present aggressive behaviour
•• Reilly Bodies •• Reilly Bodies Inclusions
Inclusions found not found mostly

2. I Cell/Inclusion body Disease


•• Enzyme deficiency: N-acetyl glucosamine
phosphotransferaseQ
166
Cerebellum Quick Revision Notes

Important Information
•• Fabry’s disease: Only sphingolipidosis which is
X-linked recessive as all others are autosomal
recessive.
•• No Mental Retardation in Gaucher’s disease
(as glucocerebroside is not found in CNS)Q
•• No hepatosplenomegaly in Tay Sach’s disease
as gangliosides are absent in liver
•• All Sphingolipidoses have cherry red spot Clinical feature of Fabry’s disease:
except Fabry’s, Gaucher’s & Krabbe’s disease
Angiokeratomas: Benign cutaneous lesion of capillaries,
•• Sphingolipidoses with angiokeratoma: Fabry’s resulting in small marks of red to blue color &
disease- this also resembles Sickle cell crisis characterized by hyperkeratosis.
•• SLP resembling Rheumatoid arthritis: Farber’s
DiseaseQ
6. Wolman’s Disease
•• Enzyme deficient: Acid lipaseQ
Clinical images related to Sphingolipidoses:
•• It is lysosomal storage disease but not a
sphingolipidosis.
•• It is also k/a cholesterol ester storage disease
as here TG & cholesterol esters accumulate.
•• Pathognomonic feature is calcification of
adrenal glandsQ
•• Other clinical features
–– Watery green diarrhoea
–– Relentless Vomiting and failure to thrive
–– Hepatosplenomegaly

Crumpled paper appearance of macrophages called


Gaucher’s cell in Gaucher’s disease

Calcification of adrenals

Protein Targeting Disorders


Disorder is in targeting/sending the protein
at a particular site

Macular cherry red spot 1. I-Cell disease


2. Primary hyperoxaluria
167
Biochemistry

3. Zellweger Syndrome
•• Most severe peroxisomal biogenesis disorder
•• Organelle affected is peroxisomes, so that no
enzymes reach there (‘ghost’ peroxisomes)
•• In peroxisomes, α-oxidation occurs, which
breaks down Phytanic Acid. And oxidation of
VLCFA (Very Long Chain Fatty Acids)Q
•• In this disease, both these processes do not
occur, causing accumulation of phytanic acid &
VLCFA.
4. Cystic fibrosis
5. Familial Hypercholesterolemia

Clinical features of Zellweger syndrome Type I - Von Gierke’s Disease: most common GSD in
children- Clinical features
•• Severe hypoglycemia
•• Massive hepatomegaly & kidneys also enlarged
due to glycogen accumulation
•• Ketosis
•• Hyperlipidemia
•• Moon like/round/Doll like faces
•• Lactic acidosis
•• Hyperuricemia
High Forehead
Clinical features of Von Gierke’s disease
Garrod’s Tetrad
•• Inborn errors of metabolism
•• 4 diseases MNEMONIC: CAAP
–– C - Cystinuria
–– A - Alkaptonuria
–– A - Albinism
–– P - Pentosuria (Essential Pentosuria) - enzyme
deficient is Xylulose Reductase or Xylitol
Dehydrogenase

Glycogen Storage Diseases


Hepatosplenomegaly
•• If Liver affected → C/F - Hypoglycemia
•• If Muscle affected → C/F - Exercise intolerance
& muscle cramps
168
Cerebellum Quick Revision Notes

AMINO ACID TRANSPORTER DEFECTS - Common


feature is that amino acid not increased in blood, but
getting excreted in urine

Defect in
Disorder Amino acid in urine
transporter
CystinuriaQ MNEMONIC: COAL
(m/c Transporter for C - Cystine
transporter basic amino acid & O - Ornithine
defect) Cysteine A - Arginine
L - Lysine
Common
transporter for
Doll like facies
Hartnup’s tryptophan and Tryptophan (mainly)
Organ Affected in Type II, III and IV are: Muscles, DiseaseQ neutral amino acids and Neutral amino
Liver and Brain (mono-amino-mono- acids
Important Information carboxy amino acid
transporter)
•• If a GSD Q given with Muscle, Liver and Brain
Common
affected, then its easy to solve: Glycinuria
transporter for Glycine & Proline
•• Its clear that it can be either Type II, III glycine & proline
or IV
–– If Lysosomes mentioned in the Q then it Important Information
is Type II
•• In Hartnup’s disease
–– If you see limit dextrins in the Q then it is
–– Patient have pellagra i.e. Vit-B3 deficiency
Type III & if amylopectins given in the Q,
as tryptophan has role in forming Vit-B3
then type IV
(60 mg Tryptophan makes 1 mg Niacin)
Type V - Mc Ardle’s disease
–– Serum tryptophan levels not raised
•• Most common GSD in Adults
•• Characteristic feature: Lactate normal (not
Amino Acid Disorders
increased) after exerciseQ Glycine disorders
Important Information Non-ketotic
Hyperoxaluria Glycinuria
•• In Type 1 - Blood glucose level does not increase hyperglycinemia
after glucagon hormone •• Glycine •• Glycine •• Glycine cleavage
•• In Type III - Blood glucose level increase after transaminase transporter system defect
glucagon in fed state but not in fasting state deficient/ defect •• C/f
•• Cori’s disease - Glycogen accumulated with short defective •• C/f ○○ ↑ glycine in
outer branches •• C/f ○○ ↑ glycine serum, urine
•• Anderson’s disease - Glycogen accumulated with very
○○ ↑ oxalate in urine and CSF
long outer branches
in urine ○○ Urine ○○ Mental
•• Normal structure glycogen accumulated in Von
Gierke’s and Her’s disease ○○ extra oxalate retardation
•• Recombinant enzyme → recombinant human GAA renal normal ○○ Seizures
→ rhGAA; alglucosidase alpha, Myozyme → used in oxalate ○○ Risk of
Pompe’s disease deposits oxalate
•• GSD Type VII (Tarui disease) → deficiency of PFK-1 ○○ Oxalate stones
•• GSD Type 11 (XI) → Fanconi Bickel syndrome → stones
GLUT-2 mutation Q
169
Biochemistry

Phenylketonuria (PKU) Maple Syrup Urine Disease (MSUD)


•• Enzyme deficient: Phenylalanine hydroxylaseQ •• Enzyme deficient: Branched Chain Keto Acid
(BCKA) dehydrogenase or decarboxylaseQ
•• Accumulation of Branched chain amino acids and
keto acids occurs
•• Urine odor is of Burnt sugar / Maple syrup
odor
•• Clinical features
[Phenyl-keto-uria → phenyl- keto acid (phenyl Pyruvate) –– Mental Retardation
- in urine] –– Ketosis
•• C/F
–– If untreated, coma & death can occur
–– Mousy/musty body odour (due to
phenylacetate)Q –– Has high mortality rate
–– Severe MR (due to Phenylalanine) •• Rx
•• Diagnosis 1. Vitamin B1 supplementation (coenzyme for
–– FeCl3 urine test → Green colour (d/t Phenyl BCKA dehydrogenase)
Pyruvate)Q 2. Dietary restriction of Branched Chain Amino
•• Rx Acids

1. Lifelong Restriction of phenyl alanine in diet Alkaptonuria/Black bone disease


ƒƒ Aspartame (artificial sweetener) is contraindicated •• Enzyme defect: Homogentisate dioxygenase
as aspartame is made of Aspartate + Phenylalanine (previously k/a Homogentisate oxidase)Q
2. Give Tyrosine & Tryptophan •• Homogentisic acid accumulates
3. Tetra Hydro Biopterin (THB) Supplementation •• C/F
helps in some patients
–– On standing color of urine turns black due to
Clinical Features of phenylketonuria oxidation of Homogentisic acid
–– Age of onset: 30 - 40 years
–– no MR (Mental Retardation)
–– Homogentisic acid gets polymerized in to
alkapton bodies which accumulates in:
→ Cartilages (Nose, Ear Pinna, IV discs):
Arthritis
→ Connective tissue: Bluish black colour
→ Intervertebral discs: lower back pain
(due to
–– Collectively, accumulation of homogentisic
acid in all body parts is termed as Ochronosis
and arthritis as Ochronotic arthritis.

Clinical features of AlkaptonuriaQ


170
Cerebellum Quick Revision Notes

1. Acquired
•• Due to vitamin B6, B9 and B12 deficiency
2. Genetic
•• Due to Enzyme Cystathionine beta Synthase
defectQ
•• Here cysteine becomes essential.
•• c/f
–– ↑ Serum Homocysteine (has SH - group)
–– ↑ urine Homocystine (made of two
homocysteine joined by S-S bond)
Cyanide nitroprusside test is positive.

Clinical features of homocystinuria

Albinism
•• Enzyme deficient: Tyrosinase (makes melanin
from tyrosine)
•• Tyrosine is an oxidase enzyme & requires copper
for its action.
•• c/f
•• Hypopigmentation (milky white skin, Blonde Important Information
hair, and Red eye color)
•• How to distinguish between vit B6, B9 and B12
deficiency
Vitamin
Substrate accumulated
deficiency
B6 Homocysteine, Xanthurenic acid
Homocysteine, FIGLU (Form Imino
B9
GLUtamate)
B12 Homocysteine, L-methyl malonyl acid
Important Information
Vitiligo
•• Patchy hypopigmentation
•• Tyrosinase is normal

Homocystinuria (HCU)
Two types:
171
Biochemistry

DISORDERS OF CARBOHYDRATE •• Accumulation of Fructose-1-P occurs in liver &


kidney.
METABOLISM
•• Clinical features
Galactosemia
Minor Classical (M/C Galactosemia) –– Jaundice and hepatomegaly
–– Hypoglycemia (as fructose-1-PO4 inhibits
•• Enzyme deficient: •• Enzyme deficient: GALT liver glycogen phosphorylase thereby
Galactokinase [Galactose-1- PO4 uridyl blocking glycogenolysis)
•• C/F transferase]Q
–– If untreated it causes renal failure
○○ Oil drop •• GALT converts Gal-1-P to
cataract UDP-galactose Features Disease
•• Accumulation of galactose-1 CNS, Skin and hair affected Multiple carboxylase
-PO4 with Tom cat urine odor deficiency
•• C/f
CNS affected +
○○ Oil drop cataract PKU
hypopigmentation
○○ Jaundice & Hepatomegaly
CNS affected + Burnt sugar
○○ Mental retardation MSUD
odour

Child (young person)


Clinical features of galactosemia with stroke, skeletal
abnormalities, knock knees, HCU
pectus carinatum, elongated
limbs

Patient with organomegaly,


easy bruising, bony pain and Gaucher’s disease
frequent fractures

Coarse facial features Lysosomal storage disease

Snow-flake cataract Diabetes

Oil drop cataract Galactosemia

Sunflower cataract with KF Wilson’s disease due to


ring Cu def

Oil drop cataract

Hereditary Fructose Intolerance


•• Enzyme deficient: Aldolase B
172
Cerebellum Quick Revision Notes

PREVIOUS YEAR QUESTIONS C. Alkaptonuria


D. MSUD
Q. Glutamine is increased in CSF, blood & urine in
which defect? (NEET Jan 2019) Exp: Alkaptonuria is a metabolic disorder
characterized by a deficiency of homogentisic acid
A. Arginosuccinate Synthetase oxidase, leading to the accumulation of homogentisic
B. OTC acid, which causes dark urine and pigmentation of
C. CPS – I various tissues.
D. Arginase Q. What is seen in G6PD deficiency ? (INICET July
Exp: In all given enzyme deficiency, ammonia will 2021)
increase but CPS-1 is rate limiting enzyme, so it will A. Decrease membrane lipid peroxidation
increase ammonia most & increase in ammonia lead to
B. Decrease generation of reduced glutathione
increase in glutamine.
C. Decrease NADH generation
Q. A boy develops hypoglycemia after moderate D. Decrease scavanging of RBC by macrophages
activity. Blood report shows raised ketones, lactic acid
E. Hemolysis
and triglycerides. On examination, liver and kidney
were found to be enlarged and patient is diagnosed Exp: Glucose-6-phosphate dehydrogenase (G6PD)
with Von Gierke’s disease. Which enzyme deficiency deficiency leads to hemolysis of red blood cells,
he has? (FMGE Dec 2020) particularly under conditions of oxidative stress due
to low availability of NADPH and reduced form of
A. Glucose 6 phosphatase
Glutathione.
B. Acid Maltase
C. Glycogen synthase Q. In MSUD (Maple Syrup Urine disorder) which of
the following is not restricted: (INICET May 2022)
D. G-6-PD
A. Methionine
Exp: Von Gierke’s disease: enzyme deficient is Glucose
6 phosphatase, leading to impaired glucose release B. Valine
from glycogen and resulting in hypoglycemia and other C. Leucine
metabolic abnormalities D. Isoleucine

Q. Glycogen stored in liver in all the following Exp: In MSUD, the branched-chain amino acids
conditions except? (FMGE Aug 2020) (leucine, isoleucine, and valine) are restricted in the
diet, due to defect in their catabolism.
A. Gaucher’s disease
B. Cori’s disease Q. Brunt sugar smell of urine is due to the defect of?
C. Mc Ardle’s disease (INICET Nov 2022)
D. Pompe’s disease A. Phenylalanine hydroxylase
Exp: Gaucher’s disease, is a lysosomal storage disorder B. Isovaleryl CoA dehydrogenase
of sphingolipids and not glycogen, but all others Cori’s C. Fumarylacetoacetate hydrolase
disease, Mc Ardle’s disease, or Pompe’s disease are D. Branched chain keto acid dehydrogenase
Glycogen storage disorders.
Exp: Burnt sugar like odor is a characteristic feature
Q. A 40-year-Old Male Patient come in OPD and of Maple Syrup Urine Disease (MSUD) which occurs
complaining of long-standing black Urine ferric due to the inherited deficiency of branched Chain
Chloride and Benedict’s test both were found to be Keto-acid dehydrogenase.
positive in this patient. Pinna and sclera of eyes have
Q. Zellweger syndrome is caused by the defect of
black pigmentation. What is your diagnosis? (FMGE
which of the following organelles? (INICET Nov 2022)
Dec 2020)
A. Mitochondria
A. Phenylketonuria
B. Lysosomes
B. Galactosemia
C. Peroxisomes
D. Nucleus
173
Biochemistry

Exp: In Zellweger syndrome, there are empty C. Tarui’s disease


peroxisomes in all body cells causing defect in alpha- D. Anderson’s disease
oxidation and oxidation of VLCFA. It is the most
severe Peroxisomal Biogenesis Disorder (PBD). Exp: Mc Ardle’s disease is characterized by deficient
glycogen phosphorylase in muscle cells, leading
Q. A young male with a H/o dyslipidemia having ABC-A1 to impaired glycogen breakdown and decreased
mutation shows the following. See the image and tell availability of glucose for energy production during
the diagnosis. (INICET Nov 2022) exercise.

Q. A 4-year-old child easy fatigability. The mother


also complained that the child was more hungry
between meals, and the child recovers after food.
Liver examination revealed no glycogen? (NEET PG
2023)
A. Branching enzyme
B. Debranching enzyme
C. Glucose 6 phosphatase
D. Glycogen synthase
A. Wolman’s Disease
B. Gaucher’s Disease Exp: Glycogen synthase deficiency (also known as
Glycogen Storage Disease Type 0) leads to impaired
C. Tangier’s disease
glycogen synthesis in the liver, resulting in low glycogen
D. Tay Sach’s disease stores and symptoms such as fatigue and hunger
Exp: Large orange/ yellow tonsils (due to fat Q. A child presented with difficulty in vision on
deposition) are a characteristic feature of Tangier’s examination cherry red spots were seen on macula.
disease due to defect in ABC-A1 transporters. There was no organomegaly. Identify the disease.
Q. Identify the given image (INICET Nov 2022) (NEET PG 2023)
A. Gaucher disease
B. Hunter disease
C. Tay-Sachs disease
D. Niemann pick disease
Exp: Tay-Sachs disease is characterized by a
deficiency of the enzyme Hexosaminidase A, leading
to the accumulation of gangliosides in the brain and
retina, causing vision problems and cherry red spots
on the macula.

Q. In Pompe’s disease, enzyme deficiency is? (INICET


A. Gaucher’s disease May 2023)
B. Wolman’s disease
A. Lysosomal acid alpha glucosidase
C. Niemann pick disease
B. Phosphofructokinase
D. Fish eye disease
C. Muscle glycogen phosphorylase
Exp: The given image shows crumpled tissue D. Lactate dehydrogenase
paper appearance of macrophage cells which is a
characteristic feature of Gaucher’s disease Exp: In Pompe’s disease, there is a deficiency of the
enzyme lysosomal acid alpha glucosidase (also known
Q. A child was fatigued after exercise and the blood as acid maltase), which leads to the accumulation of
sample collected soon after exercise showed low glycogen in lysosomes and subsequent muscle and
glucose and lactate level. The probable diagnosis is. organ dysfunction.
(NEET PG 2023)
Q. Xanthurenic Acid in urine indicates: (INICET May
A. Mc Ardles disease 2023)
B. Von Gierke’s disease
174
Cerebellum Quick Revision Notes

A. Vitamin B6 deficiency Optical Isomerism


B. Vitamin B1 deficiency
•• When plane polarized light is allowed to pass
C. Niacin deficiency through the solution of carbohydrate, then it
D. Vitamin B2 deficiency gets rotated:
Exp: Xanthurenic acid in urine is an indicator of niacin –– If it gets rotated to right side, the compound
(vitamin B3) deficiency. It is formed when tryptophan is called dextro-rotatory and designated as
metabolism is impaired due to insufficient niacin levels. d or (+) isomer
–– If it gets rotated to left side, the compound
CARBOHYDRATE CHEMISTRY
is called levo-rotatory and designated as l or
Carbohydrates (-)

Definition: Poly hydroxy (many OH) Aldehyde or Structural isomerism


Ketones
•• 4 Types
•• Number of OH groups are always one less than
1. Functional Isomers → Different functional group
the number of carbons. e.g. Glucose (6C) has 5
(aldehyde or keto)Q
OH & Ribose (5 C) has 4 OH groups

Isomerism
•• Isomerism is possible due to Asymmetric/chiral
carbon
–– Asymmetric C - When all of 4 valencies of
C are occupied by different atoms/group of
atoms. e.g.

•• Both carbohydrates and amino acids have


Asymmetric C, so both show isomerism.
Important Information:
–– C2 to C5 are asymmetric carbons in glucose,
•• Racemic mixture (equal d & l) So, it is optically so total 4 asymmetric carbons
inactive
2. Enantiomers- D-L isomerism/Mirror imagesQ
•• Racemase enzyme: Enzyme which interconverts
•• Different H & OH orientation around the 2nd
‘D’ ↔ ‘L’ not d and l. Thus, name Racemase is a
last carbon
misnomer. D & L are structural isomers, but d
& l are optical isomers.

Isomerism is of 2 Types:
1. Structural isomerism a.k.a. stereoisomerism (same
molecular formula, different Structure)
2. Optical isomerism (same molecular formula,
different optical properties)
175
Biochemistry

4. Anomerism

•• Seen only in cyclic structures


•• Anomers have different H and OH orientation
around functional carbon (C1 or C2)

Important Information
•• In case of carbohydrates → D-Form is
abundant.
•• In case of amino acids in Proteins → L-form Important Information
is abundant.
•• Pyranose: Six membered ring with 5 C & 1 Oxygen
•• Free amino acid (Amino acid not in protein) → •• Furanose: Five membered ring with 4 C & 1 Oxygen
can be L or D Form, e.g. D-serine & D-aspartate ○○ Glucose → mainly Pyranose
(Found in brain)
○○ Fructose → mainly Furanose
•• Amino acid synthesized in body → L-form ○○ Hexoses (6C) → Both Pyranose & Furanose exists
•• Source of D-Amino acid → always exogenous ○○ Pentoses (5C) → Only Furanose exists
(Diet or Bacteria flora)
Anomers are of two types- alpha and beta

Q. Which form of amino acid is present in body?

Ans: Both D & L

Q. Which form of amino acid is present in proteins?

Ans: Only L

3. Epimerism

•• Different H or OH orientation only around one


Glucose Transport
carbon other than penultimate Carbon •• Methods
–– Mannose is epimer of glucose at C2 Active (Secondary) Facilitative Diffusion
Transport
–– Galactose is epimer of glucose at C4
•• Na-dependent Glucose •• Na-independent GLUTs
•• But mannose and galactose are not epimers of Transport (SGLT) (GLUcose Transporters)
each other •• Occurs against •• Occurs down the
concentration concentration gradient
gradient
•• Bidirectional (allow both
•• Unidirectional (only
entry as well as exit of
allow entry of glucose
into the cell)Q glucose from cell)Q
•• Also called Na-Glu
symport
•• ATP is indirectly
used for Na+/K+
ATPase pump. So, it
is a secondary active
transport.
176
Cerebellum Quick Revision Notes

Types of SGLTs 3. Dextran → used as plasma volume expander


Transporter Sugar transported Location 4. Cellulose → made of β-glucose
Name
5. Inulin → made of β-fructose
SGLT-1 Glucose and Galactose Both Intestine
and kidneys 6. Chitin → made of N-acetyl glucosamine
SGLT-2 Glucose only Only Kidneys Hetero-Polysaccharides: made of repeating unit of
Amino sugar & Uronic acid (oxidation at C6)
•• Mutation in SGLT-1 leads to Glu-Gal
malabsorption •• Only GAG which is not
sulfatedQ
•• Mutation in SGLT-2 leads to Familial Renal
•• Longest GAG
glycosuria or Renal Glucosuria
Hyaluronic Acid •• Has role in wound healing
and cell migrationQ
•• Located in Synovial Fluid &
Vitreous humour
•• Most abundant GAGQ
Chondroitin Sulfate
•• Present in Cartilage, Bone,
(CS)
Tendon
•• Found in Skin, Blood
Dermatan Sulfate (DS)
vessels, heart valves
•• Only GAG without uronic
acidQ
•• Most heterogenous GAG
Keratan Sulfate
•• Present in cornea &
connective tissue
Types of GLUTs
•• Responsible for
Name Tissue Location Function
Transparency of corneaQ
Brain, RBC, Placenta,
GLUT-1 Basal Glucose Uptake •• More sulphated than
Kidneys
Heparan sulfate
Intestine- absorption
•• Highest negative charge
Liver- Glycogen
Liver, Pancreas, •• Very good but costly
GLUT-2 formation
Intestine, Kidneys anti-coagulant (prevent
Pancreas- Insulin
coagulation but do not
secretion Heparin
dissolve clots)
Brain (Neuronal),
GLUT-3 Basal Glucose uptake •• Activates antithrombin III
Placenta, Kidneys
to slow down or delay the
Skeletal muscles, Insulin stimulatedQ progression of DVT
GLUT-4 Cardiac muscles, Glucose uptake after •• Released from mast cell &
Adipose tissues meals liver
Small intestine, •• Present on cell surfaces
GLUT-5 Testis (Sperms), Fructose transport Heparan Sulfate (HS) •• Has a role in cell–cell
Kidneys
adhesion e.g. retinal cell-
cell attachment.Q
Polysaccharides
Homopolysaccharides – made of same carbohydrate
units
1. Starch → made of α-glucose (storage form in plants)
2. Glycogen → made of α-glucose (storage form in animals)
177
Biochemistry

Proteoglycan Glycoprotein

Carbohydrate >> Protein Protein >> Carbohydrate

Carbohydrate is either a mono-


Carbohydrate is always
saccharide or, oligosaccharide
a Heteropolysaccharide
but never a Polysaccharide
e.g. Aggrecan is a
e.g. Fibrous protein- Collagen,
proteoglycan made of 2. Benedict’s testQ
All plasma proteins except
Chondroitin sulfate and
Albumin •• Given positive by reducing sugars
protein
•• a semi-quantitative test as different colors
Fibres gives an idea of the amount of sugar present

Insoluble fibres Soluble fibres


•• Excreted unchanged •• Absorbs H2O & converted
from intestine to Gel form, which is
•• Examples: excreted
1. Cellulose •• Better in preventing
2. Hemicellulose Constipation
•• Provide more osmotic load
to intestine
•• Examples:
1. Pectins
2. Gums
3. Inulin Test For Ketone Bodies

Disaccharides Rothera’s TestQ


Glycosidic •• Done for detection of ketone bodies
Disaccharides Constituents Nature
bond
•• A positive Rothera’s test is when purple color
Maltose Glucose + Glucose α (1→4) Reducing ring is seen at the junction of two liquids
Isomaltose Glucose + Glucose α (1→6) Reducing
Non-
Trehalose Glucose + Glucose α (1→1)
Reducing
Glucose + Non-
Sucrose α (1→2)
Fructose Reducing
Galactose +
Lactose β (1→4) Reducing
Glucose
Tests For Amino Acids And Proteins
Tests For Carbohydrates
Ninhydrin Test
1. Molisch test
•• Done for detection of α-amino acids
•• General test given by all carbohydrates
•• Positive Ninhydrin test = Purple colour solution
•• Condition: No. of carbons must be ≥ 5
178
Cerebellum Quick Revision Notes

Biuret test A. Liver


B. Pancreas
•• It is done for detection of proteins and peptides
C. Muscle and adipose tissues
•• Positive Biuret Test = purple colour solution
D. Placenta
•• A tripeptide and higher peptides will give a
Exp: Insulin facilitates glucose uptake by muscle and
positive test.
adipose tissues by promoting the translocation of
glucose transporters (GLUT4) to the cell membrane.

Q. A 15-year old boy, a known case of type I DM,


undergoes the following test for the detection of KB.
Which is this test: (INICET May 2023)

OSAZONES: are crystals formed by Reducing sugars,


can be seen in microscope

A. Rothera’s Test
B. Benedict’s test
C. Seliwanoff’s Test
D. Biuret Test
Exp: The Rothera’s test is used to detect the presence
of ketone bodies (ketones) in the urine, which can be
elevated in conditions like diabetic ketoacidosis.

CARBOHYDRATE METABOLISM
GLYCOLYSIS or EMP (Embden Meyerhof pathway)
•• Inhibitors of Glycolysis
•• Iodoacetate & Arsenite → inhibits glyceraldehyde-
3-P dehydrogenase
PREVIOUS YEAR QUESTIONS –– Na Fluoride → inhibits Enolase (used in blood
glucose estimation)
Q. A patient report is as follows; FPS - 82: PPBS -
140; Urine was found to be positive for benedict test. Important Information
What is the probable diagnosis? (JIPMER - Dec -
•• When arsenite inhibits glycolysis, then no
2019)
ATP is produced but glycolysis continues, and
A. Renal glycosuria pyruvate is formed.
B. Transient glycosuria
C. Alimentary glucosuria Irreversible steps / Substrate level
D. DM Regulatory steps phosphorylation (SLP)
1. Hexokinase 1. PG Kinase
Exp: The presence of glucose in the urine (positive
Benedict’s test) in a patient with normal fasting 2. PFK 1 2. Pyruvate Kinase
and postprandial blood sugar levels suggests renal 3. Pyruvate Kinase
glycosuria, which is a benign condition.
Link Reaction
Q. Which of the following tissues are dependent on
Enzyme - PDH (Pyruvate Dehydrogenase) complex,
insulin for glucose uptake (INICET Nov 2022)
irreversible reaction, occurs in mitochondria
179
Biochemistry

Important Information
Malate (4C): Intermediate of TCA Cycle
Malonate (3C): Inhibitor of TCA Cycle
Malonate / Malonyl CoA is an inhibitor of:
1. TCA Cycle (inhibits Succinate Dehydrogenase)
2. ETC (inhibits Complex II)
3. Beta Oxidation of Fatty Acids (inhibits CPT-1)

Important information
•• 5 coenzymes are required for release of
energy from link reaction & TCA: Lipoic acid,
B1, B2, B3, B5. So, multivitamins are given
•• In B1 deficiency (Beri-Beri) or PDH complex during LethargyQ
deficiency, lactic acidosis will occur due to
excess pyruvate converting in to lactate in ETC
cytoplasm. •• Components of ETC (Located in Inner
Mitochondrial Membrane)
TCA Cycle
–– 5 protein complexes (I to V)- fixed within
•• Vital cycle of cell (occur in fed as well as fasting
the membrane
state), occurs in mitochondria in aerobic state
–– 2 Mobile molecules: Coenzyme Q and
•• TCA depends on 2 things:
Cytochrome C (a Peripheral membrane protein)
–– Energy status of cell: If ATP present, TCA
Note: Coenzyme Q is the only non-protein component
will not occur & vice-versa.
of ETC
–– Availability of oxaloacetate: Oxaloacetate
–– In ETC, oxidation & phosphorylation occur
is also regarded as carrier of TCA cycle or
together (coupled).
1st Substrate of TCA cycle. Also, it plays a
catalytic role in TCA cycle. –– Uncoupling means that oxidation occurs but
not phosphorylation.
Important Information
•• Uncouplers e.g.
•• Only 1 SLP occurs in TCA done by enzyme succinate
thiokinase which mostly produces ATP, but in liver & –– Dinitrophenol (drug)
kidney during fasting/starvation state, it produces –– Natural/Physiological uncouplers
GTP, as during this state, gluconeogenesis occurs & it
1. Thermogenin: present in brown fat in
requires GTP.
hibernating animals & in neonates & is
•• Acetyl CoA is not an intermediate of TCA cycle. responsible for non-shivering thermogenesis
•• 2 CO2 which are removed in TCA comes from
2. Thyroxine
Oxaloacetate. If oxaloacetate not given, then mark
Acetyl CoA. 3. Free fatty acids

Inhibitors of TCA Important Information

•• Malonate (3C) → Inhibits Succinate dehydrogenaseQ Q. ADP to ATP conversion in ETC is inhibited by?
•• Arsenite → Inhibits α-ketoglutarate Ans. Oligomycin (inhibits complex V which converts ADP
dehydrogenaseQ to ATP)
•• Fluorocitrate → Competitively inhibits Aconitase Q. ADP to ATP transfer in ETC is inhibited by?
•• Fluoroacetate → Non- competitively Inhibits Ans. Atractyloside (inhibits ADP-ATP translocase, which
Aconitase transfers ADP & ATP)
180
Cerebellum Quick Revision Notes

Inhibitors of ETC ComplexesQ


Complex Inhibitor
Complex I Rotenone, Phenobarbitone, Amobarbital,
Piericidin A
Complex II Malonate (3C)
Carboxin (fungicide)
TTFA (Thenoyltrifluoroacetone)
Complex III Phenformin, Antimycin A
BAL {British anti-Lewisite} or
Dimercaprol
Important Information:
Complex IV CO, CN, H2S, Sodium Azide •• If both leucine and lysine are given in options for
ketogenic amino acid, mark leucine as it is more
Gluconeogenesis ketogenic

•• 4 Enzymes of Gluconeogenesis which are •• Purely ketogenic amino acids = 2


different from Glycolysis (refer fig in •• Purely glucogenic = 13
Energetics) •• Total ketogenic = 2+5 = 7

Enzyme Feature •• Total Glucogenic = 13+5 = 18

•• Present in mitochondria
Pyruvate carboxylase Reciprocal regulation of Glycolysis &
•• Uses ATP
•• Activated by acetyl CoA Gluconeogenesis
PEPCK
(Phosphoenolpyruvate
Both are present is cytoplasm
carboxykinase)
Fructose -1-6-
Bisphosphatase
•• Present in endoplasmic
reticulum (to prevent glucose-
6-P breakdown to glucose in
cytoplasm)
Important Information
•• Common enzyme of
Glucose-6 - •• Fructose 1,6-Bisphosphate compound → Glycolysis
gluconeogenesis and
phosphatase intermediate
Glycogenolysis
•• Present in liver but absent in •• Fructose 1,6-Bis Phosphatase enzyme →
muscles Gluconeogenesis enzyme

•• Deficient in Von Gierke’s •• Fructose 2,6-Bisphosphate compound → Reciprocal


disease Regulator
•• Fructose 2,6-Bis Phosphatase enzyme → active in
•• Substrates of Gluconeogenesis
Cancerous cells (not in normal cells) causes down
–– Pyruvate and Lactate regulation of glycolysis
–– Glycerol and Propionic acid TP-53 induced cancer suppressor mechanism
–– Glucogenic amino acid (most glucogenic is
Alanine)
–– Both Glucogenic & Ketogenic amino acids
–– Any TCA intermediate
181
Biochemistry

Other Names:
•• Debranching enzyme → Amylo-α(1→6)-glucosidase
•• Glucan transferase → oligo-1,4-1,4-glucantransferase
or 4:4 transferase
•• Branching enzyme → amylo-(1,4→1,6)-trans
glycosylase or 1,4-alpha-glucan-branching enzyme
•• Acid maltase also called as Lysosomal acid alpha
glucosidase

HMP
•• Glucose 6–P (a hexose phosphate) is the
TIGAR- TP-53 Induced Glycolysis and Apoptosis starting material hence the name Hexose
Regulator Monophosphate Pathway

•• TIGAR decreases glycolysis, regulates •• Other name is Pentose Phosphate Pathway


apoptosis & is functionally similar to Fructose- (PPP) as Pentose Phosphate i.e. Ribose 5–P is
2,6-bisphosphatase synthesized only by this pathway.Q

Glycogen •• Produces NADPH in phase-I of HMP (irreversible


•• Stored mainly in liver & muscle and minor amount oxidative phase)Q
in brain •• Produces Ribose-5-P in phase-II of HMP
Storage Main Function End-product (reversible Non-oxidative phase)
Organ
•• It’s an anabolic pathway, thus is activated by
Liver Maintains Blood Glucose Free Glucose
Insulin & occurs in cytoplasm
Muscle Muscle contraction Glucose- 6-P
•• When muscle glycogen is used for anaerobic •• RLE → Glucose-6-P dehydrogenase (G6PD)
glycolysis, then 3 ATPs are obtained
•• Site of HMP → Liver, Lactating mammary
•• RLE for glycogenesis: Glycogen synthase, makes glands, adipose tissue, placenta, gonads & RBC.
α (1-4) bond
•• RLE of glycogenolysis: Glycogen Phosphorylase, •• Tissues which are never the site of HMP → Skin
breaks α (1-4) bond, Requires Vitamin B6, and Non-lactating mammary glands
Releases 90% of glucose in the form of glucose-
1-P •• Molecule which is intermediate as well as end
product in HMP → Glucose-6-P
•• Debranching enzyme in glycogenolysis breaks
α (1-6) bond & releases 10% of glucose in the •• Molecule which is substrate as well as product
form of D-glucose
in HMP → Glyceraldehyde-3-P
Important Information
•• Both RLE for Glycogen metabolism are Transferases
•• Both pathway of glycogen metabolism occurs in
cytoplasm
•• For Glycogen Synthesis, a Primer Glycogenin (protein) Requires
is required.
•• Enzyme common between glycolysis and glycogenesis:
Hexokinase/ Glucokinase.
•• Enzyme common between glycogenesis &
glycogenolysis: PhosphoGluco-Mutase (interconverts
glucose-1-P and glucose-6-P)
182
Cerebellum Quick Revision Notes

Important Information: PREVIOUS YEAR QUESTIONS


•• Marker of Vitamin B2 deficiency: RBC Glutathione Q. Amino acid linking kreb’s cycle & urea cycle?
Reductase activityQ (NEET Jan 2019)
•• Marker for Vitamin B1 deficiency: RBC Transketolase A. Aspartate
activityQ
B. Fumarate
In G6PD deficiency C. Alanine
D. Arginine
•• Oxidative Stress is main reason for hemolysis
Exp: Aspartate serves as the precursor for
•• G6PD deficiency → ↓ NADPH production in RBC
fumarate, an intermediate in the Krebs cycle, and also
→ ↓ reduced glutathione → ↑ H2O2 → cause cell participates in the urea cycle by donating its amino
membrane lysis thus, hemolysis group.

Q. A Mediterranean person didn’t receive Primaquine. Q. All are features of gluconeogenesis except?
Which pathway gets affected by G6PD deficiency? (JIPMER Dec 2019)
A. Gluconeogenesis is synthesis of glucose from non-
Ans: HMP, as antioxidant drugs like Anti-malarials carbohydrate source
increase the oxidant stress, which further increases B. Mainly takes place in liver
H2 O 2 C. Seen in fasting state
D. Step are simple reversal of glycolysis
Sorbitol Pathway
Exp: Gluconeogenesis involves different enzymatic
reactions from glycolysis, including bypass reactions
to overcome the irreversible steps of glycolysis.

Q. Glucose is stored in Glycogen form, why? (AIIMS


June 2020)
A. Compact
•• Sorbitol accumulation in lens is responsible B. Can be reduced from multiple branches lends
for snow flake cataract in Diabetes (enzyme C. Can be stored ale multiple Sites
involved is Aldose Reductase)Q D. Can provide glucose an muchas needed and when
•• Galactitol accumulation in lens is responsible needed for 1 week
for oil drop cataract in Galactosemia (enzyme Exp: Compactness of glycogen allow storage in smaller
involved is Aldose Reductase) space as compared to equivalent glucose. Note that
400mM glucose stored as glycogen is only about 0.01
•• No cataract in HFI (Hereditary Fructose
uM.
Intolerance) because Fructose is not the
substrate for Aldose Reductase Q. Complex IV in Election Transport chain is inhibited
by? (FMGE Dec 2020)
Wilson’s disease: Copper in Descemet membrane
A. Cyanide
leads to KF (Kayser Fleischer) rings. Also, these
B. Barbiturates
patients have Sunflower cataract due to accumulation
C. Phenylnitrone
of Copper in lens
D. Malonate
Exp: Cyanide binds to and inhibits cytochrome c
oxidase, which is a component of complex IV in the
electron transport chain.

Q. Enzymes used in gluconeogenesis are? (INICET


Nov 2020)
183
Biochemistry

A. Hexokinase phosphate pathway, provides a source of NADPH and


B. PEPCK ribose 5-phosphate, which are important for various
biosynthetic processes.
C. Pyruvate carboxylase
D. GLU-6 Phosphatase Q. True about Warburg effect? (NEET 2022)
E. Pyruvate kinase A. Makes cancer cells immortal
Exp: These three enzymes play key roles in the B. Aerobic glycolysis with lactate formation
gluconeogenesis pathway to bypass irreversible steps C. Produce more ATP
of glycolysis.
D. Decrease glucose uptake
Q. Enzyme present in both glycolysis and Exp: The Warburg effect refers to the phenomenon
gluconeogenesis? (AIIMS June 2020) of cancer cells preferring aerobic glycolysis over
oxidative phosphorylation, leading to increased
A. PFK lactate production.
B. PEP-CK
Q. FAD status in body is determined by? (NEET 2022)
C. Phosphoglycerate kinase
D. Pyruvate kinase A. Thiamine
B. Glutathione reductase
Exp: The common enzyme of glycolysis and
gluconeogenesis is phosphoglycerate kinase C. Hexokinase
which interconverts 1,3-Bisphosphoglycerate and D. Transketolase
3-phosphoglycerate.
Exp: Glutathione reductase is involved in the
regeneration of reduced glutathione, which, in turn,
Q. Cytochrome C oxidase is inhibited by all EXCEPT?
maintains the reduced state of FAD.
(INICET July 2021)
Q. Which of the following compound(s) is most likely
A. Methane to cause diabetic cataract: (FMGE Jan 2023)
B. H2S
A. Accumulation of sorbitol
C. CO
B. Accumulation of galactose + glucose
D. Cyanide
B. Accumulation of galactitol + glucose
Exp: Methane is not known to inhibit cytochrome c
oxidase, whereas the other options H2S, CO, and C. Accumulation of glucose only
cyanide are known inhibitors of this enzyme. Exp: In diabetes, the accumulation of sorbitol due to
increased aldose reductase activity can lead to the
Q. Glycogen phosphorylase is activated directly or development of diabetic cataract.
indirectly by? (INICET Nov 2022)
Q. Which of the following enzymes simultaneously
A. Calcium incorporates molecular oxygen and produces water?
(FMGE Jan 2023)
B. Glucose 6 Phosphate
C. Insulin A. Catalase
D. Glucose B. Cytochrome C oxidase
Exp: Calcium as Ca-Calmodulin complex directly C. Acetyl CoA carboxylase
activates glycogen phosphorylase, leading to the D. Choline esterase
breakdown of glycogen into glucose.
Exp: Cytochrome c oxidase, found in the mitochondrial
Q. The significance of HMP shunt are all except? inner membrane, uses molecular oxygen as a substrate
(INICET Nov 2022) to produce water as the final product of the electron
transport chain.
A. Source of NADPH
B. Source of ribose 5 phosphate ENZYMES
C. Source of acetyl CoA •• Enzymes are proteins except ribozyme in which
D. Utilizes ribose RNA acts as enzyme.
Exp: The HMP shunt, also known as the pentose •• Coenzymes are heat stable
184
Cerebellum Quick Revision Notes

•• Enzymes are not heat stable This graph is for Simple enzymes
•• Enzymes are stereo-specific •• Michaelis-Menton constant [Km]Q
–– Km is defined as that substrate concentration
at which velocity of reaction is half of Vmax
–– Km is signature of Enzyme as it is a constant
value for a particular enzyme
–– Km does not change with change in either
enzyme or substrate concentration
–– So, in case of competitive inhibition; affinity↓
so, Km ↑

Graph between [E] & [S] for Allosteric


•• Active site: Binding Site + Catalytic site or Regulatory Enzymes
–– Serine proteases have a role in tumor cell
metastasis (e.g. Chymotrypsin, Trypsin,
elastase, plasmin, thrombin, clotting factor
10 & 11 and prostate specific Antigen)

Important Information
•• Chymotrypsin cuts at C- terminal of large
hydrophobic amino acids like Phe, Tyr and Trp
•• Trypsin cuts at carboxy terminal of basic amino acids
like Lys and Arg
•• Elastase cuts at carboxy terminal of small neutral
amino acids e.g. glycine, serine.

Enzyme Kinetics Graphs

•• Kcat → The number of Substrates converted to


product per enzyme molecule per sec

Q. Catalytic efficiency of enzyme is best expressed


by which kinetic constant?

Ans: Kcat/ KmQ


185
Biochemistry

Enzyme Classification
Enzyme Class (EC no.) Distinguishing Feature
1. Oxidoreductases
•• Oxidases •• Use O2 as an electron acceptor like cyt C Oxidase
•• Dehydrogenase •• Use molecules other than O2 as electron acceptor (NAD, FAD, NADP); e.g. PDH in
link reaction.
•• Use H2O2 as electron acceptor; e.g., Glutathione peroxidase
•• Peroxidase
•• Incorporate O2 into the substrate; 2 types:
•• Oxygenase
○○ Dioxygenase: incorporates 2 atoms of molecular O2 into the substrate e.g.
homogentisate dioxygenase
○○ Monooxygenases/Hydroxylases/mixed function oxidases: incorporates 1 atom of
molecular O2 into the substrate. e.g. phenylalanine hydroxylase (converts Phe to
Tyr)
•• Reductase •• Example: Glutathione Reductase
2. Transferases
•• Methyltransferase •• Transfer one carbon units
•• Aminotransferase •• Transfer amino groups
•• Kinase •• Transfer phosphate from ATP
•• Phosphorylase •• Transfer phosphate from Pi
3. Hydrolases
•• Phosphatase •• Remove phosphate from a substrate using water
•• All digestive enzymes •• Any enzyme that breaks macromolecule e.g. amylase, maltase etc.
4. Lyases
•• Synthases •• Link 2 molecules without using ATP
•• Aldolase A & B •• Produce aldehydes via elimination reactions
•• Simple Decarboxylases •• Produce CO2 via elimination reactions
•• Hydratase •• Add or remove water but do not break bond e.g. Enolase, aconitase, fumarase, PEPCK
5. Isomerases
•• Racemase •• Interconvert L & D stereoisomers
•• Mutase •• Transfer group b/w atoms within a molecule
•• Epimerase •• Interconvert epimers
6. Ligase
•• Synthase
•• Link 2 molecule via an ATP-dependent
•• Carboxylase
•• Use ATP, Biotin and CO2 (Mn- ABC) and also uses Mg2+

•• Hydratase belongs to Enzyme category number: Enzyme InhibitorsQ


4
Type of
Km Vmax
•• Hydroxylase (Mono-oxygenase) E.C. number: 1 Inhibition

•• Hydrolase E.C. number: 3 Competitive Increased Same

Important Information Non-Competitive Same Decreased

Un-Competitive Decreased Decreased


•• Oxidases which do not require copper:
Xanthine oxidase and Sulfite oxidase
186
Cerebellum Quick Revision Notes

Electrophoretic Mobility of Isoenzymes A. Competitive inhibitor


B. Non-competitive inhibitor
•• Isozyme number inversely related to mobility.
i.e. C. Allosteric inhibitor
D. Uncompetitive inhibitor
–– Least number → moves maximum
Exp: In the presence of competitive inhibitor as
–– Highest number → moves least
affinity decrease so Km changes while Vmax remains
→ In five LDH isoenzymes, LDH-1 moves max same.
and LDH-5 moves least
Q. Identify the type of inhibition? (INICET Nov
→ Out of 3 isoenzymes of CK, CK-1 moves 2022)
max and CK-3 moves least

A. Competitive
B. Uncompetitive
C. Non-competitive
D. Suicidal
Properties of Enzymes Exp: In the presence of non-competitive inhibitor
Vmax changes while Km remains same.
•• Increase velocity/rate of reaction
•• ↓ Activation energy
AMINO ACIDS & PROTEINS
•• Do not change the equilibrium of reaction
BASICS OF AMINO ACIDS
•• Do not change the free energy of substrates/
products

PREVIOUS YEAR QUESTIONS


Q. Identify the type of inhibitor in the graph?
(INICET May 2022)

•• All amino acids have one asymmetric Carbon:


BUT 2 Exceptions:
–– No Asymmetric carbon → Glycine
–– 2 Asymmetric carbons → Isoleucine &
Threonine (both are also essential amino
acids)
187
Biochemistry

Important Information Classification of Amino Acids


Q. Which is Semi essential amino acid? Amino Acid Classification Based on Polarity
a. Arginine POLAR POLAR NON – POLAR
CHARGED UNCHARGED AMINO ACIDS
b. Histidine
1. Basic amino 1. Cysteine 1. Aliphatic
Ans. Both are semi essential, but arginine is more acids (Sulhydryl group) amino acids:
essential compared to histidine. So, best answer is
•• ArginineQ 2. OH – containing •• Glycine
arginine- A
•• Lysine amino acids •• Alanine
Q. Which amino acid is essential in children but not •• Histidine •• SerineQ •• Valine
in adults?
2. Acidic •• Threonine •• Leucine
Ans. Histidine amino acids •• Tyrosine •• Isoleucine
•• Glutamate 3. Amides of Acidic 2. Aromatic
Zwitterion or Ampholyte amino acid
•• Aspartate amino acids:
•• Glutamine •• Phenylalanine
•• Asparagine •• Tyrosine
•• Tryptophan
•• 3. ProlineQ
(Imino acid)
•• 4.
Methionine
•• Isoelectric pH (pl): pH at which zwitterion
# Note: There is a controversy for the polarity of
exists & at which protein precipitates out.
Tyrosine and Glycine. According to the question, one has
Protein is insoluble and has minimum buffering
to analyze what should be marked. Once you know it is
capacity.
controversy, you will be able to solve the question.
Important Information
Essential amino acids
•• Acidic amino acids have -ve charge
•• Basic amino acids have +ve charge "Any Help in Learning These Little Molecules
•• In acidic medium, Proteins & AA exist as +vely Proves Truly Valuable"
charged structures A - Arginine
•• Similarly, in basic medium, Proteins & AA exist as -vely
H - Histidine
charged structures
•• Cation → (+) I - Isoleucine
•• Anion → (-) L - Leucine
•• Cathode → (-) T - Threonine
•• Anode → (+)
L - Lysine
M - Methionine
P - Phenylalanine
T - Tryptophan
V - Valine
188
Cerebellum Quick Revision Notes

Products of Amino AcidsQ Selenocysteine and PyrrolysineQ

Amino acid Products •• 21st AA → Selenocysteine → Given by UGA


Phenyl alanine Tyrosine •• 22nd AA → Pyrrolysine → Given by UAG
Tyrosine Melanin, Catecholamines, Thyroxine
•• UGA and UAG are stop codons and usually do not
Tryptophan Niacin, Serotonin, Melatonin code for any amino acid, but by Co–translational
Arginine NO, Creatine, Urea & Ornithine modification, they make Selenocysteine
Ornithine Polyamines like Putrescine, Spermidine & and Pyrrolysine (not by Post-translational
Spermine modification)
Histidine Histamine, Carnosine, Anserine
Glycine Serine, Creatine, Glutathione, Haem,
Porphyrin, Glyoxylate, Choline, Betaine,
Sarcosine

Body & urine odours in various amino acid diseasesQ


Disease Body odour
Rancid cheese/
Isovaleric acidemia
sweaty feet odour
Transamination
PKU Mousy/ Musty •• These are reversible reactions in the first step
MSUD Burnt sugar like
of catabolism of AA
Hawkinsinuria Swimming pool •• Requires B6 /PLP (pyridoxal phosphate)
Multiple Carboxylase deficiency Tom cat •• Common AA involved: Glutamate
Tyrosinemia type I,
boiled cabbage
hypermethioninemia

Tyrosinemia Enzyme defective Another name

Fumaryl Aceto Tyrosinosis/


Type I
acetate HydrolaseQ Hepatorenal
Richner Hanhart
Tyrosine syndrome/
Type II
Transaminase Oculo-Cutaneous
•• Transaminases e.g. SGOT and SGPT
syndrome
PHPP Hydroxylase •• only α-amino group can take part in transamination
/ HPPD (Hydroxy Neonatal
Type III –– Exception is δ(delta) amino group of ornithine
Phenyl Puruvate Tyrosinemia
Dioxygenase)

Important Information
•• AA having max. tendency to bind phosphate →
OH containing amino acid
•• 17 amino acids can take part in transamination
•• AA which is a site for covalent modification
→ OH containing amino acid •• 3 Amino acids that can’t take part in
transamination: Mnemonic: POLYTHENE
•• AA which is involved in O–Glycosidic bonds →
OH containing amino acid –– PO - PrOline
Q. Which AA is involved in N–Glycosidic bonds? –– LY - LYsine
Ans: Asparagine (has CONH which can provide N for –– THENE - THrEoNinE
N-glycosidic bond)
189
Biochemistry

Important Information Structures of Proteins


Transport form of NH3? Name Definition
•• From body and brain – Glutamine 1° Structure Sequence of amino acids
•• From Muscles - Alanine (via Cahill cycle) Obtained from folding of 1° structure
•• α-Helix (Symmetrical / helical
2° structure structure)
•• β-Sheets
•• β-Turns (Proline & Glycine)
Further folding of 2° structure to form
3° structure
a fully folded 3° structure
> 1 polypeptide chain e.g. Hb (four
4° structure
polypeptide chains)

Monomeric protein: Those proteins which have only


one monomer. They do not have Quaternary structure
e.g. myoglobin
Important Information
AA not found in α-helix:
•• Proline and glycine → create ‘bend’ in α helixQ
•• Tryptophan → due to bulky side chain
•• Charged amino acids like Aspartate, Glutamate,
arginine, lysine

Chaperones
•• These are proteins which help in protein folding
Transdeamination: Transamination (peripheral cells) •• e.g. HSP-10 & 70, Calnexin, Calreticulin
+ Oxidative deamination (liver)
•• HSP-60 class is chaperonins
Proteins •• Calbindin is a Ca binding protein, not a chaperone
•• Protein: polymers of amino acids
•• Mostly present in RER

Important Information
•• Bonds in Enzyme-Substrate interactions
(Mnemonic: HHI)
–– H - Hydrophobic
–– H - Hydrogen
–– I - Ionic
–– Sometimes covalent but Never Van der
Note: In fats, double bond is in cis-configuration Waals Forces
•• Bonds in Protein-DNA interactions (Mnemonic:
HIV)
–– H - Hydrophobic
–– I - Ionic
–– V – Van der Waals Forces
–– Never Covalent bond
190
Cerebellum Quick Revision Notes

Features 1° 2° 3° 4°
S~S Hydrophobic
Hydrophobic H-bond
Bond Covalent/ Peptide/ Amide Hydrogen bond
Hydrogen Ionic
Ionic (Mnemonic- HHI)
Functional
Absent Absent Present Present
activity

Retained because peptide


Denaturation Lost Lost Lost
bond is very strong

X-ray crystallography: Best for


Mass spectrometry,Q crystallizable proteins Q
Detection
Edman’s Technique NMR spectrometry: Best for
non-crystallizable proteins

ONE LINERS •• Beta-alanine is seen in Vitamin B5

•• Most water-soluble porphyrin – Uroporphyrin •• At pI, maximum precipitation & minimum


Buffering action
•• Least water-soluble porphyrin – Proto porphyrin
•• Most NP amino acid → Isoleucine
•• 1 gm Hb → produces 35 mg Bilirubin
•• Most Polar amino acid → Arginine
•• In DM, TCA intermediate which is depleted is
OAA (Oxaloacetate) •• Most basic amino acid → Arginine

•• In Hyperammonemia, TCA intermediate which is •• Most acidic AA → Glutamate


depleted is Alpha-ketoglutarate •• THB (Tetra Hydro Biopterin) resembles Folic
•• PHPP hydroxylase is also called as HPPD acid but it is not a vitamin
(4-Hydroxy Phenyl Pyruvate Dioxygenase) •• THB – required for the 3 aromatic AA
•• Homogentisate is also known as Di hydroxy Hydroxylases & NOS (Nitric Oxide Synthase)
Phenyl Acetate •• Segawa syndrome → def. of THB → Parkinsonism
•• Glutathione is a tripeptide made of → Glutamate, like features
Cysteine & GlycineQ •• Canavan disease → deficiency of Asparto
•• Creatine is made from 3 AA→ Arginine, Methionine acylase
& GlycineQ •• Fish odour syndrome → def. of flavin containing
•• Sources of N & C of Urea → CO2, Ammonia, Mono-oxygenase-3
Aspartate •• Edman reagent- PITC- Phenyl Iso-Thio Cyanate
•• Amino Acid linking Kreb’s and Urea cycle → •• Sanger’s reagent → 1-Fluoro 2,4 DiNitro
Aspartate Benzene (FDNB) or dinitrofluorobenzene is a
•• Amino Acid for Purines → Aspartate, Glutamine, reagent used for protein sequencing
Glycine •• While Sanger’s method is used for DNA
•• Amino Acid for Pyrimidines → Aspartate, sequencing using didexoynucleotides
Glutamine •• Vitamin required for interconversion b/w serine
•• Molecule linking Kreb’s and Urea cycle → & glycine → B6 & B9
FumarateQ •• Histidine has max. buffering capacity among all
•• Fibrinopeptide A & B are highly negatively amino acids
charged proteins made of → Aspartate & •• Histidine Side chain - imidazole ring
Glutamate
•• Tryptophan side chain - indole ring
191
Biochemistry

•• Arginine side chain - guanidino group B. Citrulline


•• Proline side chain - pyrrolidine ring C. Aspartate
D. Glycine
•• Derived amino acids: AA which do not have
codons e.g. Ornithine, Citrulline, Hydroxyproline, Exp: Arginine serves as the precursor for the
Hydroxylysine, Homocysteine synthesis of creatinine, nitric oxide (NO), and urea.

Q. Ammonia formed in brain is transported into blood


by conversion to which of following (FMGE Dec
2019)
A. Glutamine
B. Glycine
•• Vit B2 & B6 deficiency will also leads to Vit B3
deficiency C. Cysteine
D. Urea
•• Vit B3 (Niacin): Atypical vitamin (formed in the
body) Exp: Ammonia is detoxified in astrocytes by converting
it to glutamine, which can then be transported to the
•• Atypical vitamins: Vitamin D & vitamin B3 liver for further processing.

•• In carcinoid syndrome, pellagra occurs because Q. Which of the following is not used for protein
too much tryptophan used in the formation of precipitation? (INICET July 2021)
serotonin that very less tryptophan is available A. Heavy metals
for Niacin formation
B. Alcohol & acetone
C. Change in pH other than isoelectric pH
PREVIOUS YEAR QUESTION
D. Trichloroacetic acid
Q. Bends in alpha-helix structure are formed by Exp: Only at isoelectric pH, a protein has no net
which amino acid ? (JIPMER Nov 2017) charge and readily precipitate out of the solution. At
A. Glycine all other pH a protein will remain soluble in solution.
B. Lysine Q. Selenocysteine is present in which of the following
C. Methionine enzymes? (INICET Nov 2022)
D. Glutamine A. Glutathione reductase
Exp: Due to its small side chain, glycine allows for B. Glutathione peroxidase
greater flexibility and can easily accommodate the B. Thioredoxine peroxidase
conformational changes required for bends in the
D. Thioredoxine oxidase
alpha-helix structure.
Exp: Selenocysteine is an essential amino acid
Q. Which amino acid does not include post incorporated into the active site of enzymes such as
translational modification? (AIIMS Nov 2017) glutathione peroxidase
A. Selenocystiene
Q. Levels of D5 – hydroxyl indole acetic acid (5-HIAA)
B. Triiodothyronine in urine are raised: (FMGE June 2022)
C. Hydroxy-proline
A. Carcinoid syndrome
D. Hydroxy-lysine
B. Hartnup’s disease
Exp: Selenocysteine is encoded by a specific codon C. PKU
and is directly incorporated into proteins during
D. Alkaptonuria
translation, without the need for post-translational
modification Exp: Increased levels of 5-HIAA in urine are indicative
of carcinoid syndrome, a condition associated with
Q. Creatinine, NO & urea are synthesized from neuroendocrine tumors.
which, amino acid? (NEET Jan 2019)
Q. Which amino acid is not used for synthesis of
A. Arginine
192
Cerebellum Quick Revision Notes

hormones? (INICET May 2023) Phospholipid (PL)


a) Tyrosine
b) Phenylalanine
c) Tryptophan
d) Glycine
Exp: While tyrosine, phenylalanine, and tryptophan
are precursors for the synthesis of various hormones
such as melatonin, serotonin etc., glycine is not
directly involved in any hormone synthesis.

LIPIDS
Classification of Lipids

Phospholipases: Enzymes which hydrolyse phospholipids


at different places

•• Base alcohol is Glycerol

Cardiolipin - a complex phospholipid

Simple Lipids
•• Phosphatidic acid = Glycerol + 2 FAs + P
•• So, product of hydrolysis of Cardiolipin = 3
Glycerol + 4 FA + 2 P
•• Can be antigenic due to its complex nature.
•• Anti-Phospholipid Syndrome - Occurs due to
Anticardiolipin antibodies. It is a thrombotic
condition and diseased female has history
of recurrent abortions. Anti-phospholipid
syndrome patients give false positive VDRL
test, as they have antibodies against cardiolipin
193
Biochemistry

& VDRL test reagent has Cardiolipin antigen. Categories of PUFAs


•• Barth syndrome: Rare X-linked - so mostly Omega-3 category Omega-6 category
affecting males, Defect in cardiolipin 1. Cervonic acid/DHA 1. γ–Linolenic acid
remodelling, Cardio skeletal myopathy – muscle (DocosaHexaenoic Acid) •• 18 C & 3 =
weakness, cardiomyopathy (enlarged & weak
•• 22 C (docosa) & 6 = •• Source: oil of evening
heart), Short stature, Neutropenia leads to
(hexaenoic) primrose
Recurrent infectionsQ
•• Source: Fish oil 2. Linoleic acid
Phospholipid Types: 2. α-Linolenic acid •• 18 C & 2 =
•• 18 C & 3 = •• Source: Safflower oil
1. Glycero Phospholipids 2. Sphingo Phospholipids
•• Source: Flaxseed oil, 3. Arachidonic acid
• • Parent alcohol: •• Parent alcohol: Soybean oil) •• 20 C & 4 =
Glycerol Sphingosine 3. Timnodonic acid/EPA •• Source: Animal Fats
–– Sphingosine is an (EicosaPentaenoic Acid)
unsaturated amino •• 20 C (eicosa) & 5 =
alcohol (pentaenoic)
•• Source: Fish oil
Sphingo Phospholipids (Alcohol + FA + Phosphate)

Important information
OH OH P Choline
•• DHA
NH2 NH —FA1
○○ is required for brain development of first 2-3
Amide bond
OH OH years of life so, Health drinks are fortified with
DHA.
C18 C18 ○○ Breast milk always contains DHA.
Sphingosine
(18 C amino alcohol) Ceramide
•• α–Linolenic acid is precursor of ω-3 category means
Sphingomyelin
(present in Myelin sheet of brain) if α–Linolenic acid is taken in diet, other two ω-3 FAs
can be made form it in the body.
Glycolipids / Glycosphingolipids •• Linoleic acid is precursor of ω-6 category and can be
used to make other two ω-6 FAs in the body
•• Made of Alcohol (sphingosine) + FA + Carbohydrate
•• Most Essential FA is - Linoleic acid as it can make
Types: arachidonic acid which is required for PGs and
1. Glucosylceramide / Glucocerebroside Leukotrienes synthesis.Q

•• Never found in CNS but always found in extra •• PUFAs which are cardioprotective: ω-3 PUFAs
neural tissues Signs of Essential fatty acid (EFA) deficiency
2. Galactosylceramide/Galactocerebroside •• Generalized scaly dermatitis composed of
•• Ceramide (Sphingosine + FA) + galactose thickened, erythematous, desquamating plaques,
Acanthosis nigricans
•• Always found in CNS
•• Alopecia, Thrombocytopenia
PUFAs or Essential Fatty Acids •• Failure to thrive, Growth retardation and poor
•• ≥ 2 double bonds wound healing
•• Intellectual disability in children
194
Cerebellum Quick Revision Notes

Lipoproteins
Composition of various lipoproteins

Protein/Apoprotein
Lipoprotein Lipid present
present
Chylomicron TG (exogenous) Apo B48
Chylomicron
TG + Cholesterol Apo B48 + Apo E
Remnant
VLDL TG (endogenous) Apo B100
Tangier’s disease / Familial alpha-
VLDL
lipoprotein deficiency / Hypo alpha
Remnant TG + Cholesterol Apo B100 + Apo E
(IDL)
lipoproteinemia
LDL Cholesterol Apo B100 + Apo E •• Mutation in ABCA-1 gene
Apo-A, Apo-C and •• Decreased HDL
HDL Cholesterol ester
Apo-E
•• Cholesterol accumulation in various body tissues
Like in tonsils → Large orange/yellow tonsils →
Q characteristic featureQ
• • Enlargement of throat, Liver, Spleen
(hepatosplenomegaly) and Lymph Nodes
•• Peripheral Neuropathy (Mononeuritis multiplex)

Important Information
•• Exogenous TG is transported to peripheral tissues by
Chylomicron.Q
•• Endogenous TG is transported from liver to
peripheral tissues by: VLDLQ
•• Cholesterol is transported from liver to peripheral
High density lipoproteins (HDL) tissues by: LDL
•• Cholesterol is transported from peripheral tissues to
Function- Reverse cholesterol TransportQ liver by: HDL
195
Biochemistry

Important Information
Clinical Features to tackle hyperlipoproteinemia
related clinical questions
•• Tendon xanthoma → ↑ Cholesterol
•• Eruptive xanthoma → ↑ TG
•• Palmar & Tubero eruptive xanthoma → ↑ Chylo-
remnant & ↑ VLDL remnant
•• Milky plasma → ↑ Chylomicrons
•• Acute pain in abdomen [Acute pancreatitis] → ↑ TG

Ligands on lipoproteins for uptake by liver

Hyper lipoproteinemia (Fredrickson classification)Q


Type Defect LP TG Ch Common names
Lipoprotein Lipase Or Apo C-II Chylo > VLDL ↑ Normal Familial Hyperchylomicronemia
I
defect
LDL Receptor or Apo B100 ↑ LDL N ↑ Familial Hypercholesterolemia
II a

Unknown ↑ VLDL ↑ ↑ Familial combined


II b
↑ LDL hyperlipoproteinemia
apo E ↑ Chylo–remnant ↑ ↑ Broad β diseases / Dys β
III
↑ VLDL remnant lipoproteinemia

•• Neurological deficit present


Important Information
•• Other C/F → failure to thrive, diarrhoea,
Lipoprotein ‘x’
vomiting, and malabsorption of fat
•• Abnormal Lipoprotein found in two conditions:
•• Hematologic manifestations → acanthocytosis
○○ LCAT deficiency
(irregularly spiculated erythrocytes), anemia,
○○ Cholestatic states
reticulocytosis, and hemolysis with resultant
•• Rich in amphipathic lipids (PL & cholesterol) hyperbilirubinemia.
•• Poor in neutral lipids (TG & cholesterol ester)

Abetalipoproteinemia, or Bassen-
Kornzweig syndrome
•• A stands for Absent (or negligible) beta-
lipoproteins - VLDL , IDL and LDL
•• defective assembly and secretion of (apo)
B-containing lipoproteins → Apo B 48 & Apo B Fatty Acid Synthesis
100
•• Mutation in Microsomal Triglyceride Transfer
Protein - MTP (transfers TGs from intestine &
liver to Chylomicrons & VLDL)Q
•• Decreased plasma TGs (not absorbed from
intestine)
•• Defective absorption of fat-soluble vitamins as
chylomicrons not formed
196
Cerebellum Quick Revision Notes

FA synthase complex - Main enzyme of fatty acid & also in muscles but never occurs in liver, due
synthesis: a multienzyme complex & has 2 monomers to absence of Thiophorase in liver. Q
Important Information
Q. FA is synthesized from?
Ans: Acetyl CoA and not malonyl CoA
•• Because extra Carbon of Malonyl CoA is not
getting added in newly synthesized FA
•• 1st enzyme carboxylase adds one CO2 to form
malonyl CoA but 2nd enzyme FA synthase
removes the CO2. So, only CO2 of acetyl CoA
are used.
•• But, Main Donor of carbon for fatty acid
synthesis – Malonyl CoA
Important information
•• Thiolase is a common enzyme for 4 lipid
metabolic pathways:
1. Ketone body synthesis
2. Ketone body utilization
3. Cholesterol synthesis
4. β-oxidation of FA

Types of Hypoglycemia
Ketotic-hypoglycaemia Non-ketotic hypoglycaemia
Von Gierke’s Disease Insulinoma

Alcoholism Any defect in β-oxidation e.g.


MCAD deficiency, Jamaican
vomiting sickness, Carnitine or
CPT-I or CPT-II deficiency
Starvation Defects in KB pathway

Important Information on Lipids


Lipids Functions
•• Role as lung surfactant
Phosphatidyl choline/ •• Deficient in preterm infants
Lecithin (Di palmitoyl with RDS (Respiratory
phosphatidyl choline)
Distress Syndrome) Q
Phosphatidyl •• A structural phospholipid in
ethanolamine/ brain also has a role in blood
Cephalin clotting
•• Role in cell cycle signalling in
Phosphatidyl Serine
relation to apoptosis

•• KB synthesis occurs only in Liver whereas KB


utilization occurs in vital organs (Brain & Heart)
197
Biochemistry

•• Role in signal →Thiophorase


transmission across •• Immediate precursor of acetoacetate → HMG
membrane, CoA
Phosphatidyl Inositol
•• Also serve as a reservoir •• Carnitine def. → Muscle breakdown & pain,
of PG synthesis, as it myoglobinuria, hypotonia, Poor exercise
has Arachidonic acid intolerance, Increased CK-MM
•• Present in myelin sheath has
•• Lanosterol is the first steroid or first cyclic
Sphingomyelin structural role in lipid raft & compound formed in cholesterol synthesis
cell signalling
•• MCAD deficiency → Non ketotic hypoglycemia
•• Its carbohydrate
portion act as antigenic & dicarboxylic acidosis Q
Glycolipid/ determinant for Blood •• LCAD or LCHAD deficiency (Long-chain
Glycosphingolipid group Ag & Cell surface 3-hydroxyacyl-coenzyme A dehydrogenase) →
receptor for cholera, in acute fatty liver of pregnancy Q
tetanus & botulinum.

PREVIOUS YEAR QUESTION


One liners
Q. Active metabolite form in synthesis of fatty acid
•• Cholera toxin binds oligosaccharide portion of
is? (AIIMS Nov 2017)
GM1 ganglioside receptor. Q
A. Acetyl CoA
•• Tetanus and botulinum toxins selectively bind
B. Malonyl CoA
gangliosides of the G1b series, namely, GT1b,
GD1b, and GQ1b. C. Stearate
D. Palmitate
•• Treatment in Gaucher’s → Acid glucosidase →
Imiglucerase, Velaglucerase alpha, Taliglucerase Exp: Malonyl CoA is the active metabolite in the
alpha synthesis of fatty acids as it serves as active acetyl
CoA donor in fatty acid synthesis.
•• First intermediate of LOX pathway → HPETE
(Hydro Peroxy Eicosa Tetra Enoic acid) Q. Major product of fatty acid synthesis is? (AIIMS
•• First step in the synthesis of eicosanoids Nov 2017)
(prostaglandins & leukotrienes) → hydrolysis by A. Acetyl CoA
Phospholipase A2, to release arachidonic acid. B. ATP
•• Primary prostaglandins are PG-E2, PG-D2 & PG- C. Citrate
F2 alpha D. Palmitate
•• Many patients of Retinitis pigmentosa have Exp: Palmitate is the major product of fatty acid
decreased DHA (Docosa Hexaenoic Acid) levels synthesis and serves as the starting point for the
•• Apolipoprotein responsible for Alzheimer dis → synthesis of longer fatty acids
Apo E4
Q. Which of the following is an essential fatty acid?
•• Citrate activates Acetyl CoA carboxylase (FMGE JUNE 2018)
•• Acyl carnitine is the intermediate formed in the A. Citric acid
transport of fatty acids through mitochondrial B. Linoleic acid
membrane via Carnitine.
C. Stearic acid
•• Starting point of ketone body synthesis → D. Palmitic acid
Acetyl CoA
Exp: Linoleic acid is an essential fatty acid that cannot
•• First enzyme of ketone body synthesis → be synthesized by the body and must be obtained
Thiolase from the diet.
•• First enzyme of ketone body utilization
198
Cerebellum Quick Revision Notes

Q. Type I hyperlipoproteinemia is characterized by? Exp: Fatty acid transport through the mitochondrial
(NEET Jan 2019) membrane is facilitated by carnitine, which helps
transport long-chain fatty acids into the mitochondria
A. Elevated LDL
for beta-oxidation.
B. Elevated HDL
C. Elevated lipoprotein lipase Q7. For Coronary artery prevention, which needs to
be given: (FMGE Jan 2023)
D. Elevated chylomicrons.
A. Saturated fatty acid
Exp: Type I hyperlipoproteinemia is characterized by
elevated levels of chylomicrons (large triglyceride- B. Omega 3 FA
rich lipoproteins) due to deficiency of enzyme LPL C. Omega 6 FA
which breaks down TG in chylomicrons. D. Omega 9 FA
Q. Ketone bodies are not utilized by? (JIPMER May 2019) Exp: Omega-3 fatty acids, such as eicosapentaenoic
acid (EPA) and docosahexaenoic acid (DHA), are
A. Brain
recommended for coronary artery prevention due to
B. RBC their beneficial effects on heart health.
C. Heart
Q. Liver produces KB but cannot use KB due to absence
D. Skeletal muscle
of which enzyme: (INICET May 2023)
Exp: Red blood cells do not have mitochondria and,
A. Thiophorase
therefore, cannot utilize ketone bodies for energy.
B. Thiolase
Q. A patient has multiple tendon xanthomas. Serum C. HMG CoA synthase
cholesterol (398 mg/dl) & LDL (220 mg/dl) were found
D. HMG CoA Reductase
to be raised. Statins were given to this patient. What
is the diagnosis? Exp: The absence of thiophorase enzyme (first enzyme
(NEET Sep 2021) of KB utilization) in the liver prevents the utilization
of ketone bodies produced by the liver.
A. Lipoprotein lipase deficiency
B. Familial hypercholesterolemia Q. A patient with C/O muscle weakness came to OPD.
C. Tangier’s disease Doctor found he has cardiomyopathy & diagnosed him
to be a case of Barth syndrome, where patient has
D. Huntington’s disease
mitochondrial dysfunction. What is the defect in this
Exp: The presence of tendon xanthomas, along with patient: (INICET May 2023)
elevated serum cholesterol and LDL levels, suggests a
A. Cardiolipin
diagnosis of familial hypercholesterolemia
B. Lecithin
Q. Eicosanoids are formed from? (INICET May 2022) C. Lysolecithin
A. Arachidonic acid D. Cephalin
B. Platelet aggregation Exp: Barth syndrome is characterized by a defect in
C. 4 fused rings the synthesis of cardiolipin, a phospholipid primarily
D. Arginine found in the inner mitochondrial membrane
Exp: Eicosanoids, such as prostaglandins and
leukotrienes, are derived from arachidonic acid, which MOLECULAR BIOLOGY
is an omega-6 fatty acid
Nucleic Acid
Q. Fatty acid transport through mitochondrial
•• Nucleic Acid (DNA/RNA) = Polymer of
membrane is via? (NEET PG 2022)
Nucleotides
A. Carnitine
•• NucleoTide = Nitrogenase base + Sugar +
B. Acyl carrier protein
Phosphate
C. Cholesterol transporter
•• NucleoSide = Nitrogenase base + Sugar
D. LCAT
199
Biochemistry

•• So, Nucleotides = Nucleoside + Phosphate NITROGENOUS BASES


•• Purine - Adenine (A) & Guanine (G) 1. Purines
•• Pyrimidines – Uracil (U), Thymine (T) and
Cytosine (C)
•• DNA → Bases are A, T, C, G
•• RNA → Bases are A U, C, G

Nitrogenous Base
N–base Nucleoside Nucleotide form

AMP, ADP, ATP, dAMP, dADP,


Adenine Adenosine 2. Pyrimidines
dATP
GMP, GDP, GTP,
Guanine Guanosine
dGMP, dGDP, dGTP
CMP, CDP, CTP, dCMP, dCDP,
Cytosine Cytidine
dCTP
UMP, UDP, UTP
Uracil Uridine
(deoxy forms not available)
dTMP, dTDP, dTTP
Thymine Thymidine
(ribose forms not available)

Ribose to Deoxyribose conversion occur Tip to remember: Thymine matching with Methyl
at Di-Phosphate level Note: Thymine (Pyrimidine) is different from
Thiamine (Vitamin B1)

Source of C & N atoms of Purine and


Pyrimidines

Uracil to Thymine conversion


•• Occurs at the level of Mono-Phosphate i.e.
UMP→TMP
•• CH3 group is added from THF (Tetra Hydro
Folate) Thymine
•• Enzyme involved: Thymidylate Synthase THF
Glutamine O
Uracil to cytosine conversion at tri- 4 CH3
H N3 5
Phosphate level
2 1 6
•• Occurs at level of triphosphate i.e. UTP → CTP O N
Aspartate
•• NH2 group provided by glutamine CO2 H
•• Enzyme involved: CTP Synthetase
Purine Pyrimidine removal of amino group
N1 Aspartate N1, C4, C5, Aspartate •• Important in B & T Lymphocytes
C6 –– if ↑ ADA → Suggestive of TB
N3, N9 Glutamine N3 Glutamine
–– if ↓ ADA → Leads to Severe Combined
Immunodeficiency (SCID)
C4, C5, N7 Glycine C2 CO2
Lesch Nyhan SyndromeQ
C6 CO2 Extra CH3 THF •• Complete deficiency of HGPRT
C2, C8 THF
•• Gout d/t ↑ ↑ Nitrogenous bases → ↑ ↑ Uric acid
•• Self-mutilation d/t ↑ PRPP (neurotoxic)
Important information
Product of catabolism of N-bases
Kelly Seegmiller Syndrome
•• Partial deficiency of HGPRT
Purines Uric Acid (Soluble)
Pyrimidines CO2, NH3, β-Alanine
•• Only gout present but no neurological problems
Thymine CO2, NH3, β-Amino PYRIMIDINE SYNTHESIS
isobutyrate

PURINE SYNTHESIS
1.De novo pathway
•• High energy consuming pathway (15-20 steps)
•• RLE: PRPP glutamyl amido transferase

2. Salvage pathway
•• Less energy consuming pathway (only 1 step)
•• Main enzyme: HGPRT (Hypoxanthine Guanine
Phosphoribosyl Transferase)

PURINE CATABOLISM

OPRT + Decarboxylase
•• Bi-functional enzyme (single protein with 2
enzymatic activities)
Q
•• Deficiency leads to Orotic Aciduria

Orotic aciduria
•• Growth Retardation and developmental delay
–– Neurological defects
Adenosine Deaminase (ADA)
–– Megaloblastic anaemia which is non-
•• Used for conversion of adenosine to Inosine by responsive to B12 or folic Acid Rx
201
Biochemistry

•• Rx: Give only Uridine as other pyrimidines can be Centromere


synthesized from it Chromosome Diagram Example
Position
Centre Metacentric Primitive,
SUGAR
Chr 1,3
1. Ribose

Near the Submetacentric most of human


centre autosome & X
chr

Close to end Acrocentric Y chr


and some
autosomes
(13, 14, 15,
21, 22)
At the end Telocentric Not present
in human
2. Deoxyribose
BARR BODIES
•• Inactive condensed X chromosome
•• Number of Barr Bodies = No. of X chromosomes
-1Q
No. of
Genotype Barr Phenotype
Bodies

Normal Male XY 0 Normal

Normal
XX 1 Normal
female
•• Formed by removal of one O atom of OH at 2 nd
Turner Female with no
position carbon of ribose XO 0
Syndrome Barr body
•• But If oxygen is removed from both 2’ & 3’ Klinefelter’s Male with Barr
position, then it is called 2’-3’ dideoxyribose. XXY 1
Syndrome body
If this is used, it will stop DNA synthesis as
Female with two
free 3’ OH is lost, which is required to add next Super Female XXX 2
Barr bodies
nucleotide in a growing DNA/RNA chain

CHROMOSOMES ABSORPTION OF UV-LIGHT


•• 23 chromosomes = 22 Autosomes + 1 sex •• Due to conjugated double bonds present in the
chromosome ring.
•• Nucleic acids absorb UV light at 260 nm due to
Haploid state (n) Diploid state (2n)
Nitrogenous bases.
•• 23 chromosomes •• 23 pairs of chromosomes
•• This absorption is more for Purines as compared
•• Present in Germ Cells •• Present in somatic Cells to pyrimidines.

Types of chromosome - Depending upon position of •• Amino acids and proteins also absorb UV light
at 280 nm
centromere
–– It is due to aromatic amino acids
–– maximum for tryptophan Q
202
Cerebellum Quick Revision Notes

•• NAD and NADP absorb light at 340 nm Palindrome definition: same sequence on both strands
(when read in 5’ to 3’ direction)
•• Porphyrin absorb at 400 nm (Soret Band)

Due to conjugated
DNA REPLICATION
Absorption of
Molecule double bonds in the Enzymes of DNA replication
light at:
rings
1. Helicase: Causes strand separation, Use ATP,
DNA 260 nm (UV) Nitrogenous bases Creates positive supercoils
Aromatic amino 2. Topoisomerase- Relieve positive supercoils
Proteins 280 nm (UV) acids (maximum by
tryptophan) 3. Single Strand DNA Binding Proteins (SSBs)-
Prevents reannealing by binding single DNA
NAD/NADP 340 nm (UV) Adenine strands in prokaryotes.
Porphyrins 400 nm (Visible) Pyrrole rings
Note: Helicases, Topoisomerases & SSBs
BONDS IN DNA constitute Unwinding proteins
4. Primase
•• Synthesizes the primers (RNA primer) using
DNA as Template
–– 1 primer is required for leading strand
–– Multiple primers are required for lagging
strand

DNA
•• Right-handed
•• ds in both Prokaryotes and Eukaryotes
–– Prokaryotic DNA is circular (two ends are
joined to form closed circle)
–– Eukaryotic DNA is linear (two ends are free) 5. DNA Polymerase III- Synthesizes both leading
•• Eukaryotic nuclear DNA has introns that & lagging strands
prevents mutations 6. DNA Polymerase I
•• But eukaryotic mitochondrial DNA is like •• Removes RNA primers from both leading &
prokaryotic DNA with no introns, so there are lagging strands
more chances of mutation in it.
•• Fills gap only on lagging strand
Palindrome •• Creates a single gap on leading strand
Q. Which of the following is a palindrome? 7. DNA Ligase- Creates 3’-5’ Phosphodiester bond
A. TAAT to join DNA with DNA, Uses ATP & acts only on
lagging strand
B. GGCC
Ans: B
203
Biochemistry

Prokaryotic vs Eukaryotic DNA Polymerase


E. coli (Prokaryotic) Eukaryotic Function

DNA Pol I Nucleus Mitochondria


RNase H RNase H
FEN-1 FEN-1 (Flap Remove primer and fill the gapQ

δ−polymerase (minor role) endonuclease)

DNA Pol II DNA proof reading and repairQ

β DNA repair

γ Mitochondrial DNA synthesis

DNA Pol III ε Leading strand

δ Lagging strand

DNA-G α Primase

Mnemonic for DNA polymerases •• Lεading (γ) → ε-Polymerase


•• gaMMa (γ) → Mitochondrial Q
•• LAδδing → δ-polymerase

Proof reading Repair


Correction •• Correction during synthesis •• Correction after synthesis
Enzymatic activity •• Mostly endonuclease activity
•• 3’ → 5’ Exonuclease activity
•• Sometimes 5’ → 3’ Exonuclease activity
Enzyme in prokaryotes •• DNA Polymerase II •• DNA Polymerase II

•• All Polymerases except α & β •• β polymerase (main)


Enzyme in eukaryotes
polymerase •• ε polymerases (minor role)
Phase of cell cycle •• Occurs in S phase •• Most repairs occur in G1 phase

Somatic cells •• Gap of leading strand is filled by Telomerase

•• Have limited number of divisions due to the gap 4. Telomerase


present in leading strand
•• Protein with RNA attached to it (a
–– The gap left with each cell division leads to Ribonucleoprotein)
telomere shortening, so cell has limited
divisions. •• RNA acts as template on which DNA is
synthesized
TELOMERE •• Not a Ribozyme as RNA do not act as enzyme,
•• Ends of chromosome instead RNA acts as a template to synthesize
DNAQ
•• Has (TTAGGG)n sequence repeated ‘n’ no. of
timesQ •• RNA dependent DNA polymerase (Reverse
Transcriptase)
–– Telomere Shortening is responsible for aging
& death •• Activity increases in cancer & decreases with
aging
Germ cells/Stem cells
•• Have infinite number of divisions
204
Cerebellum Quick Revision Notes

DNA REPAIR
Type of repair Cell cycle phase Damage Cause Disease
Nucleotide excision repair G1 T-T dimers UV radiation damage Xeroderma pigmentosaQ
Spontaneous, heat, IR
G1 mainly but can C → U conversion Rare, MUTYH associated
Base excision repair rays, viral infection,
occur in any phase by deamination polyposis
Nitrous oxide
HNPCC (Hereditary
Mismatch repair G2 Mismatched base Proofreading error Non-Polyposis Colorectal
Cancer)Q

TRANSCRIPTION RNA (see fig)

•• Formation of RNA from DNA as template in •• Sense strand: as it has same sense of direction
nucleus as new RNA (see fig)

•• Enzyme of transcription: DNA dependent RNA


polymerase Q

RNA polymerase Holoenzyme (complete


enzyme)
•• 5 subunits: 2α, β, β’, ω, σ
Subunits Function
α, ω Enzyme assembly
β main catalytic subunit
β’ Template (DNA) binding
σ Initiation of transcription (recognize promoter)

Types of RNA polymerase


RIBOZYMES
1. In Prokaryotes, only single type present
•• Ribozyme means RNA acting as enzyme
2. In Eukaryotes
Eukaryotic RNA polymerase
RNA synthesized
types
Type I All rRNA except 5S RNA
mRNA, miRNA, lncRNA
Type II
few snRNA & snoRNA
tRNA, 5S rRNA,
Type III
few snRNA & snoRNA

Mitochondrial RNA Polymerase Mitochondrial RNA

Note: Telomerase is not a Ribozyme; RNAase H is not


Template / Anti Sense / Non-Coding / a Ribozyme
Minus Strand Codons
•• Template strand: as new RNA is getting •• Nucleotide Triplets e.g. ACG → 3 bases make 1
synthesized using this strand as template codon
Non-Template / Sense / Coding / Plus •• 4 bases are used to make codons: A U C G
Strand •• 43 = 64 codons combinations are possible
•• Coding strand: as it has the same codons as new
205
Biochemistry

Q. If 4 bases make 1 codon, then how many codons are –– Misnomer i.e. name indicates that it helps in
possible? release. However, it does not help in release.
It only recognises the stop codon
Ans: Total 44 = 256 codons are possibleQ
•• Peptidyl transferase releases the polypeptide
•• Out of 64, 3 are stop codons i.e. do not code for from P site
amino acids
1. UAA Important Information

2. UAG Q. How many ATP & GTP are used to add one AA in
the growing polypeptide chain
3. UGA
Ans:
•• So, for 20 AA, 61 codons are present
2 ATPs → for the activation of AA
–– or on an average, for each amino acid 3 codons
are present 2 GTPs → 1 GTP used in elongation for adding AA

–– Each amino acid has more than 1 codon, known → 1 GTP used for translocation
as Degeneracy/Redundancy of codon So, 4 high energy phosphates are used to add one
•• AA that do not show degeneracy (only 1 codon AA in the growing polypeptide chain
present):
Crispr-CAS 9 System
1. Methionine - AUG
•• CRISPR-Clustered Regularly Interspersed
2. Tryptophan – UGG Short Palindromic Repeats

Translation •• CAS-9 – a CRISPR associated endonuclease


–– It will cause a double strand DNA break
Activation of AA/charging of
tRNA - occurs before Initiation •• Cheap and easy method to cut the DNA at
desired place
•• It is an immune system in bacteria against
bacteriophages
–– Memory of bacteriophage infection and
immunity is transferred to future generation
also, So protects many future generations
Amino Acyl tRNA Synthetase from viruses

•• Have 20 isoenzymes (one for each AA) PCR (Polymerase Chain Reaction)
•• Is the only point of proofreading during Used for amplification of DNA
translation
Components of PCRQ
•• Responsible for fidelity/accuracy of protein
synthesis 1. Heat is used for denaturation and 2 strands
separation
Factors in translation in prokaryotes &
2. DNA to be amplified
eukaryotes
3. 2 primers (1 for each strand)
Prokaryotes Eukaryotes
Initiation IF 1, 2, 3 eIF 1, 2, 3, 4F (RL) 4. Enzyme: Taq polymerase [derived from Thermus
aquaticus bacteria]
Elongation EF– Tu, Ts, G eEF 1α, 1β, 1γ, eEF2
Termination RF 1, 2, 3 eRF 5. Substrates: Deoxyribonucleotides
Factors used in translocation: EF-G (Prokaryotic) 6. Mg2+ in Buffer
and eEF-2 (Eukaryotic) Note: Dideoxyribonucleotide is never the component
•• Releasing factor (RF) of PCR
206
Cerebellum Quick Revision Notes

Real Time PCR / Quantitative PCR Disorders caused due to genomic imprinting
•• Normal PCR is End-PCR i.e. only at the end of 1. Prader Willi Syndrome (PWS)Q
whole PCR process, we get the products and do
•• Paternal allele is deleted, and Maternal allele is
the analysis.
imprinted/inhibited means both copies (father
•• Real time PCR displays the amount of DNA and mother) of gene not working giving rise to
synthesized in real time, as it has SYBR-green sign and symptoms
dye , which gives fluorescence when bound to
the synthesized ds-DNAQ 2. Angel Man syndromeQ
•• Maternal copy of allele is deleted, and Paternal
RT-PCR (REVERSE TRANSCRIPTASE PCR)
allele is imprinted/inhibited leading to various
sign and symptoms

MICRO ARRAY/chip

GENOMIC IMPRINTING
•• It is gene inhibition at the level of transcription
Lot of DNA fragments (probes) are placed on a small
Mechanisms of Genomic Imprinting material looking like a chip.
1. DNA methylation- most common mechanismQ
Uses of chip
•• occurs at CG site/CG Island/CpG sites (C & G •• Can detect multiple mutations
together present on same strands)
•• Can do multiple gene expression analysis
•• cytosine is usually methylated in CG site that
causes inactivation of the gene •• Can do comparative genomic hybridization
•• Can detect SNPs (single nucleotide
polymorphisms)

Limitation
•• Can’t detect aneuploidy (monosomy & Trisomy)

Karyotyping
•• Best technique for detecting monosomy &
•• DNA methylation can be detected by Na- trisomy
bisulfite method •• Limitations
2. PTM of Histones (Post Translational Modifications) –– Lengthy (culture of cell required)
•• Histone Deacetylation –– It can only be done in metaphase arrest
•• Histone Methylation –– Cannot detect micro deletions, amplifications
•• Method to detect PTMs: Chromatin Immuno and complex translocationsQ
Precipitation (ChIP)
Fish (Fluorescent In Situ-
Hybridization)
•• Advantages
207
Biochemistry

–– Can detect aneuploidy If there is no male to male


X-linked
–– Can be done in any phase of cell cycle transmission
More males are affected,
–– Rapid (result in 24 hours)
and affected son born to X-linked recessive
–– Tells gene location on chromosomes unaffected mother (carrier)
–– Can detect microdeletion, amplification and
complex translocationQ KLENOW FRAGMENT
Classical /Mendelian Inheritance It is the larger fragment towards the ‘C’ terminal in
Disorders DNA Polymerase I, which is lacking 5’ → 3’ exonuclease
activity
AD (Autosomal Dominant)
•• Familial hypercholesterolemia

AR (Autosomal Recessive)
•• Most of biochemical enzyme defects
•• All MPS disorders except Hunter
•• All urea cycle disorder’s except OTC
•• All sphingolipidoses except Fabry’s disease
•• All amino acid disorders e.g. PKU, Albinism, HCU
etc Severity of damage in point mutation
•• All glycogen storage disorders Frameshift > Non-Sense > Missense

•• Orotic aciduria •• Frameshift Mutation: when insertion or deletion


is not in multiple of 3
•• Wilson’s disease
–– Whole codon reading frame is shifted leading
•• ADA deficiency to misreading of entire gene
•• Hemochromatosis •• Missense mutation: AA changed

X-linked Recessive •• Non-sense Mutation: Insertion of stop codon

•• Menke’s Disease Methylation has role in DNA


•• Lesch Nyhan Syndrome •• DNA Replication & Transcription
•• Mismatch repair
•• G6PD deficiency
•• X-chromosome inactivation
Important pointers to determine inheritance
•• Chromatin remodelling or chromosome
patterns segregation (heterochromatin formation)
If both male & female
RNA
are affected with equal Autosomal disorder
frequency •• Splicing, Alternate splicing and regulation of
splicing
If atleast one parent is
Dominant disorder
affected •• 7-methyl guanosine cap in mRNA
If neither of parent is •• Gene regulation or regulation of gene
Recessive disorder
affected expression - Gene silencing & Genome imprinting
If all sons are affected Y-linked (Epigenetics)
If affected mother to all •• Others: Embryonic development, Maintenance of
Mitochondrial Inheritance genome stability, Carcinogenesis, Atherosclerosis,
offspring
ageing, Exercise, B-cell differentiation
208
Cerebellum Quick Revision Notes

Gene transfer methods: ONE LINERS


•• Transformation – transfer of naked DNA •• We always read in 5’ to 3’ direction.
from environment or dead bacteria to a living
bacterium •• Synthesis of DNA & RNA occurs in 5’ to 3’
direction.
•• Transfection – transfer of DNA into eukaryotic
cell deliberately •• Centromeres are made up of satellite DNA
repeats.
•• Transduction – transfer of DNA & RNA through
virus, called bacteriophage •• Banding technique for dicentric chromosomes-
C-bandingQ
•• Conjugation – by cell to cell contact, formation
of pilus in donor bacteria to recipient bacteria •• TATA box is rich in A & T
•• Lipofection – when liposome used (liposome can •• Shine Dalgarno is rich in A & G
carry DNA or drugs)
•• Telomeres are rich in T & G
•• Rifampicin inhibits Prokaryotic RNA Polymerase
•• Amanita phalloides – a wild mushroom, also
known as mushroom death cap, produces toxin:
alpha-Amanitin, which inhibits Eukaryotic RNA
Polymerase
•• Deamination of methylated cytosine will form →
ThymineQ
•• Apo B48 is synthesized from ApoB 100 gene
Operon in Prokaryotes in intestine by process: RNA editing (Post
transcriptional modification)Q
Lac operon Tryptophan operon
•• Vitamin deficiency which can lead to the
•• Inducible (Lactose •• Repressible (Trp is co-
presence of Uracil in the genetic code- FolateQ
is the inducer) repressor)
•• Catabolic system •• Anabolic system •• Methotrexate (an antimetabolite) inhibits
enzyme dihydrofolate reductase, so it inhibits
•• Usually off •• Usually on (inactive when
the synthesis of the purines and pyrimidines
high trp is present)
Rasburicase
•• 3 structural genes- •• 5 structural genes (A, B,
Z, Y & A C, D & E) –– a recombinant form of an enzyme, urate
oxidase.
•• Repressor protein •• Repressor protein is
is active inactive –– rapidly reduces existing hyperuricemia in
•• Turned on when •• cAMP is not necessary tumor lysis syndrome.
glucose decreased, here
–– It converts hypoxanthine and xanthine into
cAMP increased &
allantoin, a more soluble molecule, easily
Lactose present
cleared by kidneys.
•• Werner syndrome/Adult Progeria: patients
Important Information have accelerated telomere shortening with
•• DNA replication & repair occurs in 5’ to 3’ damaged DNA & loss of helicase, so premature
direction. ageingQ
•• DNA proofreading occurs in 3’ to 5’ direction.
•• Transcription occurs in 5’ to 3’ direction. PREVIOUS YEAR’S QUESTION
•• RNA Editing is directional, from 3’ to 5’, along Q. Which of the following does not favor permissive
an unedited RNA.
euchromatin due to changes occurring at cytosine
•• tRNA & rRNA are stable, so no cap or tail is residues at CpG islands in DNA? (AIIMS May 2018)
required.
A. Methylation
209
Biochemistry

B. Phosphorylation Exp: Poly A polymerase is an enzyme involved in the


C. Alkylation addition of poly(A) tails to the 3’ end of messenger
RNA (mRNA) molecules. It is not a ribozyme but is a
D. Sumoylation
protein enzyme.
Exp: Methylation of cytosine residues at CpG islands
in DNA leads to the formation of heterochromatin, Q. In CRISPR – Cas 9 System, which repair mechanism
which is associated with gene silencing rather than is used for genome editing? (NEET Nov 2019)
permissive euchromatin. A. Non homologous end repair
Q. The enzyme deficient in Lesch Nyhan syndrome B. Homologous repair
is? (NEET May 2018) C. Mismatch repair
A. Adenosine Deaminase D. Nucleotide excision repair
B. PRPP synthetase Exp: The CRISPR-Cas9 system most often uses the
C. HGPRTase non-homologous repair mechanism for genome editing,
where a desired DNA sequence is inserted using a
D. Xanthine oxidase
template.
Exp: HGPRTase, or hypoxanthine-guanine
Phosphoribosyltransferase (main enzyme of Q. Restriction endonuclease will act on which of the
salvage pathway) is the enzyme deficient in Lesch- following? (AIIMS Nov 2019)
Nyhan syndrome. A. AAGCTT
Q. Which is incorrect about DNA polymerase - I? B. AAGAAG
(NEET May 2018) C. TACGAG
A. Not required in bacteria D. GAGAGG
B. Role in primer removal Exp: Restriction endonucleases cleave DNA at
C. Fill gaps between okazaki fragments palindromic sites (sites with same DNA sequence
on both strands when read in 5’-3’ direction), and
D. Involved in DNA replication
AAGCTT is a palindromic site.
Exp: DNA polymerase I is required for various
functions in bacteria, including primer removal, Q. Amino acid which absorbs UV light at 280 nm?
filling gaps between Okazaki fragments during DNA (NEET Jan 2020)
replication A. Tryptophan
Q. True about DNA polymerase III? (NEET May B. Histidine
2018) C. Aspartate
A. Required for translation D. Ornithine
B. Has DNA repair function Exp: Tryptophan absorbs UV light at a wavelength of
C. Forms okazaki fragments & needs RNA primer 280 nm, which is commonly used to quantify proteins
using the absorbance at this wavelength.
D. Has no proof-reading activity
Exp: DNA polymerase III is responsible for Q. PCR steps are? (INICET Nov 2020)
synthesizing the leading and lagging strands during A. Denaturation, annealing, elongation
DNA replication, including the formation of Okazaki
B. Annealing, denaturation, ligation
fragments, which require an RNA primer.
C. Ligation, denaturation, annealing
Q. Which of the following is not a Ribozyme? (NEET D. Denaturation, annealing, elongation, hybridization
Jan 2019)
Exp: The steps of PCR (polymerase chain reaction)
A. Transpeptidase include denaturation of DNA, annealing of primers,
B. Ribonuclease and elongation by DNA polymerase.
C. Peptidyl transferase
Q. A patient has swelling in MCP joints. Serum uric
D. Poly A polymerase acid levels were raised. Doctor will prescribe medicine
against which enzyme? (FMGE Aug 2020)
210
Cerebellum Quick Revision Notes

A. Thymidylate synthase D. Gene body


B. Xanthine oxidase Exp: The binding site of miRNA (microRNA) on mRNA
C. ADA (Adenosine Deaminase) is typically located in the 3’ untranslated region (UTR)
D. HGPRT of the mRNA.

Exp: In a patient with swelling in MCP joints and Q. Which of the following methods uses RNA?
elevated uric acid levels, the enzyme xanthine (INICET July 2021)
oxidase, which converts hypoxanthine and xanthine to
A. Western blot
uric acid, is targeted for treatment.
B. RT-PCR
Q. Mother side uncle has disease... Her son has C. Sangers method
disease. Which type of inheritance is this? (FMGE
D. G-banding
Aug 2020)
Exp: Reverse transcription polymerase chain reaction
A. X-linked recessive
(RT-PCR) is a method that uses RNA as a template
B. X-linked dominant to synthesize complementary DNA (cDNA), which is
C. Autosomal dominant then amplified by PCR.
D. Autosomal recessive
Q. Deamination of methylated cytosine will form?
Exp: In X-linked recessive inheritance, there is (INICET May 2022)
no male to male transmission, but more males are
A. Uracil
affected, and affected son is born to unaffected
mother (carrier). B. Guanine
C. Adenine
Q. All are true about telomerase EXCEPT? (INICET
D. Thymine
Nov 2020)
Exp: Deamination of methylated cytosine results in
A. has reverse transcriptase activity
the conversion of cytosine to thymine.
B. Maintains fidelity of DNA replication
C. Maintains length of DNA Q. Banding technique used for dicentric chromosomes?
(INICET May 2022)
D. Found only in eukaryotes
A. G
Exp: While telomerase has an important role in
maintaining the length of telomeres and compensating B. NOR
for their shortening during DNA replication, it does C. C
not maintain the fidelity of DNA replication which is D. R
mainly done by DNA pol I and III.
Exp: C banding stains heterochromatin which are regions
Q. RNAi acts through? (INICET July 2021) of the chromosomes at or near centromere. So, C banding
will be most useful for dicentric chromosomes.
A. Knock out
B. Knock down Q. DNA-Protein interactions can be studied by using?
C. Knock in (INICET May 2022)
D. Knock up A. DNA fingerprinting

Exp: RNA interference (RNAi) acts by reducing or B. DNA footprinting


“knocking down” the expression of specific genes by C. Northern Blotting
degrading mRNA produced from that gene. D. ELISA

Q. Binding site of miRNA on mRNA? (INICET July Exp: DNA footprinting is a technique used to study
2021) DNA-protein interactions by identifying regions of
DNA protected by protein binding.
A. 5’ UTR
B. 3’ UTR Q. dd-NTPs in Sanger’s Sequencing technique uses?
C. Gene promotor (INICET May 2022)
A. Its fluorescence
211
Biochemistry

B. DNA polymerization D. GUAGAUC


C. Termination of polymerase Exp: The base sequence of the RNA product
D. Removal of primer synthesized from the given DNA template sequence
GATCTAC would be GUAGAUC, which is same as base
Exp: ddNTPs (dideoxynucleotides) lack the 3’-OH
sequence of non-template strand except that all Ts
group required for the formation of phosphodiester
are replaced with U.
bonds, leading to termination of DNA polymerization
during Sanger sequencing by not allowing DNA Q. Although Apo B 48 and Apo B 100 are derived from
polymerase reaction. the same gene but are finally translated into separate
proteins. This is mainly due to ? (FMGE Jan 2023)
Q. DNA packing is done by? (NEET 2022)
A. DNA gene mutation in intestines
A. Histone
B. Point mutation in liver
B. Glycoprotein
C. DNA gene splicing
C. Nucleic acid
D. RNA editing
D. Helicases
E. Cytochromes Exp: The Apo B mRNA undergoes RNA editing in the
intestine, where a cytosine (C) is deaminated to uracil
Exp: DNA packing is primarily done by histones, which (U), resulting in the synthesis of Apo B48 protein
are proteins that form complexes with DNA to create instead of Apo B100.
a condensed structure called chromatin.

Q. When a codon is changed to stop codon, then it is: MISCELLANEOUS


(NEET 2022)
Respiratory QuotientQ
A. Stop codon mutation
B. Non-sense mutation Amount of CO2 Produced
C. Mis sense Mutation RQ =
Amount of O2 Utilized
D. Silent Mutation
RQ values
Exp: When a codon is changed to a stop codon, it
is referred to as a non-sense mutation, leading to •• Glucose (Carbohydrates) = 6/6 = 1
premature termination of protein synthesis.
•• Proteins = 0.8
Q. True about telomeres and telomerase is: (NEET •• Fats = 0.7
2022)
•• Mixed diet = 0.85
A. Increased telomere length with ageing
•• Alcohol = 0.66
B. Gradually decreasing length of telomeres with
ageing •• Excessive/High Carbohydrate diet > 1 (1.2).
C. Telomeres have coding sequences •• RQ decreases in Diabetes as RQ of fats is
D. Telomerase is found in somatic cells less than the RQ for carbohydrates, But on
giving Insulin, RQ again rises
Exp: Telomeres, the protective caps at the ends of
chromosomes, tend to shorten with each cell division RQ decreases in RQ increases in
and are associated with aging. •• Fasting •• Exercise
•• Starvation •• Fever
Q. The base sequence of the strand of DNA used as
•• Diabetes •• Acidosis
the template for transcription has the base sequence
GATCTAC. What is the base sequence of RNA •• Alkalosis
product? (INICET May 2023)
Collagen
A. CUAGAUG
•• Most abundant human protein
B. GTAGATC
•• 1° structure:
C. GTAGATC
212
Cerebellum Quick Revision Notes

–– (Glycine - X - Y) n i.e. every 3rd amino acid is Copper (Cu) as cofactor, so Cu deficiency also
GlycineQ leads to decrease strength in collagen
–– X, Y can be Proline, Hydroxyproline, Lysine,
Plasma Proteins
Hydroxylysine
1. Transthyretin/ Pre-albumin
•• 28 types found in body
•• Used in transport of thyroxine & Retinol Binding
Important Collagen typesQ Protein (RBP)

Type Mnemonic Location


2. Ceruloplasmin

I S Skin (most abundant) •• Present in plasma and help in transport of Cu

II C Connective tissue •• It is synthesized in liver as apoceruloplasmin


(Apo Cp)
III A Arteries and CVS (healing and
granulation tissues) •• Apo cp binds 6 Cu atoms & forms active
IV B Basement membrane of Glomerulus ceruloplasmin (Cp)
VII Junction of dermis & epidermis •• So, it carries Cu from liver to peripheral
tissuesQ
Disease of collagen •• It also has Ferro-oxidase activity and converts
1. Alport Syndrome Fe+2 to Fe+3 which is required for iron transport
in body.
•• Collagen Type IV is defectiveQ
•• Clinical features are: Hematuria and End stage Menke and Wilson Disease
renal disease
Difference between Menke’s and Wilson’s
2. Epidermolysis bullosa Disease
•• Collagen Type VII is defectiveQ Menke’e Disease Wilson’s disease

•• Clinical features are skin blisters •• Cu Deficiency •• Cu Excess


•• ATP-7A protein •• ATP-7B protein defective
Post Translational Modifications (PTM) defective
of Collagen •• X-linked recessive •• Autosomal recessive

1. Hydroxylation of proline and lysine residues, to •• ATP-7A is present •• ATP-7B is present in liver
increase H-bonds in intestine for to throw Cu in bile and
absorption of Cu. Its also incorporates Cu in Cp.
deficiency causes: Its deficiency causes:
○○ ↓ Cu (Cu stays ○○ ↑ Cu in liver (due to
in intestinal cell, non-excretion from
unable to enter body)
blood) ○○ ↓ ceruloplasmin
○○ ↓ ceruloplasmin (Cp) (due to non-
(Cp) (due to incorporation of Cu in
insufficient Cu to Cp)
Clinical Significance: The deficiency of vitamin C will form Cp)
hinder these reactions resulting in fragile collagen of
blood vessels of gums which can be easily bruised and Note: ↓ Ceruloplasmin (Cp) is common between Menke
bleed during brushing causing scurvy. and Wilson Disease
1. Glycosylation Menke’s kinky hair syndrome- Clinical
•• Addition of carbohydrate in collagen which features
helps in making Aldol condensation
•• Premature birth
•• Enzyme required is Lysyl oxidase which require
•• Hypotonia
213
Biochemistry

•• Growth retardation
•• Mental retardation
•• Grey depigmented hair:
–– Tyrosinase (an oxidase) is affected as
oxidases require Cu
–– Tyrosinase is required for synthesis of
Melanin.
–– So, melanin synthesis is decreased causing
grey hair
•• Brittle kinky hair
–– As lysyl oxidase affected, so defective and
weak collagen synthesis
•• Decreased Cu in blood and urine

•• Treatment: Give penicillamine – as it will chelate


Cu

Vitamins
Water soluble Fat soluble
•• Vit B, vit C •• Vit A, D, E, K
•• Usually acts as •• Stored in body & don’t act
coenzyme & are not as coenzyme
Wilson’s hepatolenticular degeneration stored in body •• Exception: Fat soluble
•• Exception: One Vitamin which act as
Clinical features water-soluble vit coenzyme - Vit K
•• Cu accumulates in liver causing liver damage which get stored in
body - Vit B12
•• When excess in liver, ↑ Cu can go to other
extrahepatic tissues and cause following
changes: Important Information

–– In Brain – Neurological degeneration Q. Water Soluble form available for which fat-
soluble vitamin?
–– In Kidneys- Renal damage, urolithiasis
Ans: Vit K - Synthetic form – K3 /menadione is water
–– In Bone marrow & RBC - Hemolytic anemia
soluble
–– In Eyes -
→ Sunflower cataract
→ Kayser-Fleisher (KF) rings - green/golden
ring in Descemet’s membrane of cornea due
to Cu deposition
214
Cerebellum Quick Revision Notes

One Liners WHO Vitamin A deficiency grading

Vitamin Deficiency Anemias Clinical Feature Grading


Night blindness XN
Microcytic hypochromic Megaloblastic anemia Conjunctival xerosis X1A
anemia
Bitot’s spots X1B
•• Vit C - help in •• Vit B9 - No neurological
absorption of Fe symptoms Corneal xerosis X2

•• Vitamin B6 - coenzyme •• Vit B12 -progressive Corneal ulcer covering less than 1/3 of the X3A
of heme synthesis enz peripheral neuropathy corneal surface
ALA synthase Corneal ulcer covering greater than 1/3 of X3B
the corneal surface
Important Information Corneal scarring XS

•• Raw eggs have protein called avidin, which Xerophthalmic fundus XF


tightly binds to biotin causing its deficiency
by not allowing its absorption. Important Information
Most sensitive screening for Vitamin A deficiency:
•• Biotin is required for carboxylases so it
Blood concentrations of retinol in plasma or serum, which
will affect gluconeogenic enzyme pyruvate
is the main circulating form of Vitamin A. This can assess
carboxylase causing hypoglycemia.Q
subclinical vitamin A deficiency.
Vitamin A
•• Active form: Retinol Vitamin A toxicity
•• Main C/F is raised Intracranial tension
resembling Brain Tumour aka Pseudo Tumour
Cerebri

Vitamin D

•• Active form: Calcitriol or 1,25 dihydroxy


cholecalciferol
–– Three -OH groups present at positions 1,3
215
Biochemistry

and 25 in cholesterol Vitamin K


Vitamin D deficiency Biochemical role: Blood clotting by activation of
clotting proteins 2 (II), 7 (VII), 9 (IX), and 10 (X) by
Deficiency in Children – RicketsQ carboxylation of their glutamate residues.Q
•• Beaded appearance of RIBs aka Rachitic Rosary
•• Hot cross bun appearance of head due to non-
closure of fontanelle
•• Bowlegs and Knock Knees
Deficiency in adults – Osteomalacia
•• Soft bones with frequent fracture
•• Waddling gait

Hypervitaminoses D
Symptoms
•• Hypercalcemia
•• Hypercalciuria causing Renal stones
•• Calcification of Soft tissues: calcium deposited Vit K deficiency
in blood vessels causing hypertension
•• Easy bruising
Vitamin E •• Bleeding tendencies
•• Most potent lipid phase antioxidant (chain •• ↑ Prothrombin time
breaking antioxidant)Q
•• Haemorrhagic disease of new-bornQ
•• Other antioxidant vitamins: Vit C, A and D
–– Fatal and occurs quite often, so, all new-borns
•• Vit E work synergistically with Se, Glutathione are given vit K injection
and Vit C for its antioxidant role
Important Information
C/F of Vit E Deficiency Reasons of Haemorrhagic disease of newborn
•• progressive peripheral neuropathy •• Intestine of a new-born is sterile (i.e. intestinal
bacteria for Vit K synthesis not present)
•• Hemolysis Q

•• Poor placental transfer


•• No megaloblastic anaemia
•• Low Vit K content in early breast milk (colostrum)
•• Opthalmoplegia •• Hepatic Immaturity - Inadequate synthesis of
coagulation proteins

Difference between Vit K and Vit C deficiency


Vit K deficiency Vitamin C deficiency
•• ↑ Prothrombin time •• ↑ Bleeding time
•• Associated with Fat malabsorption and Long-term •• Associated with dietary deficiency of citrus fruits and
antibiotic therapy fresh vegetables
216
Cerebellum Quick Revision Notes

ONE LINERS
cGMP as second messenger inQ
•• Vitamin A visual cycle
•• NO induced vasodilation effects
•• Activation of ANP and BNP
Lipid deficiency causing retinitis pigmentosa: DHA (DocosaHexaenoic Acid)

Best method to determine HbA1c – Ion exchange chromatographyQ

Best Investigation for any inborn error of Metabolism (IEM) - Mass spectrometryQ

Buffer is most effective when its: pH = pKaQ


Maximum buffering zone of a buffer at: pH = pKa ± 1
Minimum buffering when pH = pI
Major extracellular buffer – Bicarbonate buffer
An ideal buffer as both components (HCO3- / CO2) can be altered for blood pH maintenance
Major intracellular buffer/urinary buffer – Phosphate buffer
Vitamin C cannot be formed in humans due to absence of enzyme: L-Gulono Lactone Oxidase
Vitamins synthesized by bacterial flora - Vitamins B2, B5, B7, and Vit KQ
Atypical vitamins – Vit that can be synthesized in our body by human enzymes e.g. Vitamin D and Niacin (B3)
Vitamins which is not present in vegetarian diet – Vitamin D & vitamin B12
Vitamin def with neurological symptoms – B1, B3, B6, B12 and Vit E.
Note: both B12 and E have peripheral Neuropathy but there is always megaloblastic anemia in B12 def whereas Vit E
def causes hemolysis
Site of absorption:
Vitamin B9 – Jejunum and duodenum
Vitamin B12 – Ileum (requires intrinsic factor def of which causes pernicious anemia)
Fructosamine (which is used for assessing long term glycemic control in a person with hemolysis) is a glycated product
of Albumin.
Alcohol is called Spurious liquor or Hooch, when the amount of methanol added is high enough that it can turn fatal
due to increased production of Formic acid, which leads to Metabolic acidosis, blurred vision, blindness, neurological
damage, respiratory failure & death. This was the reason for Hooch Tragedy.
•• Two B-complex vitamins which causes toxicity are:
1. B3 (Flushing, nausea, vomiting, glucose intolerance, hyperuricemia, hepatic toxicity, macular edema & cysts)
2. B6 (sensory neuropathy)
•• DASH diet - Dietary Approach to Stop Hypertension
○○ Poor in Na
○○ Increased K, Ca, Mg (Fruits, whole grains & Vegetables)

PREVIOUS YEAR’S QUESTION Exp: Type I collagen is the most abundant collagen
type in the human body, including the skin. It provides
Q. Type of collagen Maximum in skin? (NEET Jan structural support and tensile strength to the skin
2019)
Q. Defect in Menke disease? (NEET Jan 2019)
A. Type I
A. Lysyl hydroxylase
B. Type II
B. Lysyl oxidase
C. Type III
C. Prolyl hydroxylase
D. Type IV
D. Prolyl oxidase
217
Biochemistry

Exp: Menke disease is a genetic disorder characterized D. 1.25


by impaired copper absorption leading to def of Cu
Exp: Respiratory quotient (RQ) is the ratio of
which is a cofactor for enzyme lysyl oxidase. So,
carbon dioxide produced to oxygen consumed during
a decreased activity of this enzyme is seen in this
metabolism. RQ is calculated by dividing the volume of
disease.
CO2 produced (200 ml) by the volume of O2 consumed
Q. Most potent lipid phase antioxidant? (FMGE June (250 ml), resulting in an RQ of 0.8.
2019)
Q. Which factor in warfarin therapy, have decreased
A. Vitamin A gamma carboxylation of glutamate residue? (NEET
B. Vitamin E Jan 2020)
C. Vitamin C A. Factor II
D. Vitamin K B. Factor V
Exp: Vitamin E is a potent antioxidant that protects C. Factor VIII
cell membranes from oxidative damage by lipid D. Factor III
peroxidation. It scavenges free radicals in the
Exp: Warfarin inhibits the enzyme vitamin K epoxide
lipid phase, preventing the propagation of oxidative
reductase (VKOR), which is responsible for the
reactions.
conversion of vitamin K epoxide to its reduced form,
Q. Alcoholic patient with neurological Symptoms, vitamin K hydroquinone. This reduced form is required
there is deficiency of which Vitamin? (FMGE Aug for the gamma carboxylation of glutamate residues in
2020) coagulation factors, including Factor II (prothrombin).

A. B1 Q. Vitamin D synthesis sequence is? AIIMS June 2020)


B. A A. Skin → liver → kidney
C. B9 B. Kidney → Live → Skin
D. B12 C. Skin → kidneys → liver
Exp: Chronic alcohol consumption can lead to thiamine D. Live → Skin → kidney
deficiency, which can manifest as neurological
Exp: Vitamin D synthesis begins in the skin, where
symptoms such as Wernicke-Korsakoff syndrome.
sunlight converts 7-dehydrocholesterol to previtamin
Thiamine is essential for normal brain function and
D3. Previtamin D3 is then converted to vitamin D3
energy metabolism.
in the liver. Finally, in the kidneys, vitamin D3 is
Q. Vitamin deficiency related to neonatal seizures? converted to its active form, calcitriol.
(AIIMS June 2020)
Q. Methyl tetrahydrofolate accumulation and
A. Thiamine functional deficiency of folate is seen in the deficiency
B. Pyridoxine of: (INICET 2021)
C. Riboflavin A. Folic acid
D. Pantothenic Acid B. Vit B12
Exp: Neonatal seizures can be associated with C. Vit B1
pyridoxine (Vitamin B6) deficiency. Pyridoxine is D. Vit B6
required for the synthesis of neurotransmitters and
Exp: Methyl tetrahydrofolate is the active form of
proper functioning of the nervous system.
folate, and its accumulation occurs in vitamin B12
Q. A person of 50 kg weight, his CO2 & O2 levels deficiency. Vitamin B12 is required for the conversion
were measured from spirometry as CO2 = 200 ml, O2 of methyl tetrahydrofolate to tetrahydrofolate.
= 250 ml. What is RQ in this case? (INI CET Nov
Q. Which of the following supplement is required
2020)
more for a lactating woman than the pregnant female?
A. 4 (INICET 2021)
B. 5 A. Vit A
C. 0.8
218
Cerebellum Quick Revision Notes

B. B12 4. Albumin - b. Free fatty acids


C. Calcium
Q. Fructosamine (which is used for assessing long
D. Thiamine term glycemic control in a person with hemolysis) is a
Exp: Lactation increases the demand for various glycated product of: (INICET Nov 2022)
nutrients, including vitamin A. Therefore, a lactating A. Glycosaminoglycan
woman requires a higher amount of vitamin A compared
B. Urea
to a pregnant female.
C. Fructose
Q. Sideroblastic anemia can be treated by? (INICET D. Albumin
July 2021)
Exp: Fructosamine is a glycated product formed
A. Pyridoxine by the non-enzymatic glycation of serum proteins,
B. B1 primarily albumin. It is used to assess long-term
C. B3 glycemic control in individuals with conditions such as
hemolysis that can interfere with traditional glycated
D. B12
hemoglobin (HbA1c) measurements
Exp: Sideroblastic anemia is a group of blood
disorders characterized by impaired iron utilization Q. Hooch tragedy is related to which of the following
in the production of hemoglobin. Pyridoxine (Vitamin alcohols? (INICET Nov 2022)
B6) supplementation can improve the condition, as it A. Methyl alcohol
is involved in heme synthesis.
B. Ethyl alcohol
Q. Uracil was found in genetic code. Which vitamin C. Ethylene glycol
deficiency leads to this condition: (INICET May 2022) D. Hyporglycin
A. B12 Exp: The term “hooch tragedy” refers to incidents
B. B6 involving the consumption of illicitly produced or
C. Folate contaminated alcohol. Methanol (methyl alcohol)
is commonly associated with such tragedies, as its
D. Vit E consumption can lead to severe toxicity and even death.
Exp: Uracil is converted to thymine by enzyme
Q. A woman on anti-depressants presents with bleeding.
thymidylate synthase by methylation where methyl
She gives a history of bulky stools which sticks to the
group is donated by methyl-THF. So, a deficiency
pan. Which of the following vitamin deficiency can
Folate will result in thymine deficiency and may lead
cause bleeding in this condition? (NEET 2022)
to uracil appearing in the DNA in place of thymine.
A. Vitamin A
Q. Match the following transporters with the
B. Vitamin D
molecule they carry. (INICET May 2022)
C. Vitamin E
1. Hemopexin a. Thyroxin D. Vitamin K
2. Haptoglobin b. Free fatty acids
Exp: Vitamin K deficiency can lead to impaired blood
3. Pre-albumin c. Heme clotting and bleeding tendencies. Bulky stools that
4. Albumin d. Haemoglobin stick to the pan may indicate malabsorption, which can
affect the absorption of fat-soluble vitamins such as
A. 1b, 2a, 3c, 4d vitamin K.
B. 1a , 2c, 3b, 4d Q. Condition shown below is due to which vitamin
C. 1c, 2a, 3d, 4b deficiency? (NEET 2022)
D. 1c, 2d, 3a, 4b

Exp: 1. Hemopexin - c. Heme


2. Haptoglobin - d. Haemoglobin
3. Pre-albumin - a. Thyroxin A. Vitamin A
219
Biochemistry

B. Vitamin B Exp: Scurvy is characterized by defective collagen


C. Vitamin C synthesis due to the deficiency of vitamin C, which is
required for the hydroxylation of proline and lysine
D. Vitamin K
residues in collagen synthesis
Exp: The condition shown in the image appears to
be Bitot’s spot (superficial deposition of keratin Q. A patient was presented to OPD with features of
in conjunctiva) which occur in case of vitamin A collagen vascular disorder. Which of the following
deficiency. deficits is most likely to be associated with defective
collagen formation? (FMGE Jan 2023)
Q. Diarrhea, dementia and dermatitis is caused by
A. Ascorbic acid
deficiency of (NEET 2023)
B. Copper and Iron
A. Biotin
C. Both A & B
B. Niacin
D. Zinc & Magnesium
C. Thiamine
Exp: Post translational modification of collagen on
D. Riboflavin
proline and lysine residue is required for its proper
Exp: Diarrhea, dementia, and dermatitis are classic hydrogen bond formation and crosslinking which
symptoms of pellagra, which is caused by niacin provide strength to collagen fibers. These reactions
(vitamin B3) deficiency. Niacin is important for energy require vitamin C, copper and iron as cofactors for
production and metabolism and deficiency can lead to the enzyme involved in these processes.
various systemic symptoms
Q. Xerophthalmia/ conjunctival xerosis occurs due to
Q. Chronic alcoholic with features of ataxia confusion deficiency of which of the following vitamin: (FMGE
and 6th nerve involvement? Most commonly seen in Jan 2023)
deficiency of? NEET 2023)
A. Vitamin B1
A. Thiamine B. Vitamin B3  
B. Riboflavin C. Vitamin A
C. Niacin D. Vitamin D
D. Cobalamin
Exp: Xerophthalmia and conjunctival xerosis are
Exp: Alcohol impairs thiamine absorption and ocular manifestations of vitamin A deficiency, which
increases its excretion, leading to thiamine deficiency can lead to dryness, night blindness, and eventually
and neurological symptoms. blindness.
Q. Baby with h/o raw egg ingestion. (picture of baby’s
scalp hairloss) deficiency of which vitamin? (NEET
2023)
A. Biotin
B. Thiamine
C. Riboflavin
D. Pyridoxine
Exp: Raw egg white contains avidin, which binds to
biotin and prevents its absorption, leading to biotin
deficiency and hair loss.

Q. The biochemical basis of scurvy is? (NEET 2023)


A. Impaired collagen synthesis
B. Increased keratinization of epithelium
C. Inhibition of clotting factors
D. Low calcium

You might also like